Currently Empty: ₹0.00
Nephrology October 2022 MRCP Part – 1
Time limit: 0
Quiz Summary
0 of 121 Questions completed
Questions:
Information
You have already completed the quiz before. Hence you can not start it again.
Quiz is loading…
You must sign in or sign up to start the quiz.
You must first complete the following:
Results
Quiz complete. Results are being recorded.
Results
0 of 121 Questions answered correctly
Your time:
Time has elapsed
You have reached 0 of 0 point(s), (0)
Earned Point(s): 0 of 0, (0)
0 Essay(s) Pending (Possible Point(s): 0)
Categories
- Nephrology October 2022 MRCP Part – 1 0%
- Neurology Module July 2022 MRCP Part-1 0%
- 1
- 2
- 3
- 4
- 5
- 6
- 7
- 8
- 9
- 10
- 11
- 12
- 13
- 14
- 15
- 16
- 17
- 18
- 19
- 20
- 21
- 22
- 23
- 24
- 25
- 26
- 27
- 28
- 29
- 30
- 31
- 32
- 33
- 34
- 35
- 36
- 37
- 38
- 39
- 40
- 41
- 42
- 43
- 44
- 45
- 46
- 47
- 48
- 49
- 50
- 51
- 52
- 53
- 54
- 55
- 56
- 57
- 58
- 59
- 60
- 61
- 62
- 63
- 64
- 65
- 66
- 67
- 68
- 69
- 70
- 71
- 72
- 73
- 74
- 75
- 76
- 77
- 78
- 79
- 80
- 81
- 82
- 83
- 84
- 85
- 86
- 87
- 88
- 89
- 90
- 91
- 92
- 93
- 94
- 95
- 96
- 97
- 98
- 99
- 100
- 101
- 102
- 103
- 104
- 105
- 106
- 107
- 108
- 109
- 110
- 111
- 112
- 113
- 114
- 115
- 116
- 117
- 118
- 119
- 120
- 121
- Current
- Review
- Answered
- Correct
- Incorrect
-
Question 1 of 121
1. Question
A 30-year-old lady is referred with a four-month history of tiredness and generalized weakness.
_x000D_
On examination, Pulse Rate is 82 Beats Per Minute and Blood Pressure is 130/70 mmHg.
_x000D_
No specific abnormalities are evident on examination of the cardiovascular, respiratory, abdominal, or neurological systems.
_x000D_
Investigations Reveal:
_x000D_
_x000D_ _x000D_
_x000D_ _x000D_ Serum Sodium
_x000D_
_x000D_
_x000D_ 141 mmol/L
_x000D_
_x000D_
_x000D_ (137-144)
_x000D_
_x000D_
_x000D_
_x000D_ _x000D_ Serum Potassium
_x000D_
_x000D_
_x000D_ 3.1 mmol/L
_x000D_
_x000D_
_x000D_ (3.5-4.9)
_x000D_
_x000D_
_x000D_
_x000D_ _x000D_ Serum Urea
_x000D_
_x000D_
_x000D_ 4.4 mmol/L
_x000D_
_x000D_
_x000D_ (2.5-7.5)
_x000D_
_x000D_
_x000D_
_x000D_ _x000D_ Serum Creatinine
_x000D_
_x000D_
_x000D_ 81 µmol/L
_x000D_
_x000D_
_x000D_ (60-110)
_x000D_
_x000D_
_x000D_
_x000D_ _x000D_ Serum Chloride
_x000D_
_x000D_
_x000D_ 75 mmol/L
_x000D_
_x000D_
_x000D_ (95-107)
_x000D_
_x000D_
_x000D_
_x000D_ _x000D_ Plasma Glucose
_x000D_
_x000D_
_x000D_ 5.7 mmol/L
_x000D_
_x000D_
_x000D_ (3.0-6.0)
_x000D_
_x000D_
_x000D_
_x000D_ _x000D_ Urinary Chloride
_x000D_
_x000D_
_x000D_ 64 mmol/L
_x000D_
_x000D_
_x000D_ (20-350)
_x000D_
_x000D_
_x000D_
_x000D_
_x000D_
Which of the following is the likely diagnosis?
CorrectIncorrectHint
This young lady has Hypokalaemia and Hypochloraemia.
_x000D_
The normal blood pressure would exclude a diagnosis of Conn’s Syndrome, Congenital Adrenal Hyperplasia (CAH), or Liddle’s Syndrome (Apparent Mineralocorticoid Excess).
_x000D_
Similarly, drug ingestion associated with Hypokalaemia Liquorice / Carbenoxolone is again associated with Hypertension (and Low Urinary Chloride, less than 20 mmol/L).
_x000D_
Bartter Syndrome is a rare, recessive condition associated with weakness, lethargy, and growth retardation and is found in youngsters.
_x000D_
_x000D_
Hypokalaemic Hypochloareamic Alkalosis is seen in the condition but symptoms would be more apparent at a much younger age than this lady.
_x000D_
If Gitelman’s Syndrome were offered then that would be a better option.
_x000D_
Therefore the most likely diagnosis is Diuretic Abuse, as the symptoms have arisen only over the last four months and no other features are apparent on examination.
_x000D_
-
Question 2 of 121
2. Question
Antidiuretic Hormone (ADH) plays an important role in Osmoregulation.
_x000D_
Which of the mechanisms listed below most accurately describes the action of Antidiuretic Hormone (ADH) on the Kidney?
CorrectIncorrectHint
Antidiuretic Hormone (ADH) plays a crucial role in maintaining the normal concentration of the Serum.
_x000D_
Released by the Posterior Pituitary, it results in the Insertion Of Aquaporin Channels in the Collecting Duct, allowing Water Reabsorption. Antidiuretic Hormone (ADH) does not inhibit Carbonic Anhydrase, nor is it related to Constriction Of Afferent or Efferent Arterioles.
_x000D_
Antidiuretic Hormone (ADH) is not associated with action on the Thick Ascending Limb Of The Loop Of Henle, or in the Distal Tubule.
_x000D_
-
Question 3 of 121
3. Question
In which of the following situations would a Percutaneous Needle Biopsy of the Kidney be most helpful and appropriate?
CorrectIncorrectHint
The Renal Manifestations of Systemic Lupus Erythematosus (SLE) are highly variable, ranging from Mild Asymptomatic Proteinuria and / or Haematuria to Rapidly Progressive Uraemia. The various presentations are difficult to classify into clinical syndromes and histological classes. Although Lupus Nephritis affects a third of patients early in the disease it is frequently unrecognised until Nephritic and / or Nephrotic Syndrome with Renal Failure occur.
_x000D_
Histologically, a number of different types of Renal Diseases are recognised in Systemic Lupus Erythematosus (SLE) with Immune-Complex Mediated Glomerular Disease being the most common. The up to date International Society of Nephrology / Renal Pathology Society 2003 classification divides these into six different patterns:
_x000D_
- _x000D_
- I – Minimal Mesangial.
- II – Mesangial Proliferative.
- III – Focal.
- IV – Diffuse.
- V – Membranous.
- VI – Advanced Sclerosis.
_x000D_
_x000D_
_x000D_
_x000D_
_x000D_
_x000D_
_x000D_
Glomeruli appear normal by Light Microscopy in Minimal Mesangial Lupus Nephritis, but Immunofluorescence demonstrates Mesangial Immune Deposits.
_x000D_
Mesangial Proliferative Nephritis presents clinically as Microscopic Haematuria and / or Proteinuria. Hypertension is uncommon and Nephrotic Syndrome and Renal Impairment are very rarely seen. Biopsy demonstrates Segmental Areas Of Increased Mesangial Matrix and Cellularity, with Mesangial Immune Deposits. A few Isolated Subepithelial or Subendothelial Deposits may be visible by Immunofluorescence. The prognosis is good and specific treatment is only indicated if the disease progresses.
_x000D_
Focal Disease is more advanced, but still affects less than 50% of Glomeruli. Haematuria and Proteinuria are almost always seen, and Nephrotic Syndrome, Hypertension and Elevated Serum Creatinine may be present. Biopsy demonstrates Active or Inactive Focal, Segmental or Global Endocapillary Glomerulonephritis or Extracapillary Glomerulonephritis involving less than 50% of Glomeruli, typically with Focal Subendothelial Immune Deposits, with or without Mesangial Alterations. It is further subdivided:
_x000D_
- _x000D_
- A: Active Lesions: Focal Proliferative Lupus Nephritis
- A/C: Active And Chronic Lesions: Focal Proliferative And Sclerosing Lupus Nephritis.
- C: Chronic Inactive Lesions With Glomerular Scars: Focal Sclerosing Lupus Nephritis.
_x000D_
_x000D_
_x000D_
_x000D_
Prognosis is variable.
_x000D_
Diffuse Glomerulonephritis is the most common and severe form of Lupus Nephritis. Haematuria and Proteinuria are almost always present, and Nephrotic Syndrome, Hypertension and Renal Impairment are common. Biopsies demonstrate Active or Inactive Diffuse, Segmental or Endocapillary Glomerulonephritis or Extracapillary Glomerulonephritis involving less than 50% of Glomeruli, typically with Diffuse Subendothelial Immune Deposits, with or without Mesangial Alterations. This class is divided into Diffuse Segmental (IV-S) when more than 50% of the Involved Glomeruli have Segmental Lesions, and Diffuse Global (IV-G) when more than 50% of Involved Glomeruli have Global Lesions. Segmental is defined as a Glomerular Lesions that involves less than half of the Glomerular Tuft.
_x000D_
- _x000D_
- IV-S (A): Active Lesions, Diffuse Segmental Proliferative Lupus Nephritis.
- IV-G (A): Active Lesions, Diffuse Global Proliferative.
- IV-S (A/C): Active And Chronic Lesions, Diffuse Segmental Proliferative and Sclerosing Lupus Nephritis.
- IV-S (C): Chronic Inactive Lesions With Scars, Diffuse Segmental Sclerosing Lupus Nephritis.
- IV-G (C): Chronic Inactive Lesions With Scars: Diffuse Global Sclerosing Lupus Nephritis.
_x000D_
_x000D_
_x000D_
_x000D_
_x000D_
_x000D_
Immunosuppressive Therapy is required in these cases to prevent Progressive to End-Stage Renal Failure (ESRF).
_x000D_
Patients with Membranous Lupus Nephritis tend to present with Nephrotic Syndrome. Microscopic Haematuria and Hypertension may also be seen. Biopsies show Global or Segmental Subepithelial Immune Deposits or Their Morphologic Sequelae, With or Without Mesangial Alterations. It may occur in combination with Class III or IV, in which case both are diagnosed. Progression is variable, and Immunosuppression is not always needed.
_x000D_
In Advanced Sclerosis, more than 90% of Glomeruli are Globally Sclerosed without Residual Activity.
_x000D_
Features associated with a poorer prognosis, and increased risk of progression to End-Stage Renal Failure (ESRF) include:
_x000D_
- _x000D_
- Young Age (<23)
- Increased Serum Creatinine
- Diffuse Proliferative Lesions (WHO Classification: Class IV), and
- A High Chronicity Index On Renal Histological Analysis.
_x000D_
_x000D_
_x000D_
_x000D_
_x000D_
With regard to the management of Lupus Nephritis, a biopsy is indicated in those patients with Abnormal Urinalysis and / or Reduced Renal Function. This can provide a Histological Classification as well as information regarding Activity, Chronicity and Prognosis. Cyclophosphamide, Mycophenolate Mofetil and Azathioprine reduce mortality in proliferative forms of Lupus Glomerulonephritis.
_x000D_
_x000D_
-
Question 4 of 121
4. Question
In which of the following circumstances would the treatment of Anaemia with Erythropoietin (EPO) still be expected to be effective?
CorrectIncorrectHint
Epoetin (Recombinant Human Erythropoietin) is used:
_x000D_
- _x000D_
- In Chronic Renal Failure
- To Shorten The Period Of Anaemia In Those Receiving Platinum-Based Chemotherapy, and
- Prevention Of Anaemia In Premature Babies With Low Birth Weight.
_x000D_
_x000D_
_x000D_
_x000D_
Its efficacy may be impaired in certain circumstances particularly with Iron Deficiency but also with Aluminium Toxicity, Folate Deficiency and Infection. In the latter, the switch to the Acute Phase Proteins impairs its function.
_x000D_
Its efficacy is unimpaired in Hyperkalaemia.
-
Question 5 of 121
5. Question
As per current classification, which of the following Glomerular Filtration Rate (GFR) ranges in ml/min/1.73 m2 is representative of Stage IV Chronic Kidney Disease (CKD)?
CorrectIncorrectHint
This question tests knowledge of the classification of Chronic Kidney Disease (CKD).
_x000D_
The table below lists Stages Of Chronic Kidney Disease (CKD) and Corresponding Glomerular Filtration Rates (GFRs).
_x000D_
The incorrect options listed above are not reflective of Stage IV Chronic Kidney Disease (CKD).
_x000D_
_x000D_ _x000D_
_x000D_ _x000D_ Stage
_x000D_
_x000D_
_x000D_ GFR
(ml/min/1.73 m2)_x000D_
_x000D_
_x000D_ Description
_x000D_
_x000D_
_x000D_
_x000D_ _x000D_ 1
_x000D_
_x000D_
_x000D_ >90
_x000D_
_x000D_
_x000D_ Normal Or Increased Glomerular Filtration Rate (GFR) with other
_x000D_
Evidence of renal damage.
_x000D_
_x000D_
_x000D_
_x000D_ _x000D_ 2
_x000D_
_x000D_
_x000D_ 60-89
_x000D_
_x000D_
_x000D_ Slight Decrease In Glomerular Filtration Rate (GFR), With Other
_x000D_
Evidence Of Renal Damage.
_x000D_
_x000D_
_x000D_
_x000D_ _x000D_ 3A
_x000D_
_x000D_
_x000D_ 45-59
_x000D_
_x000D_
_x000D_ Moderate Decrease In Glomerular Filtration Rate (GFR), With Or Without
_x000D_
Other Evidence Of Renal Damage.
_x000D_
_x000D_
_x000D_
_x000D_ _x000D_ 3
_x000D_
_x000D_
_x000D_ 30-44
_x000D_
_x000D_
_x000D_ As Above.
_x000D_
_x000D_
_x000D_
_x000D_ _x000D_ 4
_x000D_
_x000D_
_x000D_ 15-29
_x000D_
_x000D_
_x000D_ Severe Decrease In Glomerular Filtration Rate (GFR), With Or
_x000D_
Without Other Evidence Of Renal Damage.
_x000D_
_x000D_
_x000D_
_x000D_ _x000D_ 5
_x000D_
_x000D_
_x000D_ <15
_x000D_
_x000D_
_x000D_ Established Renal Failure.
_x000D_
_x000D_
_x000D_
_x000D_
_x000D_
The Suffix (p) is used to denote the presence of Proteinuria when Staging Chronic Kidney Disease (CKD). These patients have a worse prognosis, which usually warrants secondary care follow-up.
-
Question 6 of 121
6. Question
A 16-year-old girl underwent a Kidney Transplant. She was concerned about the effects of Long-Term Ciclosporin Treatment.
_x000D_
Which one of the following is a common adverse effect of this drug?
CorrectIncorrectHint
Ciclosporin causes Hypertrichosis rather than Alopecia and the most frequent adverse side effect of this drug is Nephrotoxicity.
_x000D_
Post Renal Transplant, the two most common causes of declining renal function are Graft Rejection and Ciclosporin Toxicity.
_x000D_
Hepatotoxicity and Paraesthesia are less common side effects of the drug. Hypertension can also be seen.
-
Question 7 of 121
7. Question
A 17-year-old boy presents to the Emergency Department with facial and periorbital oedema. This is the third episode over the past three years, and on each of the previous three occasions, the problem has been treated with Oral Corticosteroids.
_x000D_
On examination, he has periorbital and bilateral lower limb pitting oedema. His Blood Pressure is 130/70 mmHg, Pulse Rate is 72/Minute and Regular. He has no significant findings on auscultation of the chest.
_x000D_
Investigations Show:
_x000D_
_x000D_ _x000D_
_x000D_ _x000D_ Haemoglobin
_x000D_
_x000D_
_x000D_ 123 g/L
_x000D_
_x000D_
_x000D_ (135-177)
_x000D_
_x000D_
_x000D_
_x000D_ _x000D_ White Blood Cell Count
_x000D_
_x000D_
_x000D_ 5.5 ×109/L
_x000D_
_x000D_
_x000D_ (4-11)
_x000D_
_x000D_
_x000D_
_x000D_ _x000D_ Platelet Count
_x000D_
_x000D_
_x000D_ 265 ×109/L
_x000D_
_x000D_
_x000D_ (150-400)
_x000D_
_x000D_
_x000D_
_x000D_ _x000D_ Serum Sodium
_x000D_
_x000D_
_x000D_ 140 mmol/L
_x000D_
_x000D_
_x000D_ (135-146)
_x000D_
_x000D_
_x000D_
_x000D_ _x000D_ Serum Potassium
_x000D_
_x000D_
_x000D_ 4.4 mmol/L
_x000D_
_x000D_
_x000D_ (3.5-5)
_x000D_
_x000D_
_x000D_
_x000D_ _x000D_ Serum Creatinine
_x000D_
_x000D_
_x000D_ 111 μmol/L
_x000D_
_x000D_
_x000D_ (79-118)
_x000D_
_x000D_
_x000D_
_x000D_ _x000D_ Serum Albumin
_x000D_
_x000D_
_x000D_ 26 g/L
_x000D_
_x000D_
_x000D_ (35-50)
_x000D_
_x000D_
_x000D_
_x000D_ _x000D_ Urine Dipstick
_x000D_
_x000D_
_x000D_ Protein +++
_x000D_
_x000D_
_x000D_
_x000D_
_x000D_
Which of the following is the most appropriate initial way to treat him?
CorrectIncorrectHint
Repeated Cycles of Corticosteroid Responsive Nephrotic Syndrome suggest that this individual has Minimal Change Disease. Therefore he is likely once again to respond to High Dose Prednisolone, with a tapering dose after the first six weeks of therapy.
_x000D_
In patients who fail to respond initially to Corticosteroids, Cyclophosphamide, Mycophenolate Mofetil, Tacrolimus and Ciclosporin are Optional Second Line Agents. There are no strict indications for these, but tend to be used if there is Steroid Resistance or Steroid Dependence, Partial Response To Steroids, Frequent Relapses, and Toxicity or Contraindications to Steroids. These agents can be used sequentially if there is a failure to respond.
-
Question 8 of 121
8. Question
A 15-year-old girl presented with Henoch-Schönlein Purpura and Renal Involvement.
_x000D_
What is the most likely outcome?
CorrectIncorrectHint
Henoch-Schönlein Purpura (HSP) is a Self-Limiting Vasculitis which occurs in children and young adults.
_x000D_
It is characterised by:
_x000D_
- _x000D_
- Non-Thrombocytopenic Purpura
- Arthralgia
- Abdominal Pain, and
- Glomerular Nephritis
_x000D_
_x000D_
_x000D_
_x000D_
_x000D_
_x000D_
It is likely to be an Immune Complex Disease – involving IgA, but no treatment has proven efficacy.
_x000D_
The disease usually settles between 4 to 6 weeks without sequelae if Kidney Involvement is Mild.
_x000D_
However, this condition can occasionally relapse.
_x000D_
_x000D_
-
Question 9 of 121
9. Question
A 15-year-old boy presents with a sore throat and macroscopic haematuria.
_x000D_
What would Light Microscopy of a Kidney Biopsy most likely show?
CorrectIncorrectHint
The most common cause of Macroscopic Haematuria in a child is IgA Nephritis.
_x000D_
This usually develops one to two days after a sore throat. It is most common in the second and third decades of life and is three times more common in males.
_x000D_
The urine may be frankly bloody or may be the colour of cola. There are no clots in the urine and the haematuria is generally painless although some patients complain of mild loin pain.
_x000D_
It tends to settle spontaneously within five days although the episodes may be recurrent lasting for one to two years.
_x000D_
Renal Biopsy will show Mesangial IgA Deposition on Immunofluorescence and Light Microscopy will show Mesangial Hyper-cellularity and Matrix Expansion.
_x000D_
-
Question 10 of 121
10. Question
A 15-year-old girl was seen by her Family Physician because of increasing lethargy and easy fatigability.
_x000D_
She had a recent history of “Flu”.
_x000D_
Biochemistry Tests show that she has Renal Impairment.
_x000D_
_x000D_ _x000D_
_x000D_ _x000D_ Serum Sodium
_x000D_
_x000D_
_x000D_ 141 mmol/L
_x000D_
_x000D_
_x000D_ (137-144)
_x000D_
_x000D_
_x000D_
_x000D_ _x000D_ Serum Potassium
_x000D_
_x000D_
_x000D_ 4.4 mmol/L
_x000D_
_x000D_
_x000D_ (3.5-4.9)
_x000D_
_x000D_
_x000D_
_x000D_ _x000D_ Serum Urea
_x000D_
_x000D_
_x000D_ 29 mmol/L
_x000D_
_x000D_
_x000D_ (2.5-7.5)
_x000D_
_x000D_
_x000D_
_x000D_ _x000D_ Serum Creatinine
_x000D_
_x000D_
_x000D_ 284 µmol/L
_x000D_
_x000D_
_x000D_ (60-110)
_x000D_
_x000D_
_x000D_
_x000D_
_x000D_
Her condition does not improve after several weeks on Corticosteroid Therapy, so a Renal Biopsy is performed.
_x000D_
The Renal Biopsy demonstrates the presence of Segmental Sclerosis of 3 of 10 Glomeruli identified in the Biopsy Specimen. Immunofluorescence Studies and Electron Microscopy do not reveal evidence for Immune Deposits.
_x000D_
Which one of the following statements is most accurate regarding her condition?
CorrectIncorrectHint
The findings in this case point to Focal Segmental Glomerulosclerosis (FSGS), which leads to Chronic Renal Failure in half of cases.
_x000D_
FSGS accounts for approximately 20% of cases of Nephrotic Syndrome in children and 40% in adults.
_x000D_
- _x000D_
- It is one of the most common Primary Glomerular Disorders causing End-Stage Renal Failure (ESRF).
_x000D_
_x000D_
The cardinal feature on Renal Biopsy is Progressive Glomerular Scarring.
_x000D_
- _x000D_
- Early in the disease course, Glomerulosclerosis involves a minority of glomeruli (focal) and only a portion of the glomerular globe (segmental).
- As the disease progresses more widespread Glomerulosclerosis develops. This is due to Podocyte Injury which leads to effacement of the Podocyte Foot Processes.
_x000D_
_x000D_
_x000D_
80% of causes of Focal Segmental Glomerulosclerosis (FSGS) are Idiopathic, thought to be mediated by Circulating Permeability Factors. Secondary forms can be familial (due to mutations in Specific Podocyte Genes), Viral (HIV-1, Parovirus B19, CMV, EBV), Drug-Induced (Heroin, IFN, Lithium, Pamidronate, Anabolic Steroids, Calcineurin Inhibitors), or Adaptive (Unilateral Renal Agenesis, Hypertension, Sickle Cell Anaemia, Vaso-Occlusion).
_x000D_
Treatment is aimed at Preserving Renal Function, and Inducing Remission of Proteinuria. Secondary causes should be excluded, as treatment should be targeted at the underlying condition in these cases.
_x000D_
Idiopathic cases are treated with Renin-Angiotensin Blockade and Dietary Sodium Restriction initially.
_x000D_
If Nephrotic Syndrome is present, High-Dose Glucocorticoid Therapy should be initiated and slowly tapered over a period of 3 – 6 months if a response is seen. If Focal Segmental Glomerulosclerosis (FSGS) is Glucocorticoid Resistant, therapy is with a Calcineurin Inhibitor.
_x000D_
A significant number of patients with Focal Segmental Glomerulosclerosis (FSGS) go on to End-Stage Renal Failure (ESRF). Unfortunately, Focal Segmental Glomerulosclerosis (FSGS) recurs in 40% of Renal Transplants. Risk Factors For Recurrence Include: Age: 6 – 15 years, Caucasian, Rapid Course To End-Stage Renal Failure (ESRF) (<3 years), Heavy Proteinuria Prior To Transplantation and Previous Allograft Loss. Plasmapheresis has been showed to lead to remission, if used early in the course of recurrence.
-
Question 11 of 121
11. Question
A 15-year-old girl presents with ankle swelling three days after having had a sore throat. On examination, she had a Blood Pressure of 120/80 mmHg and Ankle Oedema.
_x000D_
Investigations Reveal:
_x000D_
_x000D_ _x000D_
_x000D_ _x000D_ Serum Creatinine
_x000D_
_x000D_
_x000D_ 95 µmol/L
_x000D_
_x000D_
_x000D_ (60-110)
_x000D_
_x000D_
_x000D_
_x000D_ _x000D_ Serum Albumin
_x000D_
_x000D_
_x000D_ 23 g/L
_x000D_
_x000D_
_x000D_ (37-49)
_x000D_
_x000D_
_x000D_
_x000D_ _x000D_ 24 Hour Urinary Protein
_x000D_
_x000D_
_x000D_ 10 g
_x000D_
_x000D_
_x000D_ (<0.2)
_x000D_
_x000D_
_x000D_
_x000D_
_x000D_
What is the most likely diagnosis?
CorrectIncorrectHint
The correct answer is IgA Nephritis, in this case presenting with Nephrotic Syndrome. IgA Nephritis is most common during the second and third decade of life. It commonly occurs within two days of an onset of an Upper Respiratory Tract Infection (URTI), or less commonly, infection of other Mucous Membranes (e.g. Gastrointestinal, Bladder, Breast). It should be diagnosed by a Renal Biopsy, where IgA is seen deposited in the Mesangium.
_x000D_
The treatment of IgA Nephritis is variable. In a patient with Haematuria only, the treatment is conservative. When there is Nephrotic Range Proteinuria (>3 gm / day – as in this case) an 8-12 week course of Prednisolone should be prescribed. If the Proteinuria is <3 gm / day an ACE Inhibitor can be used. In all patients, careful control of Blood Pressure should be achieved, by using ACE Inhibitors in the first instance, and regular follow-up of Renal Function and Urinalysis.
_x000D_
30% of children will have a spontaneous remission within 10 years, but 25% will go on to develop End-Stage Renal Failure (ESRF) within 20 years.
_x000D_
_x000D_
Idiopathic Membranous Nephropathy accounts for 2-5% of cases of Nephrotic Syndrome in children, and 20-30% of cases in adults.
_x000D_
The immune mechanism that leads to the development of Membranous Nephropathy is unknown. Histologically, it is characterised by diffuse thickening of the Glomerular Basement Membrane (GBM) on Light Microscopy. On Immunofluorescence, the thickening is caused by Immune Deposits of IgG and C3, on the Subepithelial Surface of the Glomerular Basement Membrane (GBM).
_x000D_
When not Idiopathic, it is associated with:
_x000D_
- _x000D_
- Autoimmune Diseases – Systemic Lupus Erythematosus (SLE), Rheumatoid Arthritis, Thyroid Disease
- Drugs – Gold, Penicillamine, Captopril
- Malignancy – Bronchus, Breast, Stomach, Colon, Prostate
- Infections – Hepatitis B, Syphilis, Leprosy, Filariasis, and
- Diabetes Mellitus.
_x000D_
_x000D_
_x000D_
_x000D_
_x000D_
_x000D_
Membranoproliferative (or Mesangiocapillary) Glomerulonephritis can be classed into Three Types (I, II, and III) depending on which complement pathway is activated. It is associated with Systemic Lupus Erythematosus (SLE), Cryoglobulinaemia with or without Hepatitis C, Chronic Infections [Subacute Bacterial Endocarditis (SBA)] or with Neoplasms. It is not associated acutely with Upper Respiratory Tract Infections (URTI).
_x000D_
Minimal Change Nephropathy is the most common form of Nephrotic Syndrome in children (peak incidence 2-3 years of age). The Histological Findings on Light Microscopy are normal or small looking Glomeruli. On Electron Microscopy there is effacement of the Epithelial Cell Foot Processes over the Outer Surface of the Glomerular Basement Membrane (GBM). It tends to be steroid responsive in children, but over 60% of children will have further relapses. In adults, it is associated with Hodgkin’s Lymphoma, and other Carcinomas.
_x000D_
Post-Streptococcal Glomerulonephritis, as the name implies, occurs 10-14 days after an acute infection. The typical case occurs following infection with Group A Lancefield streptococci (β-Haemolytic strep, S. pyogenes) either causing Pharyngitis or Skin Infections. It is more common in the developing world. The histology shows diffuse Proliferative Glomerulonephritis (GN), with infiltration by Neutrophil Polymorphs. The main treatment is eradication of the infection (10/7 course of Penicillin) and symptomatic relief of the Acute Nephritis. The need for dialysis is uncommon, and complete recovery of Renal Function should occur.
-
Question 12 of 121
12. Question
A 16-year-old girl is admitted with a three-day history of rigors due to a Urinary Tract Infection (UTI).
_x000D_
On examination she appears unwell, has a Body Mass Index (BMI) of 31 kg/m2, and a Temperature of 39°C; examination is otherwise normal.
_x000D_
Initial Biochemistry Revealed:
_x000D_
_x000D_ _x000D_
_x000D_ _x000D_ Serum Potassium
_x000D_
_x000D_
_x000D_ 4.1 mmol/L
_x000D_
_x000D_
_x000D_ (3.5-4.9)
_x000D_
_x000D_
_x000D_
_x000D_ _x000D_ Serum Urea
_x000D_
_x000D_
_x000D_ 6.9 mmol/L
_x000D_
_x000D_
_x000D_ (2.5-7.5)
_x000D_
_x000D_
_x000D_
_x000D_ _x000D_ Plasma Glucose
_x000D_
_x000D_
_x000D_ 31 mmol/L
_x000D_
_x000D_
_x000D_ (3.0-6.0)
_x000D_
_x000D_
_x000D_
_x000D_ _x000D_ pH
_x000D_
_x000D_
_x000D_ 7.3
_x000D_
_x000D_
_x000D_ (7.36-7.44)
_x000D_
_x000D_
_x000D_
_x000D_ _x000D_ Standard Bicarbonate
_x000D_
_x000D_
_x000D_ 13 mmol/L
_x000D_
_x000D_
_x000D_ (20-28)
_x000D_
_x000D_
_x000D_
_x000D_ _x000D_ Base Deficit
_x000D_
_x000D_
_x000D_ -10 mmol/L
_x000D_
_x000D_
_x000D_
_x000D_
_x000D_
Urinalysis is negative for Ketones.
_x000D_
Which one of the following is the best initial treatment for her Hyperglycaemia?
CorrectIncorrectHint
This girl has a Metabolic Acidosis with pH of 7.3 and low Bicarbonate, this is most likely due to sepsis.
_x000D_
She is likely to have Type 2 Diabetes Mellitus (T2DM) given the Body Mass Index (BMI) with uncontrolled Hyperglycaemia but does not have Diabetic Ketoacidosis because the Urine is Negative for Ketones, so would not require Fixed Rate Insulin Infusion.
_x000D_
It is important that her glycaemia is controlled to promote recovery from the sepsis. This is best achieved with Intravenous Insulin Initially.
-
Question 13 of 121
13. Question
A 16-year-old girl developed pulmonary haemorrhage and acute renal failure requiring dialysis.
_x000D_
She has a history of recurrent epistaxis.
_x000D_
Investigations Revealed:
_x000D_
_x000D_ _x000D_
_x000D_ _x000D_ Renal Biopsy
_x000D_
_x000D_
_x000D_ Crescentic Glomerulonephritis
_x000D_
_x000D_
_x000D_
_x000D_
_x000D_
Which one of the following antibodies is most likely to be found in the blood?
CorrectIncorrectHint
This patient manifests a Pulmonary Renal Syndrome which is most commonly due to an Antineutrophil Cytoplasmic Antibody Test (ANCA) Positive Vasculitis and less commonly due to Goodpasture’s Syndrome (Antiglomerular Basement Membrane [GBM] Antibodies). The history of Epistaxis makes Granulomatosis with Polyangiitis the more likely diagnosis.
_x000D_
_x000D_
_x000D_
Anti-Neutrophil Cytoplasmic Antibodies [ANCA] are of two types:
_x000D_
- _x000D_
- c-Anti-Neutrophil Cytoplasmic Antibodies (cANCA)
_x000D_
_x000D_
- _x000D_
- Correlates with Antiproteinase 3 Antibodies (PR3)_x000D_
- _x000D_
- Proteinase-3 is a Neutral Serine Proteinase present in Azurophil Granules of Human Neutrophils.
- Antibodies against it may be present in isolation without a c-Anti-Neutrophil Cytoplasmic Antibodies (cANCA).
_x000D_
_x000D_
_x000D_
_x000D_
_x000D_
_x000D_
- _x000D_
- cANCA and Specificity for the PR3 Antigen is most specific for Granulomatosis with Polyangiitis.
_x000D_
_x000D_
_x000D_
- _x000D_
- In Granulomatosis with Polyangiitis, the level of PR3 Antibody and Anti-Neutrophil Cytoplasmic Antibodies (ANCA) Titre are related to disease activity and the antibodies typically disappear when the disease is in remission.
_x000D_
_x000D_
- _x000D_
- p-Anti-Neutrophil Cytoplasmic Antibodies (pANCA) and / or Antibody To Myeloperoxidase (MPO)
_x000D_
_x000D_
- _x000D_
- Far less specific than c-Anti-Neutrophil Cytoplasmic Antibodies (cANCA) and can be present in a range of inflammatory conditions such as Microscopic Polyangiitis (MPA), Churg-Strauss Syndrome and Goodpasture’s Syndrome.
- Myeloperoxidase (MPO) and p-Anti-Neutrophil Cytoplasmic Antibodies (pANCA) may also be present in Systemic Lupus Erythematosus (SLE), Rheumatoid Arthritis, Sjögren’s Syndrome and occasionally in Chronic Infections.
- They are positive in 10% of patients with Granulomatosis with Polyangiitis and are the most likely antibody to be present in this case, where Proteinase-3 is not an option.
_x000D_
_x000D_
_x000D_
_x000D_
Granulomatosis with Polyangiitis is a Multi-Organ Autoimmune Disease which can be fatal.
_x000D_
The Classical Triad consists of:
_x000D_
- _x000D_
- Necrotising Granulomatous Inflammation Of The Respiratory Tract
- Glomerulonephritis, and
- A Small-Vessel Vasculitis.
_x000D_
_x000D_
_x000D_
_x000D_
A prolonged history of Epistaxis or Sinusitis is commonly found in Granulomatosis with Polyangiitis, which in some patients is also associated with an Eosinophilia. The detection of Antineutrophil Cytoplasmic Antibodies directed against Proteinase-3 is highly specific, but is found in only 50% of patients with disease localised to the Respiratory Tract and 95% with generalised Granulomatosis with Polyangiitis.
_x000D_
Standard Therapy is with Cyclophosphamide and Corticosteroids. TNF-Alpha Blocking Agents, Antithymocyte Globulin and Monoclonal Anti-T Cell Antibodies can be used in disease refractory to these agents.
_x000D_
Systemic Inflammation and Vasculitis contribute to Accelerated Atherosclerosis in patients with Granulomatosis with Polyangiitis and there is therefore a significantly increased incidence of Stroke, Myocardial Infarction (MI) and Occlusive Artery Disease.
_x000D_
Antimitochondrial Antibodies are found in Primary Biliary Cirrhosis.
_x000D_
Anticentromere Antibodies are found in CREST Syndrome / Scleroderma Syndrome.
_x000D_
Antinuclear (ANA) and Anticardiolipin Antibodies are found in Systemic Lupus Erythematosus (SLE) which is not a cause of Pulmonary Renal Syndrome.
-
Question 14 of 121
14. Question
. A 30-year-old Caribbean gentleman with Human Immunodeficiency Virus (HIV) Infection was found to have High Blood Pressure at 180/120 mmHg (high on several occasions). His father died of Stroke at 52 years of age.
_x000D_
Routine Blood Tests Showed:
_x000D_
_x000D_ _x000D_
_x000D_ _x000D_ Serum Sodium
_x000D_
_x000D_
_x000D_ 144 mmol/L
_x000D_
_x000D_
_x000D_ (135-145)
_x000D_
_x000D_
_x000D_
_x000D_ _x000D_ Serum Potassium
_x000D_
_x000D_
_x000D_ 3.3 mmol/L
_x000D_
_x000D_
_x000D_ (3.5-5.1)
_x000D_
_x000D_
_x000D_
_x000D_ _x000D_ Serum Creatinine
_x000D_
_x000D_
_x000D_ 101 µmol/L
_x000D_
_x000D_
_x000D_ (60-110)
_x000D_
_x000D_
_x000D_
_x000D_ _x000D_ Serum Urea
_x000D_
_x000D_
_x000D_ 5.1 mmol/L
_x000D_
_x000D_
_x000D_ (2.9-9.0)
_x000D_
_x000D_
_x000D_
_x000D_ _x000D_ Serum Urinanalysis
_x000D_
_x000D_
_x000D_ No Proteinuria
_x000D_
_x000D_
_x000D_
_x000D_ _x000D_ Plasma Aldosterone
_x000D_
_x000D_
_x000D_ 820 pmol/L
_x000D_
_x000D_
_x000D_ (28-445)
_x000D_
_x000D_
_x000D_
_x000D_ _x000D_ Plasma Renin
_x000D_
_x000D_
_x000D_ 0.5 ng/ml/h
_x000D_
_x000D_
_x000D_ (0.7-5)
_x000D_
_x000D_
_x000D_
_x000D_ _x000D_ Renal Ultrasound Scan
_x000D_
_x000D_
_x000D_ Normal-Sized Kidneys
_x000D_
_x000D_
_x000D_
_x000D_ _x000D_ CD4
_x000D_
_x000D_
_x000D_ 770 cells/ml
_x000D_
_x000D_
_x000D_
_x000D_ _x000D_ HIV RNA
_x000D_
_x000D_
_x000D_ 4533 copies/ml
_x000D_
_x000D_
_x000D_
_x000D_
_x000D_
He was not on any Antiretroviral Therapy.
_x000D_
Which of the following is the most likely cause of his High Arterial Blood Pressure?
CorrectIncorrectHint
High Aldosterone and Low Renin Levels indicate Primary Hyperaldosteronism which accounts for up to 10% of all Hypertensive patients. It is the most prevalent form of Secondary Hypertension.
_x000D_
Potassium Level is often normal or the lower limit of normal; as such, Hypokalaemia is not a prerequisite for the diagnosis of Conn’s Syndrome.
_x000D_
Normal Renal Ultrasound Scan makes Adult Polycystic Kidney (APK) an unlikely cause of High Blood Pressure.
_x000D_
High Aldosterone and Low Renin Level is not seen in Familial Essential Hypertension.
_x000D_
Human Immunodeficiency Virus (HIV)-associated Nephropathy causes Proteinuria.
_x000D_
Non-Hodgkin’s Lymphoma is often associated with Low CD4 Count.
_x000D_
-
Question 15 of 121
15. Question
A 13-year-old girl presents to the Clinic with Periorbital Oedema and Mild Ankle Oedema which has increased over the past two weeks.
_x000D_
Other history of note is a recent Upper Respiratory Tract Infection (URTI). She has been feeling increasingly tired and lethargic over the past two weeks.
_x000D_
On examination, her Blood Pressure is 110/70 mmHg. She has Periorbital Oedema and Pitting Ankle Oedema.
_x000D_
Investigations Show:
_x000D_
_x000D_ _x000D_
_x000D_ _x000D_ Haemoglobin
_x000D_
_x000D_
_x000D_ 122 g/L
_x000D_
_x000D_
_x000D_ (110-130)
_x000D_
_x000D_
_x000D_
_x000D_ _x000D_ Total Leucocyte Count
_x000D_
_x000D_
_x000D_ 8.1 ×109/L
_x000D_
_x000D_
_x000D_ (4-10)
_x000D_
_x000D_
_x000D_
_x000D_ _x000D_ Platelet Count
_x000D_
_x000D_
_x000D_ 194 ×109/L
_x000D_
_x000D_
_x000D_ (150-400)
_x000D_
_x000D_
_x000D_
_x000D_ _x000D_ Serum Sodium
_x000D_
_x000D_
_x000D_ 140 mmol/L
_x000D_
_x000D_
_x000D_ (134-143)
_x000D_
_x000D_
_x000D_
_x000D_ _x000D_ Serum Potassium
_x000D_
_x000D_
_x000D_ 4.4 mmol/L
_x000D_
_x000D_
_x000D_ (3.5-5)
_x000D_
_x000D_
_x000D_
_x000D_ _x000D_ Serum Creatinine
_x000D_
_x000D_
_x000D_ 108 μmol/L
_x000D_
_x000D_
_x000D_ (60-120)
_x000D_
_x000D_
_x000D_
_x000D_ _x000D_ Serum Albumin
_x000D_
_x000D_
_x000D_ 30 g/L
_x000D_
_x000D_
_x000D_ (35-50)
_x000D_
_x000D_
_x000D_
_x000D_ _x000D_ Urine Dipstick
_x000D_
_x000D_
_x000D_ Protein ++, Blood (-)
_x000D_
_x000D_
_x000D_
_x000D_
_x000D_
Which of the following is the most likely cause?
_x000D_
_x000D_
CorrectIncorrectHint
The history of Periorbital Oedema, Normal Blood Pressure and Serum Creatinine, Proteinuria and Low Serum Albumin indicates towards nephrotic syndrome.
_x000D_
The Lack Of Haematuria counts against Post-Streptococcal Glomerulonephritis, and IgA Nephropathy.
_x000D_
Hence, the most likely diagnosis, given the age of the patient and clinical scenario, is Minimal Change Disease.
_x000D_
- _x000D_
- The condition is much more common in this age range than Membranous Nephropathy (adults more often affected).
- The important point about Minimal change disease is that the condition responds very well to Corticosteroid Therapy i.e. in 90% or more of sufferers in childhood within two weeks, although treatment is usually continued for a period of eight weeks.
_x000D_
_x000D_
_x000D_
Alport’s Syndrome is a familial Nephritis associated with Haematuria and Progressive Sensorineural Hearing Loss and is not indicated in this patient.
_x000D_
-
Question 16 of 121
16. Question
A 14-year-old boy is brought by his parents to General Practitioner with complaints of feeling unwell, passing smoky dark urine and having swelling of his ankles.
_x000D_
Of note, he reports a sore throat two weeks prior. His Antistreptolysin O Titre is positive and his Renal Function is Mildly Impaired.
_x000D_
If this boy were to have a Renal Biopsy, which of the following is the most likely finding?
CorrectIncorrectHint
Not all cases of Post-Streptococcal Glomerulonephritis (GN) require a Biopsy.
_x000D_
The Characteristic Electron Microscopy findings are described in the correct option.
_x000D_
C4d Staining Positive is incorrect as this refers to the detection of BK Virus by the C4d Stain.
_x000D_
Podocyte Effacement is incorrect as this is diagnostic of Minimal Change Disease.
_x000D_
IgA Deposition In The Mesangium is a biopsy finding in IgA Nephropathy and so is incorrect.
_x000D_
The Tram Track Appearance on Light Microscopy represents Membranoproliferative Glomerulonephritis and therefore should not be selected.
_x000D_
_x000D_
_x000D_
-
Question 17 of 121
17. Question
Serum Creatinine has a number of limitations as an Estimate Of Glomerular Filtration Rate (GFR), including variation with Muscle Mass and Age.
_x000D_
Which of the following answers represents a Novel Marker of estimating Glomerular Filtration Rate (GFR)?
CorrectIncorrectHint
This question demonstrates knowledge of Markers Of Glomerular Filtration Rate (GFR).
_x000D_
Serum Creatinine has limitations as a marker of Glomerular Filtration Rate (GFR).
_x000D_
Nystatin is an Antifungal Agent and not a marker of Glomerular Filtration Rate (GFR).
_x000D_
Inulin is a Valid Marker Of Renal Function and was used experimentally in the past for many years.
_x000D_
Alpha Amyloid Protein and IGF-1 have No Role In Estimating Renal Function.
_x000D_
In regards to Cystatin C it is now a much more Sensitive Marker Of Kidney Function as opposed to Insulin and Insulin Resistance. More recent studies have proven this and can be referenced below
-
Question 18 of 121
18. Question
A 16-year-old boy was referred to a cardiology clinic for an assessment as his brother had hypertrophic cardiomyopathy. His 2D echocardiogram confirmed the condition.
Which one of the following echocardiographic features is considered to be the most important risk factor for a sudden cardiac death?
CorrectIncorrectHint
The most plausible diagnosis for this lady presenting with headache, bilateral blurring of the optic discs, and pupil-sparing third nerve palsy on the left is D. Sagittal sinus thrombosis. This is because sagittal sinus thrombosis is a rare condition that occurs when a blood clot forms in the sagittal sinus, which is a large vein that drains blood from the brain. This can cause increased pressure in the brain, leading to symptoms such as headache, blurred vision, and cranial nerve palsies. Sagittal sinus thrombosis is more common in women, especially during pregnancy and the postpartum period, due to hormonal changes and hypercoagulability.
The other options are less likely for the following reasons:
- A. Herpes simplex virus (HSV) encephalitis: Herpes simplex virus (HSV) encephalitis is a rare but serious infection of the brain caused by the herpes simplex virus. It can cause headache, fever, confusion, seizures, and focal neurological deficits. However, it is unlikely to cause bilateral blurring of the optic discs or pupil-sparing third nerve palsy, as these are signs of increased intracranial pressure, not focal brain inflammation.
- B. Brainstem cerebrovascular accident (CVA): A brainstem cerebrovascular accident (CVA) is a stroke that affects the brainstem, which is the part of the brain that controls vital functions such as breathing, heart rate, and blood pressure. It can cause headache, dizziness, nausea, vomiting, and cranial nerve palsies. However, it is unlikely to cause bilateral blurring of the optic discs, as this is a sign of increased intracranial pressure, not brainstem ischaemia. Moreover, a brainstem cerebrovascular accident (CVA) would not spare the pupil, as the third nerve fibers that control the pupil are located in the brainstem.
- C. Meningococcal meningitis: Meningococcal meningitis is a severe infection of the meninges, which are the membranes that cover the brain and spinal cord, caused by the bacterium Neisseria meningitidis. It can cause headache, fever, neck stiffness, photophobia, and rash. It can also cause cranial nerve palsies, especially of the sixth and seventh nerves. However, it is unlikely to cause bilateral blurring of the optic discs or pupil-sparing third nerve palsy, as these are signs of increased intracranial pressure, not meningeal inflammation.
- E. Sphenoidal wing meningioma: A sphenoidal wing meningioma is a benign tumour that arises from the meninges near the sphenoid bone, which is a bone at the base of the skull. It can cause headache, visual loss, and cranial nerve palsies, especially of the third, fourth, and sixth nerves. However, it is unlikely to cause bilateral blurring of the optic discs, as this is a sign of increased intracranial pressure, not a focal mass lesion. Moreover, a sphenoidal wing meningioma would not spare the pupil, as the third nerve fibers that control the pupil are located close to the tumour site.
-
Question 19 of 121
19. Question
A 46-year-old gentleman presents to the Emergency Department with acute coronary syndrome.
On examination he is found to have palmar crease xanthoma.
Which one of the following is the most likely diagnosis of his lipid abnormalities?
CorrectIncorrectHint
The most plausible diagnosis for this lady presenting with headache, bilateral blurring of the optic discs, and pupil-sparing third nerve palsy on the left is D. Sagittal sinus thrombosis. This is because sagittal sinus thrombosis is a rare condition that occurs when a blood clot forms in the sagittal sinus, which is a large vein that drains blood from the brain. This can cause increased pressure in the brain, leading to symptoms such as headache, blurred vision, and cranial nerve palsies. Sagittal sinus thrombosis is more common in women, especially during pregnancy and the postpartum period, due to hormonal changes and hypercoagulability.
The other options are less likely for the following reasons:
- A. Herpes simplex virus (HSV) encephalitis: Herpes simplex virus (HSV) encephalitis is a rare but serious infection of the brain caused by the herpes simplex virus. It can cause headache, fever, confusion, seizures, and focal neurological deficits. However, it is unlikely to cause bilateral blurring of the optic discs or pupil-sparing third nerve palsy, as these are signs of increased intracranial pressure, not focal brain inflammation.
- B. Brainstem cerebrovascular accident (CVA): A brainstem cerebrovascular accident (CVA) is a stroke that affects the brainstem, which is the part of the brain that controls vital functions such as breathing, heart rate, and blood pressure. It can cause headache, dizziness, nausea, vomiting, and cranial nerve palsies. However, it is unlikely to cause bilateral blurring of the optic discs, as this is a sign of increased intracranial pressure, not brainstem ischaemia. Moreover, a brainstem cerebrovascular accident (CVA) would not spare the pupil, as the third nerve fibers that control the pupil are located in the brainstem.
- C. Meningococcal meningitis: Meningococcal meningitis is a severe infection of the meninges, which are the membranes that cover the brain and spinal cord, caused by the bacterium Neisseria meningitidis. It can cause headache, fever, neck stiffness, photophobia, and rash. It can also cause cranial nerve palsies, especially of the sixth and seventh nerves. However, it is unlikely to cause bilateral blurring of the optic discs or pupil-sparing third nerve palsy, as these are signs of increased intracranial pressure, not meningeal inflammation.
- E. Sphenoidal wing meningioma: A sphenoidal wing meningioma is a benign tumour that arises from the meninges near the sphenoid bone, which is a bone at the base of the skull. It can cause headache, visual loss, and cranial nerve palsies, especially of the third, fourth, and sixth nerves. However, it is unlikely to cause bilateral blurring of the optic discs, as this is a sign of increased intracranial pressure, not a focal mass lesion. Moreover, a sphenoidal wing meningioma would not spare the pupil, as the third nerve fibers that control the pupil are located close to the tumour site.
-
Question 20 of 121
20. Question
A 19-year-old lady is found to have a cardiac murmur characterised by a mid-systolic click during an examination in the cardiac clinic. A 2D echocardiogram reveals mitral insufficiency with upward displacement of one leaflet. There is also aortic root dilation to 4 cm. She also has a dislocated left ocular crystalline lens.
She dies suddenly and unexpectedly. On medical examination a prolapsed mitral valve with elongation, thinning, and rupture of chordae tendineae were found.
A mutation in which one of the following genes is most likely have be present in this lady?
CorrectIncorrectHint
The most plausible diagnosis for this lady presenting with headache, bilateral blurring of the optic discs, and pupil-sparing third nerve palsy on the left is D. Sagittal sinus thrombosis. This is because sagittal sinus thrombosis is a rare condition that occurs when a blood clot forms in the sagittal sinus, which is a large vein that drains blood from the brain. This can cause increased pressure in the brain, leading to symptoms such as headache, blurred vision, and cranial nerve palsies. Sagittal sinus thrombosis is more common in women, especially during pregnancy and the postpartum period, due to hormonal changes and hypercoagulability.
The other options are less likely for the following reasons:
- A. Herpes simplex virus (HSV) encephalitis: Herpes simplex virus (HSV) encephalitis is a rare but serious infection of the brain caused by the herpes simplex virus. It can cause headache, fever, confusion, seizures, and focal neurological deficits. However, it is unlikely to cause bilateral blurring of the optic discs or pupil-sparing third nerve palsy, as these are signs of increased intracranial pressure, not focal brain inflammation.
- B. Brainstem cerebrovascular accident (CVA): A brainstem cerebrovascular accident (CVA) is a stroke that affects the brainstem, which is the part of the brain that controls vital functions such as breathing, heart rate, and blood pressure. It can cause headache, dizziness, nausea, vomiting, and cranial nerve palsies. However, it is unlikely to cause bilateral blurring of the optic discs, as this is a sign of increased intracranial pressure, not brainstem ischaemia. Moreover, a brainstem cerebrovascular accident (CVA) would not spare the pupil, as the third nerve fibers that control the pupil are located in the brainstem.
- C. Meningococcal meningitis: Meningococcal meningitis is a severe infection of the meninges, which are the membranes that cover the brain and spinal cord, caused by the bacterium Neisseria meningitidis. It can cause headache, fever, neck stiffness, photophobia, and rash. It can also cause cranial nerve palsies, especially of the sixth and seventh nerves. However, it is unlikely to cause bilateral blurring of the optic discs or pupil-sparing third nerve palsy, as these are signs of increased intracranial pressure, not meningeal inflammation.
- E. Sphenoidal wing meningioma: A sphenoidal wing meningioma is a benign tumour that arises from the meninges near the sphenoid bone, which is a bone at the base of the skull. It can cause headache, visual loss, and cranial nerve palsies, especially of the third, fourth, and sixth nerves. However, it is unlikely to cause bilateral blurring of the optic discs, as this is a sign of increased intracranial pressure, not a focal mass lesion. Moreover, a sphenoidal wing meningioma would not spare the pupil, as the third nerve fibers that control the pupil are located close to the tumour site.
-
Question 21 of 121
21. Question
A 45-year-old lady with earlier mitral valve stenosis comes to the cardiology clinic to seek advice regarding operative procedures.
Antibiotic prophylaxis against infective endocarditis would be required in which one of the following circumstances?
CorrectIncorrectHint
The most plausible diagnosis for this lady presenting with headache, bilateral blurring of the optic discs, and pupil-sparing third nerve palsy on the left is D. Sagittal sinus thrombosis. This is because sagittal sinus thrombosis is a rare condition that occurs when a blood clot forms in the sagittal sinus, which is a large vein that drains blood from the brain. This can cause increased pressure in the brain, leading to symptoms such as headache, blurred vision, and cranial nerve palsies. Sagittal sinus thrombosis is more common in women, especially during pregnancy and the postpartum period, due to hormonal changes and hypercoagulability.
The other options are less likely for the following reasons:
- A. Herpes simplex virus (HSV) encephalitis: Herpes simplex virus (HSV) encephalitis is a rare but serious infection of the brain caused by the herpes simplex virus. It can cause headache, fever, confusion, seizures, and focal neurological deficits. However, it is unlikely to cause bilateral blurring of the optic discs or pupil-sparing third nerve palsy, as these are signs of increased intracranial pressure, not focal brain inflammation.
- B. Brainstem cerebrovascular accident (CVA): A brainstem cerebrovascular accident (CVA) is a stroke that affects the brainstem, which is the part of the brain that controls vital functions such as breathing, heart rate, and blood pressure. It can cause headache, dizziness, nausea, vomiting, and cranial nerve palsies. However, it is unlikely to cause bilateral blurring of the optic discs, as this is a sign of increased intracranial pressure, not brainstem ischaemia. Moreover, a brainstem cerebrovascular accident (CVA) would not spare the pupil, as the third nerve fibers that control the pupil are located in the brainstem.
- C. Meningococcal meningitis: Meningococcal meningitis is a severe infection of the meninges, which are the membranes that cover the brain and spinal cord, caused by the bacterium Neisseria meningitidis. It can cause headache, fever, neck stiffness, photophobia, and rash. It can also cause cranial nerve palsies, especially of the sixth and seventh nerves. However, it is unlikely to cause bilateral blurring of the optic discs or pupil-sparing third nerve palsy, as these are signs of increased intracranial pressure, not meningeal inflammation.
- E. Sphenoidal wing meningioma: A sphenoidal wing meningioma is a benign tumour that arises from the meninges near the sphenoid bone, which is a bone at the base of the skull. It can cause headache, visual loss, and cranial nerve palsies, especially of the third, fourth, and sixth nerves. However, it is unlikely to cause bilateral blurring of the optic discs, as this is a sign of increased intracranial pressure, not a focal mass lesion. Moreover, a sphenoidal wing meningioma would not spare the pupil, as the third nerve fibers that control the pupil are located close to the tumour site.
-
Question 22 of 121
22. Question
Which one of the following mechanisms best explains the action of simvastatin?
CorrectIncorrectHint
The most plausible diagnosis for this lady presenting with headache, bilateral blurring of the optic discs, and pupil-sparing third nerve palsy on the left is D. Sagittal sinus thrombosis. This is because sagittal sinus thrombosis is a rare condition that occurs when a blood clot forms in the sagittal sinus, which is a large vein that drains blood from the brain. This can cause increased pressure in the brain, leading to symptoms such as headache, blurred vision, and cranial nerve palsies. Sagittal sinus thrombosis is more common in women, especially during pregnancy and the postpartum period, due to hormonal changes and hypercoagulability.
The other options are less likely for the following reasons:
- A. Herpes simplex virus (HSV) encephalitis: Herpes simplex virus (HSV) encephalitis is a rare but serious infection of the brain caused by the herpes simplex virus. It can cause headache, fever, confusion, seizures, and focal neurological deficits. However, it is unlikely to cause bilateral blurring of the optic discs or pupil-sparing third nerve palsy, as these are signs of increased intracranial pressure, not focal brain inflammation.
- B. Brainstem cerebrovascular accident (CVA): A brainstem cerebrovascular accident (CVA) is a stroke that affects the brainstem, which is the part of the brain that controls vital functions such as breathing, heart rate, and blood pressure. It can cause headache, dizziness, nausea, vomiting, and cranial nerve palsies. However, it is unlikely to cause bilateral blurring of the optic discs, as this is a sign of increased intracranial pressure, not brainstem ischaemia. Moreover, a brainstem cerebrovascular accident (CVA) would not spare the pupil, as the third nerve fibers that control the pupil are located in the brainstem.
- C. Meningococcal meningitis: Meningococcal meningitis is a severe infection of the meninges, which are the membranes that cover the brain and spinal cord, caused by the bacterium Neisseria meningitidis. It can cause headache, fever, neck stiffness, photophobia, and rash. It can also cause cranial nerve palsies, especially of the sixth and seventh nerves. However, it is unlikely to cause bilateral blurring of the optic discs or pupil-sparing third nerve palsy, as these are signs of increased intracranial pressure, not meningeal inflammation.
- E. Sphenoidal wing meningioma: A sphenoidal wing meningioma is a benign tumour that arises from the meninges near the sphenoid bone, which is a bone at the base of the skull. It can cause headache, visual loss, and cranial nerve palsies, especially of the third, fourth, and sixth nerves. However, it is unlikely to cause bilateral blurring of the optic discs, as this is a sign of increased intracranial pressure, not a focal mass lesion. Moreover, a sphenoidal wing meningioma would not spare the pupil, as the third nerve fibers that control the pupil are located close to the tumour site.
-
Question 23 of 121
23. Question
A 60-year-old gentleman with a history of schizophrenia on thioridazine is found to have episodes of Torsades de pointes (TdP) ventricular tachycardia (VT).
His blood pressure is 120/70 mmHg.
Which one of the following is the most appropriate management for this gentleman?
CorrectIncorrectHint
The most plausible diagnosis for this lady presenting with headache, bilateral blurring of the optic discs, and pupil-sparing third nerve palsy on the left is D. Sagittal sinus thrombosis. This is because sagittal sinus thrombosis is a rare condition that occurs when a blood clot forms in the sagittal sinus, which is a large vein that drains blood from the brain. This can cause increased pressure in the brain, leading to symptoms such as headache, blurred vision, and cranial nerve palsies. Sagittal sinus thrombosis is more common in women, especially during pregnancy and the postpartum period, due to hormonal changes and hypercoagulability.
The other options are less likely for the following reasons:
- A. Herpes simplex virus (HSV) encephalitis: Herpes simplex virus (HSV) encephalitis is a rare but serious infection of the brain caused by the herpes simplex virus. It can cause headache, fever, confusion, seizures, and focal neurological deficits. However, it is unlikely to cause bilateral blurring of the optic discs or pupil-sparing third nerve palsy, as these are signs of increased intracranial pressure, not focal brain inflammation.
- B. Brainstem cerebrovascular accident (CVA): A brainstem cerebrovascular accident (CVA) is a stroke that affects the brainstem, which is the part of the brain that controls vital functions such as breathing, heart rate, and blood pressure. It can cause headache, dizziness, nausea, vomiting, and cranial nerve palsies. However, it is unlikely to cause bilateral blurring of the optic discs, as this is a sign of increased intracranial pressure, not brainstem ischaemia. Moreover, a brainstem cerebrovascular accident (CVA) would not spare the pupil, as the third nerve fibers that control the pupil are located in the brainstem.
- C. Meningococcal meningitis: Meningococcal meningitis is a severe infection of the meninges, which are the membranes that cover the brain and spinal cord, caused by the bacterium Neisseria meningitidis. It can cause headache, fever, neck stiffness, photophobia, and rash. It can also cause cranial nerve palsies, especially of the sixth and seventh nerves. However, it is unlikely to cause bilateral blurring of the optic discs or pupil-sparing third nerve palsy, as these are signs of increased intracranial pressure, not meningeal inflammation.
- E. Sphenoidal wing meningioma: A sphenoidal wing meningioma is a benign tumour that arises from the meninges near the sphenoid bone, which is a bone at the base of the skull. It can cause headache, visual loss, and cranial nerve palsies, especially of the third, fourth, and sixth nerves. However, it is unlikely to cause bilateral blurring of the optic discs, as this is a sign of increased intracranial pressure, not a focal mass lesion. Moreover, a sphenoidal wing meningioma would not spare the pupil, as the third nerve fibers that control the pupil are located close to the tumour site.
-
Question 24 of 121
24. Question
A 55-year-old lady house cleaner is undergoing investigation for breathlessness.
Which one of the following does not support a diagnosis of constrictive pericarditis?
CorrectIncorrectHint
The most plausible diagnosis for this lady presenting with headache, bilateral blurring of the optic discs, and pupil-sparing third nerve palsy on the left is D. Sagittal sinus thrombosis. This is because sagittal sinus thrombosis is a rare condition that occurs when a blood clot forms in the sagittal sinus, which is a large vein that drains blood from the brain. This can cause increased pressure in the brain, leading to symptoms such as headache, blurred vision, and cranial nerve palsies. Sagittal sinus thrombosis is more common in women, especially during pregnancy and the postpartum period, due to hormonal changes and hypercoagulability.
The other options are less likely for the following reasons:
- A. Herpes simplex virus (HSV) encephalitis: Herpes simplex virus (HSV) encephalitis is a rare but serious infection of the brain caused by the herpes simplex virus. It can cause headache, fever, confusion, seizures, and focal neurological deficits. However, it is unlikely to cause bilateral blurring of the optic discs or pupil-sparing third nerve palsy, as these are signs of increased intracranial pressure, not focal brain inflammation.
- B. Brainstem cerebrovascular accident (CVA): A brainstem cerebrovascular accident (CVA) is a stroke that affects the brainstem, which is the part of the brain that controls vital functions such as breathing, heart rate, and blood pressure. It can cause headache, dizziness, nausea, vomiting, and cranial nerve palsies. However, it is unlikely to cause bilateral blurring of the optic discs, as this is a sign of increased intracranial pressure, not brainstem ischaemia. Moreover, a brainstem cerebrovascular accident (CVA) would not spare the pupil, as the third nerve fibers that control the pupil are located in the brainstem.
- C. Meningococcal meningitis: Meningococcal meningitis is a severe infection of the meninges, which are the membranes that cover the brain and spinal cord, caused by the bacterium Neisseria meningitidis. It can cause headache, fever, neck stiffness, photophobia, and rash. It can also cause cranial nerve palsies, especially of the sixth and seventh nerves. However, it is unlikely to cause bilateral blurring of the optic discs or pupil-sparing third nerve palsy, as these are signs of increased intracranial pressure, not meningeal inflammation.
- E. Sphenoidal wing meningioma: A sphenoidal wing meningioma is a benign tumour that arises from the meninges near the sphenoid bone, which is a bone at the base of the skull. It can cause headache, visual loss, and cranial nerve palsies, especially of the third, fourth, and sixth nerves. However, it is unlikely to cause bilateral blurring of the optic discs, as this is a sign of increased intracranial pressure, not a focal mass lesion. Moreover, a sphenoidal wing meningioma would not spare the pupil, as the third nerve fibers that control the pupil are located close to the tumour site.
-
Question 25 of 121
25. Question
A 60-year-old gentleman is admitted with a high blood pressure of 200/110 mmHg and episodic runs of ventricular tachycardia (VT).
Investigations confirm the presence of a right adrenal phaeochromocytoma.
Which one of the following would be the most appropriate initial therapy?
CorrectIncorrectHint
The most plausible diagnosis for this lady presenting with headache, bilateral blurring of the optic discs, and pupil-sparing third nerve palsy on the left is D. Sagittal sinus thrombosis. This is because sagittal sinus thrombosis is a rare condition that occurs when a blood clot forms in the sagittal sinus, which is a large vein that drains blood from the brain. This can cause increased pressure in the brain, leading to symptoms such as headache, blurred vision, and cranial nerve palsies. Sagittal sinus thrombosis is more common in women, especially during pregnancy and the postpartum period, due to hormonal changes and hypercoagulability.
The other options are less likely for the following reasons:
- A. Herpes simplex virus (HSV) encephalitis: Herpes simplex virus (HSV) encephalitis is a rare but serious infection of the brain caused by the herpes simplex virus. It can cause headache, fever, confusion, seizures, and focal neurological deficits. However, it is unlikely to cause bilateral blurring of the optic discs or pupil-sparing third nerve palsy, as these are signs of increased intracranial pressure, not focal brain inflammation.
- B. Brainstem cerebrovascular accident (CVA): A brainstem cerebrovascular accident (CVA) is a stroke that affects the brainstem, which is the part of the brain that controls vital functions such as breathing, heart rate, and blood pressure. It can cause headache, dizziness, nausea, vomiting, and cranial nerve palsies. However, it is unlikely to cause bilateral blurring of the optic discs, as this is a sign of increased intracranial pressure, not brainstem ischaemia. Moreover, a brainstem cerebrovascular accident (CVA) would not spare the pupil, as the third nerve fibers that control the pupil are located in the brainstem.
- C. Meningococcal meningitis: Meningococcal meningitis is a severe infection of the meninges, which are the membranes that cover the brain and spinal cord, caused by the bacterium Neisseria meningitidis. It can cause headache, fever, neck stiffness, photophobia, and rash. It can also cause cranial nerve palsies, especially of the sixth and seventh nerves. However, it is unlikely to cause bilateral blurring of the optic discs or pupil-sparing third nerve palsy, as these are signs of increased intracranial pressure, not meningeal inflammation.
- E. Sphenoidal wing meningioma: A sphenoidal wing meningioma is a benign tumour that arises from the meninges near the sphenoid bone, which is a bone at the base of the skull. It can cause headache, visual loss, and cranial nerve palsies, especially of the third, fourth, and sixth nerves. However, it is unlikely to cause bilateral blurring of the optic discs, as this is a sign of increased intracranial pressure, not a focal mass lesion. Moreover, a sphenoidal wing meningioma would not spare the pupil, as the third nerve fibers that control the pupil are located close to the tumour site.
-
Question 26 of 121
26. Question
A 63-year-old gentleman with a nine-year history of type 2 diabetes mellitus (T2DM) is being treated with metformin 1 gm twice daily and gliclazide 160 mg twice daily.
He is obese, has gained weight over the last eight months and his serum HbA1c has deteriorated from 58 to 66 mmol/mol (20-42). He is being considered for treatment with either Insulin or pioglitazone.
He wants to know the side effects of pioglitazone.
Which one of the following is regarded as a typical side effect of pioglitazone therapy?
CorrectIncorrectHint
The most plausible diagnosis for this lady presenting with headache, bilateral blurring of the optic discs, and pupil-sparing third nerve palsy on the left is D. Sagittal sinus thrombosis. This is because sagittal sinus thrombosis is a rare condition that occurs when a blood clot forms in the sagittal sinus, which is a large vein that drains blood from the brain. This can cause increased pressure in the brain, leading to symptoms such as headache, blurred vision, and cranial nerve palsies. Sagittal sinus thrombosis is more common in women, especially during pregnancy and the postpartum period, due to hormonal changes and hypercoagulability.
The other options are less likely for the following reasons:
- A. Herpes simplex virus (HSV) encephalitis: Herpes simplex virus (HSV) encephalitis is a rare but serious infection of the brain caused by the herpes simplex virus. It can cause headache, fever, confusion, seizures, and focal neurological deficits. However, it is unlikely to cause bilateral blurring of the optic discs or pupil-sparing third nerve palsy, as these are signs of increased intracranial pressure, not focal brain inflammation.
- B. Brainstem cerebrovascular accident (CVA): A brainstem cerebrovascular accident (CVA) is a stroke that affects the brainstem, which is the part of the brain that controls vital functions such as breathing, heart rate, and blood pressure. It can cause headache, dizziness, nausea, vomiting, and cranial nerve palsies. However, it is unlikely to cause bilateral blurring of the optic discs, as this is a sign of increased intracranial pressure, not brainstem ischaemia. Moreover, a brainstem cerebrovascular accident (CVA) would not spare the pupil, as the third nerve fibers that control the pupil are located in the brainstem.
- C. Meningococcal meningitis: Meningococcal meningitis is a severe infection of the meninges, which are the membranes that cover the brain and spinal cord, caused by the bacterium Neisseria meningitidis. It can cause headache, fever, neck stiffness, photophobia, and rash. It can also cause cranial nerve palsies, especially of the sixth and seventh nerves. However, it is unlikely to cause bilateral blurring of the optic discs or pupil-sparing third nerve palsy, as these are signs of increased intracranial pressure, not meningeal inflammation.
- E. Sphenoidal wing meningioma: A sphenoidal wing meningioma is a benign tumour that arises from the meninges near the sphenoid bone, which is a bone at the base of the skull. It can cause headache, visual loss, and cranial nerve palsies, especially of the third, fourth, and sixth nerves. However, it is unlikely to cause bilateral blurring of the optic discs, as this is a sign of increased intracranial pressure, not a focal mass lesion. Moreover, a sphenoidal wing meningioma would not spare the pupil, as the third nerve fibers that control the pupil are located close to the tumour site.
-
Question 27 of 121
27. Question
A 24-year-old gentleman comes to the Nephrology Clinic for discussion of Abdominal Ultrasound Scan (USS) Screening for Autosomal Dominant Polycystic Kidney Disease (ADPKD) Type 1. His father is 48-years-old and has recently begun Haemodialysis (HD) for Autosomal Dominant Polycystic Kidney Disorder (ADPKD) and End-Stage Renal Failure (ESRF).
_x000D_
His first Ultrasound Scan (USS) is reported as entirely normal.
_x000D_
Which of the following is the correct advice for him?
CorrectIncorrectHint
His chance of having Autosomal Dominant Polycystic Kidney Disorder (ADPKD) is less than 5%. This is based on Ultrasound Scan criteria developed by screening a cohort of patients at risk for Autosomal Dominant Polycystic Kidney Disorder (ADPKD). The findings that should lead to the diagnosis being made are:
_x000D_
- _x000D_
- Two Or More Unilateral Or Bilateral Cysts In Individuals Between 15 And 30 Years Of Age.
- Two Cysts In Each Kidney In Individuals 30 To 59 Years.
- Four Cysts In Each Kidney In Individuals 60 Years Or Older.
_x000D_
_x000D_
_x000D_
_x000D_
These have a Specificity of 100% with respect to diagnosis of the condition. Among patients aged between 15 and 30 years, Sensitivity is 95%, and at age 20 years the False-Negative Rate is said to be 4%. By age 30 years, Sensitivity increases to close to 100%; therefore, at this age the condition can definitely be excluded.
_x000D_
Screening at age younger than 15 years is not recommended. Screening is recommended because individuals may be asked to become Kidney Donors for other family members, and early and aggressive management of Hypertension in affected individuals may delay the onset of End-Stage Renal Failure (ESRF).
-
Question 28 of 121
28. Question
A 24-year-old gentleman developed bilateral loin pain and frank haematuria.
_x000D_
His symptoms had started 24 hours after developing a sore throat. His Blood Pressure was 140/90 mmHg. Urinalysis was Positive for Blood (4+) and Protein (2+).
_x000D_
What is the most likely diagnosis?
CorrectIncorrectHint
The acute onset of the disease is suggestive of IgA Nephropathy which characteristically occurs in young males in their 20s and 30s.
_x000D_
Haematuria occurs within 12-24 hours of Pharyngitis, accompanied also by Loin Pain, Muscle Pain and Fever. Prognosis is usually good especially in children. In adults, between 25-50% may develop End-Stage Renal Failure (ESRF). No specific treatment is available.
_x000D_
Classically, the patient has Streptococcal Infection one to three weeks before the onset of Acute Nephritic Syndrome (Post-Strep GN).
_x000D_
There is a Long Prodromal Systemic Illness lasting months or years in Microscopic Polyangiitis which differs from Granulomatosis with Polyangiitis in its absence of Respiratory Tract Granulomatous Inflammation.
_x000D_
_x000D_
-
Question 29 of 121
29. Question
A 23-year-old lady presents with joint pains and rash. On examination, her Blood Pressure was 150/100 mmHg.
_x000D_
Investigations Reveal:
_x000D_
_x000D_ _x000D_
_x000D_ _x000D_ Serum Creatinine
_x000D_
_x000D_
_x000D_ 91 µmol/l
_x000D_
_x000D_
_x000D_ (60-110)
_x000D_
_x000D_
_x000D_
_x000D_ _x000D_ Anti dsDNA Antibodies
_x000D_
_x000D_
_x000D_ Strongly Positive
_x000D_
_x000D_
_x000D_ (0-73)
_x000D_
_x000D_
_x000D_
_x000D_ _x000D_ 24 Hour Urinary Protein Excretion
_x000D_
_x000D_
_x000D_ 1.8 gm
_x000D_
_x000D_
_x000D_ (<0.2)
_x000D_
_x000D_
_x000D_
_x000D_ _x000D_ Renal Biopsy
_x000D_
_x000D_
_x000D_ Membranous Nephropathy
_x000D_
_x000D_
_x000D_
_x000D_
_x000D_
What is the most appropriate next treatment for her Nephropathy?
CorrectIncorrectHint
This patient has Systemic Lupus Erythematosus [SLE] along with Lupus Nephritis as suggested by Strongly Positive Anti dsDNA antibodies & presence of proteinuria. However if urinary RBC casts were mentioned in the question then it would become even easier to identify the renal involvement.
_x000D_
The Renal Manifestations of Systemic Lupus Erythematosus [SLE] are highly variable, ranging from Mild Asymptomatic Proteinuria and / or Haematuria to Rapidly Progressive Uraemia.
_x000D_
The various presentations are difficult to classify into clinical syndromes and histological classes. Although Lupus Nephritis affects a third of patients early in the disease it is frequently unrecognised until Nephritic and / or Nephrotic Syndrome with Renal Failure occur.
_x000D_
Histologically, a number of different types of renal disease are recognised in Systemic Lupus Erythematosus [SLE], with Immune-Complex Mediated Glomerular Disease being the most common. The up to date International Society of Nephrology / Renal Pathology Society 2003 classification divides these into six (06) different patterns:
_x000D_
- _x000D_
- Minimal Mesangial
- Mesangial Proliferative
- Focal
- Diffuse
- Membranous, and
- Advanced Sclerosis.
_x000D_
_x000D_
_x000D_
_x000D_
_x000D_
_x000D_
_x000D_
Patients with Membranous Lupus Nephritis tend to present with Nephrotic Syndrome.
_x000D_
- _x000D_
- Microscopic Haematuria and Hypertension may also be seen.
- Biopsies show Global or Segmental Subepithelial Immune Deposits or their Morphologic Sequelae, with or without Mesangial Alterations.
- It may occur in combination with Class III or IV, in which case both are diagnosed.
- Progression is variable, and immunosuppression is not always needed.
- Cyclophosphamide, Mycophenolate Mofetil and Azathioprine reduce mortality in proliferative forms of Lupus Glomerulonephritis, but the benefit is not clear in membranous forms.
_x000D_
_x000D_
_x000D_
_x000D_
_x000D_
_x000D_
More important to this patient’s renal disease in this patient is aggressive blood pressure control. An Angiotensin-Converting Enzyme (ACE) Inhibitor would be First Line, as it has been shown to reduce Proteinuria independently of its effect on Blood Pressure.
_x000D_
Warfarin is not considered an appropriate treatment as this lady has not exhibited any prothrombotic tendencies.
_x000D_
Non-Steroidal Anti-Inflammatory Medication would treat her Arthralgia but would have no effect on the prognosis of the disease.
_x000D_
Therefore the correct answer should be Angiotensin-Converting Enzyme (ACE) Inhibitor for Blood Pressure Control, but immunosuppression may well be required to manage her Extra-Renal Disease.
-
Question 30 of 121
30. Question
A 23-year-old lady, who is taking long-term Doxycycline for severe acne comes to the Clinic complaining of chronic thirst and polyuria. She has to pass urine three to four times per night, which is highly unusual for her.
_x000D_
There is no significant past medical history, and her only medication is the Oral Contraceptive Pill.
_x000D_
On examination, her lying Blood Pressure is 130/80 mmHg, with a postural drop of 15 mmHg. Her BMI is 29. There were no other significant findings on physical examination.
_x000D_
Investigations Show:
_x000D_
_x000D_ _x000D_
_x000D_ _x000D_ Haemoglobin
_x000D_
_x000D_
_x000D_ 140 g/L
_x000D_
_x000D_
_x000D_ (115-165)
_x000D_
_x000D_
_x000D_
_x000D_ _x000D_ Total Leucocyte Count
_x000D_
_x000D_
_x000D_ 6.5 ×109/L
_x000D_
_x000D_
_x000D_ (4-11)
_x000D_
_x000D_
_x000D_
_x000D_ _x000D_ Platelet Count
_x000D_
_x000D_
_x000D_ 224 ×109/L
_x000D_
_x000D_
_x000D_ (150-400)
_x000D_
_x000D_
_x000D_
_x000D_ _x000D_ Serum Sodium
_x000D_
_x000D_
_x000D_ 142 mmol/L
_x000D_
_x000D_
_x000D_ (135-146)
_x000D_
_x000D_
_x000D_
_x000D_ _x000D_ Serum Potassium
_x000D_
_x000D_
_x000D_ 4.9 mmol/L
_x000D_
_x000D_
_x000D_ (3.5-5)
_x000D_
_x000D_
_x000D_
_x000D_ _x000D_ Serum Creatinine
_x000D_
_x000D_
_x000D_ 125 μmol/L
_x000D_
_x000D_
_x000D_ (79-118)
_x000D_
_x000D_
_x000D_
_x000D_ _x000D_ Serum Urea
_x000D_
_x000D_
_x000D_ 8.7 mmol/L
_x000D_
_x000D_
_x000D_ (2.5-6.7)
_x000D_
_x000D_
_x000D_
_x000D_
_x000D_
Which of the following is the most appropriate treatment?
CorrectIncorrectHint
The history of Polydipsia, Polyuria and Nocturia, in this case, is typical of Diabetes Insipidus. From the information given in the stem, we can suggest this is Nephrogenic Diabetes Insipidus secondary to Doxycycline. Other drugs which can cause this include Lithium, some Antifungals and some Anti-Virals.
_x000D_
Whilst increasing her Oral Fluid Intake may relieve some of her symptoms of polydipsia, discontinuing the Doxycycline is the only way to resolve both her Polydipsia and Polyuria.
_x000D_
High Dose Intranasal Vasopressin may relieve her symptoms but the most appropriate treatment is discontinuation of the Tetracycline Antibiotic.
-
Question 31 of 121
31. Question
A 22-year-old lady presents to the Acute Medical Intake with lethargy and confusion.
_x000D_
On examination, she has a purpuric rash covering the abdominal wall and thighs and a fever of 38°C. Investigations reveal a Haemolytic Anaemia, Thrombocytopenia and Acute Kidney Injury (AKI).
_x000D_
Which feature of the presentation makes the diagnosis of Thrombotic Thrombocytopenic Purpura more (TTP) likely than Haemolytic-Uraemic Syndrome (HUS)?
CorrectIncorrectHint
Thrombotic Thrombocytopenic Purpura (TTP) and Haemolytic-Uraemic Syndrome (HUS) have overlapping clinical features, with up to 60% of Thrombotic Thrombocytopenic Purpura (TTP) patients missing at least one component of the classical pentad and around 30% of Haemolytic Uraemic-Syndrome (HUS) patients having Neurological Symptoms and Fever.
_x000D_
Acute Kidney Injury (AKI) can be present in both conditions. It is typically more severe in Haemolytic-Uraemic Syndrome (HUS). Neurological Symptoms are much less common in patients presenting with Haemolytic-Uraemic Syndrome (HUS).
_x000D_
Fever, Haemolytic Anaemia, and Thrombocytopenia can be present in both conditions.
_x000D_
_x000D_
_x000D_
_x000D_
-
Question 32 of 121
32. Question
A 22-year-old lady at 16 Weeks (Second Trimester) into her first pregnancy presents at the Obstetrician’s Clinic with a Blood Pressure of 145/98 mmHg for routine antenatal check-up. A 24-Hour Urine Collection reveals a Protein Excretion of 0.7 g/day (<0.2).
_x000D_
Which of the following is the most likely explanation for these findings?
CorrectIncorrectHint
This pregnant lady has Hypertension and Proteinuria (as defined by a Blood Pressure above 140/90 mmHg and Proteinuria above 0.3 g/day). It would not be regarded as normal to have this high Blood Pressure or Urine Protein content.
_x000D_
Hypertensive Disorders during pregnancy occur in women with Pre-existing Primary or Secondary Hypertension and in women who develop New-onset Hypertension (defined as those cases occurring after 20 weeks).
_x000D_
All patients who have Hypertension during pregnancy are at increased risk of Eclampsia. Although the rate of this is decreasing, Hypertension in pregnancy remains one of the leading causes of maternal death. In addition, it can lead to Stillbirth, Preterm Birth, and Small For Gestational Age Babies.
_x000D_
In pregnancy, Chronic Hypertension is defined as Hypertension that is present at the booking visit or before 20 weeks of gestation, or if the woman is already taking antihypertensive medication when she conceives. It can be primary or secondary in aetiology.
_x000D_
Gestational Hypertension is new Hypertension which presents after 20 weeks without Significant Proteinuria.
_x000D_
Pre-eclampsia is new Hypertension presenting after 20 weeks with significant proteinuria. Severe pre-eclampsia is pre-eclampsia with Severe Hypertension (>160/110 mmHg) and / or with symptoms [Headache, Visual Disturbance (Blurring of Vision), Abdominal Pain (especially Epigastric Pain), Nausea, Vomiting, Breathlessness, and Oedema), and / or Biochemical and Haematological Impairment.
_x000D_
The presence of Significant Proteinuria in Chronic Hypertension should alert the Obstetrician to suspect underlying Renal Disease, especially in a young antenatal mother. Significant Proteinuria is defined as a Urinary Protein: Creatinine Ratio greater than 30 mg/mmol or a 24-Hour Urine Collection of more than 300 mg Protein. In pregnancy, it only indicates pre-eclampsia if it develops after 20 weeks gestation.
_x000D_
Management of Chronic Hypertension in pregnancy involves determining the cause (if the Obstetrician suspects it to be Secondary), Restricting Dietary Sodium, Regular Antenatal Check-Ups, and Anti-Hypertensive Treatment. Blood Pressure should be kept less than 150/100 mmHg, unless there is evidence of target-organ damage in which case the Blood Pressure should be kept less than 140/90 mmHg.
_x000D_
Antenatal mothers with Chronic Hypertension are at increased risk of developing pre-eclampsia and are therefore prescribed 75 mg of Aspirin daily from 12 weeks, which is believed to reduce the risk. Angiotensin-Converting Enzyme (ACE) Inhibitors and Angiotensin-II Receptor Blockers are avoided due to their increased risk of Congenital Malformations. In general, treatment should be overseen by an Obstetrician if there is Primary Hypertension, and a Renal Physician or Endocrinologist if Secondary Hypertension is suspected.
_x000D_
-
Question 33 of 121
33. Question
A 22-year-old Ice Skate Player presents to the Sports Medicine Unit with frank haematuria that began some 48 hours after an Upper Respiratory Tract Infection.
_x000D_
On examination, he is apyrexial, Blood Pressure is 110/70 mmHg and Pulse Rate is 70/Minute. His chest is clear and his abdomen is soft and non-tender.
_x000D_
Investigations Reveal:
_x000D_
_x000D_ _x000D_
_x000D_ _x000D_ Haemoglobin
_x000D_
_x000D_
_x000D_ 142 g/L
_x000D_
_x000D_
_x000D_ (135-177)
_x000D_
_x000D_
_x000D_
_x000D_ _x000D_ Total Leucocyte Count
_x000D_
_x000D_
_x000D_ 8.2 ×109/L
_x000D_
_x000D_
_x000D_ (4-11)
_x000D_
_x000D_
_x000D_
_x000D_ _x000D_ Platelet Count
_x000D_
_x000D_
_x000D_ 250 ×109/L
_x000D_
_x000D_
_x000D_ (150-400)
_x000D_
_x000D_
_x000D_
_x000D_ _x000D_ Serum Sodium
_x000D_
_x000D_
_x000D_ 141 mmol/L
_x000D_
_x000D_
_x000D_ (135-146)
_x000D_
_x000D_
_x000D_
_x000D_ _x000D_ Serum Potassium
_x000D_
_x000D_
_x000D_ 4.2 mmol/L
_x000D_
_x000D_
_x000D_ (3.5-5)
_x000D_
_x000D_
_x000D_
_x000D_ _x000D_ Serum Creatinine
_x000D_
_x000D_
_x000D_ 88 µmol/L
_x000D_
_x000D_
_x000D_ (79-118)
_x000D_
_x000D_
_x000D_
_x000D_ _x000D_ Urine Dipstick
_x000D_
_x000D_
_x000D_ Blood +++
_x000D_
_x000D_
_x000D_
_x000D_
_x000D_
Which of the following is the most appropriate way to manage him?
CorrectIncorrectHint
This boy almost certainly has IgA Nephropathy, given his presentation with frank haematuria occurring in such close proximity to a recent Upper Respiratory Tract Infection.
_x000D_
The findings including normal Blood Pressure, normal Serum Creatinine and Haematuria without Proteinuria point towards a benign prognosis and for this reason he only requires regular monitoring rather than intervention with Immunosuppressive Agents.
_x000D_
Treatment with Corticosteroids is usually reserved for those patients with Hypertension and a Rising Serum Creatinine.
_x000D_
Where there is significant Hypertension and / or Proteinuria or both, then Angiotensin-Converting Enzyme (ACE) Inhibitors are the First Choice Antihypertensive Agent.
_x000D_
Post-Streptococcus Glomerulonephritis usually occurs one to three weeks after initial infection.
_x000D_
_x000D_
-
Question 34 of 121
34. Question
A 20-year-old lady attends the Clinic as her father has just died with End-Stage Renal Failure (ESRF). He had been diagnosed with Autosomal Dominant Polycystic Kidney Disease (ADPKD).
_x000D_
She is concerned of what investigations she requires.
_x000D_
Which of the following is an appropriate strategy in her management?
CorrectIncorrectHint
Important thing to see here is the age of the subject in question.
_x000D_
The sensitivity of ultrasonography to detect renal cysts (in a carrier of the ADPKD trait) depends on 1. The quality of the ultrasonography machine (with outdated machines assessments may not be reliable) and 2.Age of the propositus (first degree relative to be assessed for renal cysts).
_x000D_
As a rule of thumb, in individuals aged 30 years, cysts are demonstrable by ultrasound in approximately 68% and above 30 years of age in approximately 89% of carriers of the genetic trait.
_x000D_
There are two types of potential misdiagnosis.
_x000D_
- _x000D_
- In very young individuals, the diagnosis may be missed because the renal parenchyma presents with a hyperdense texture only, but even at this early stage hilar and intrarenal vessels exhibit unusually strong vascular reflexes._x000D_
- _x000D_
- Occasionally a CT scan permits detection of very small incipient cysts in such cases.
_x000D_
_x000D_
_x000D_
_x000D_
Ultrasonography Diagnostic Criteria according to age:
_x000D_
- _x000D_
- In PKD 1 Families, Age-Related Diagnostic Criteria Are Used: Two Cysts In Under 30 Years Age Group.
- At Least Two Cysts In Each Kidney In 30-59 Years Age Group. Four Cysts In Each Kidney For Over 60 Years Group.
_x000D_
_x000D_
_x000D_
An Ultrasound Scan of the Renal Tract, hence, may not be appropriate at this patient’s age, given that cysts may not become apparent until the age of 20 years.
_x000D_
Gross Haematuria in Autosomal Dominant Polycystic Kidney Disorder (ADPKD) carries a poor prognosis however Microscopic Haematuria may be a complication.
_x000D_
Subarachnoid Haemorrhage (SAH) may be a cause of mortality in 9% of patients with Autosomal Dominant Polycystic Kidney Disorder (ADPKD), though 8% of patients have an Asymptomatic Intracranial Aneurysm; if the diagnosis is confirmed and there is a strong history of Subarachnoid Haemorrhage (SAH) then a Magnetic Resonance Imaging (MRI) Scan would be indicated.
_x000D_
Genetic Counselling is most appropriate in this context and Genetic Linkage Analysis may be utilised.
-
Question 35 of 121
35. Question
A 17-year-old girl presents to the Clinic with a two-year history of recurrent colicky loin pain. One year ago she passed a renal calculus.
_x000D_
24-Hour Urine collection showed Normal Levels of Calcium, Phosphate and Urate, but Elevated Levels of Arginine, Cystine, Lysine and Ornithine.
_x000D_
Which one of the following features is characteristic of this condition?
CorrectIncorrectHint
This condition is typical of Cystinuria (Nephropathic Cystinosis), an Autosomal Recessive Genetic Defect in Membrane Transport for Cystine, Lysine, Ornithine and Arginine in Epithelial Cells. The Glomerulus is unable to resorb these Amino Acids, and they are therefore excreted into the Urine. The rBAT Gene is responsible, and there are three forms distinguished by the pattern of Tubular Amino Acid Transport.
_x000D_
Cystinuria usually presents with Recurrent Nephrolithiasis in the form of Cystine Stones (which are often Bilateral, Multiple, and can form Staghorns). These can present as early as the first decade. Renal Failure can occur. The stones are Radiolucent Stones, which may also be seen with Uric Acid Stones. However, they are usually opaque on a Computed Tomography Scan (Kidney-Ureter-Bladder) (CT KUB). Cystine Deposits within the Cornea are not classically seen, neither are Functional Defects Within The Glomeruli.
_x000D_
Diagnosis of Cystinuria can be made by Stone Analysis; such stones are pale yellow and analysis reveals High Cystine Levels. It can then be confirmed by an Amino Acid Chromatogram and Quantification Of Cystine Excretion.
_x000D_
First-line Management is conservative, with encouragement of large volume fluid intake (particularly in the evening, with the aim to pass urine at least once overnight). Urine pH should be regularly monitored (aiming for 7.5-8), with Sodium Bicarbonate being used if necessary (Not In Hypertensive Patients Or Those With Renal Failure).
_x000D_
The aim of such treatment is to reduce the Urinary Cystine Concentration to below 300 mg/L. If this fails, d-Penicillamine, Alpha-Mercaptopropionyl Glycine or Captopril can be used.
_x000D_
Cystine Stones are not easily broken by Lithotripsy, and therefore Percutaneous Removal is most often used if they do develop.
_x000D_
-
Question 36 of 121
36. Question
A 17-year-old boy presents with Periorbital and Peripheral Oedema which has developed over the past three weeks. He had a previous episode some three years earlier but this responded over the course of a few weeks to a course of Prednisolone.
_x000D_
On examination, he has a Blood Pressure of 140/80 mmHg, with Pulse Rate of 70/Minute. His chest is clear but he has Bilateral Pitting Oedema.
_x000D_
Investigations Show:
_x000D_
_x000D_ _x000D_
_x000D_ _x000D_ Haemoglobin
_x000D_
_x000D_
_x000D_ 130 g/L
_x000D_
_x000D_
_x000D_ (135-180)
_x000D_
_x000D_
_x000D_
_x000D_ _x000D_ White Blood Cell Count
_x000D_
_x000D_
_x000D_ 6.3 ×109/L
_x000D_
_x000D_
_x000D_ (4-10)
_x000D_
_x000D_
_x000D_
_x000D_ _x000D_ Platelet Count
_x000D_
_x000D_
_x000D_ 211 ×109/L
_x000D_
_x000D_
_x000D_ (150-400)
_x000D_
_x000D_
_x000D_
_x000D_ _x000D_ Serum Sodium
_x000D_
_x000D_
_x000D_ 141 mmol/L
_x000D_
_x000D_
_x000D_ (134-143)
_x000D_
_x000D_
_x000D_
_x000D_ _x000D_ Serum Potassium
_x000D_
_x000D_
_x000D_ 4.9 mmol/L
_x000D_
_x000D_
_x000D_ (3.5-5)
_x000D_
_x000D_
_x000D_
_x000D_ _x000D_ Serum Creatinine
_x000D_
_x000D_
_x000D_ 112 μmol/L
_x000D_
_x000D_
_x000D_ (60-120)
_x000D_
_x000D_
_x000D_
_x000D_ _x000D_ Serum Albumin
_x000D_
_x000D_
_x000D_ 25 g/L
_x000D_
_x000D_
_x000D_ (30-50)
_x000D_
_x000D_
_x000D_
_x000D_ _x000D_ Urine Dipstick Analysis
_x000D_
_x000D_
_x000D_ Protein ++
_x000D_
_x000D_
_x000D_
_x000D_
_x000D_
Which one of the following represents the most appropriate management plan for him?
CorrectIncorrectHint
The age of this patient and his previous response to Corticosteroids is suggestive of an underlying diagnosis of Minimal Change Nephropathy. As such, a further course of Corticosteroids is the treatment of choice.
_x000D_
Given the inherent but low risks associated with Renal Biopsy, it is usually only attempted when three or more episodes of oedema have occurred.
_x000D_
Very few patients with Minimal Change Disease actually progress to End-Stage Renal Disease (ESRD) and only around 10% of children with the disease suffer from Hypertension.
-
Question 37 of 121
37. Question
What is the most likely outcome of Minimal Change Nephropathy with onset at 12 years of age?
CorrectIncorrectHint
30-40% percent of children achieve spontaneous remission and 90% achieve remission following eight weeks treatment with High Dose Steroids.
_x000D_
The disease is less common in adults, but over 80% of adults also achieve remission following treatment with Oral Glucocorticoids.
_x000D_
_x000D_
-
Question 38 of 121
38. Question
Which of the following is the best Imaging to identify Renal Scarring, for instance after Childhood Febrile Urinary Tract Infection?
CorrectIncorrectHint
Renal Scintigraphy with Dimercaptosuccinic Acid (DMSA) involves administration of Radioactive Isotope which is avidly taken up by the Renal Parenchyma. This permits the identification of regions of decreased uptake that may represent acute inflammation (such as Pyelonephritis) or Renal Scarring.
_x000D_
The technique of Dimercaptosuccinic Acid Scan (DMSA Scan) also allows detection of Congenital Renal Disorder.
_x000D_
A Small Kidney with uniform uptake of Dimercaptosuccinic Acid (DMSA) is likely to represent Congenital Hypodysplasia, whereas a Focal Area of Reduced Cortical Uptake associated with Loss Of Contours is more likely to represent an Infection-Related Scar.
_x000D_
Renal DTPA Scan involves an Isotope that is exclusively filtered by the Glomeruli, and is used to give a “Perfusion Index” and evaluate excretion (Obstruction Assessment) of the kidney.
_x000D_
_x000D_
Voiding Cystourethrography is used to determine whether there is Vesicoureteral Reflux, which might give an increased risk (but not diagnostic) of Renal Scarring.
_x000D_
Ultrasonography also does not reliably detect Low-Grade Scarring.
_x000D_
-
Question 39 of 121
39. Question
Which of the following is a feature of Cystinuria?
CorrectIncorrectHint
Cystinuria is the Commonest Inborn Error of Amino Acid Transport.
_x000D_
Amino Acids excreted in Urine are Cystine, Ornithine, Arginine and Lysine (Mnemonic – COAL).
_x000D_
The Renal Stones are Radio-Opaque due to the presence of Sulphur. On Plain Film, which is not used as much in the United Kingdom anymore, they are Radiolucent. On Computed Tomography, as with almost all Stones, Cystine Stones are Radio-Opaque.
_x000D_
It is inherited as an Autosomal Recessive Condition.
_x000D_
Management includes Alkalinisation along with High Fluid Intake (>4 Litres / Day); d-Penicillamine may also be used.
_x000D_
It is Cystinosis that leads to accumulation of Cystine in the Kidney.
-
Question 40 of 121
40. Question
Which one of the following statements regarding renal function is correct?
CorrectIncorrectHint
A ten minute period of Hyperventilation would cause a Respiratory Alkalosis leading to an Increased Secretion Of Bicarbonate and Retention Of Hydrogen Ions.
_x000D_
Arginine Vasopressin (AVP) acts on the Collecting Ducts increasing permeability to water.
_x000D_
The Total Solute Excretion is approximately 700 mosmol/d.
_x000D_
Sodium reabsorption is mostly through Active Transport in the Loop Of Henle with only a modest reabsorption facilitated by Aldosterone.
_x000D_
The Rate of Ammonium Excretion is Proportional To The Rate Of Hydrogen Ion Excretion.
_x000D_
-
Question 41 of 121
41. Question
A 23-year-old lady presents with features of nephrotic syndrome and a renal biopsy is performed.
_x000D_
If a diagnosis of Minimal Change Disease is expected, what would is expected to be seen on Light Microscopy and Electron Microscopy?
CorrectIncorrectHint
Minimal Change Disease is typically seen in children less than 6-years-old and in a minority of adults with Nephrotic Syndrome.
_x000D_
There is no Glomerular Abnormality on Light Microscopy.
_x000D_
Electron Microscopy shows Effacement of the Foot Processes of Podocytes (Visceral Epithelial Cells) and a Normal Basement Membrane.
_x000D_
Treatment is with steroids; disease remission often occurs within two weeks although relapse may occur.
_x000D_
- _x000D_
- Fusion of Foot Processes of Podocytes is seen on Electron Microscopy, not Light Microscopy.
- There is no Hyalinisation; this is an abnormality seen in other renal diseases, such as Amyloidosis.
- The Glomerular Basement Membrane is Normal on Electron Microscopy, though there is an abnormality of Podocytes with Fusion of Foot Processes.
- Tubules may show accumulation of Lipid in Lining Cells on Light Microscopy, but not Calcification.
_x000D_
_x000D_
_x000D_
_x000D_
_x000D_
-
Question 42 of 121
42. Question
A 28-year-old gentleman presents to Hospital with complaint of feeling unwell, with a few days history of diarrhoea and abdominal pain. He reports having eaten at a ‘Burger Van’ a few days ago.
_x000D_
He has no previous hospitalisation. His initial Laboratory Test Reports show New Onset Renal Impairment, Anaemia and Low Platelets. His clotting is normal.
_x000D_
Which of the following pathogens is most likely to be responsible for this presentation?
CorrectIncorrectHint
The question describes the clinical picture of the Haemolytic Uraemic Syndrome with a diarrhoea prodrome. Escherichia coli is the most likely responsible pathogen.
_x000D_
Haemolytic Uraemic Syndrome (HUS) is a syndrome composing of the Triad of:
_x000D_
- _x000D_
- Microangiopathic Haemolytic Anaemia
- Low Platelets, and
- Renal Failure.
_x000D_
_x000D_
_x000D_
_x000D_
A number of pathogens are implicated including Escherichia coli, usually the O157:H7 Subtype. This produces a Verotoxin (also known as Shiga Toxin) which binds to Endothelial Receptors, particularly in the Renal, Gastrointestinal and Central Nervous Systems (CNS) thereby causing the clinical symptoms. Thrombin and Fibrin are deposited in the Microvasculature and cause Haemolysis of Circulating Erythrocytes. Platelets are also sequestered.
_x000D_
The other pathogens which can cause Haemolytic Uraemic Syndrome (HUS) are Streptococcus pneumoniae, Shigella dysenteriae (Type 1 and 3), Human Immunodeficiency Virus (HIV) and Coxsackie Virus.
_x000D_
This presentation is not classical of C. difficile as we are given no risk factors for this in the question, so this option is incorrect.
_x000D_
Enterococcus faecalis is a Bowel Commensal which is unlikely to cause the degree of pathology described in this case.
_x000D_
There is no reason to suspect this gentleman is Methicillin Resistant Staphylococcus aureus (MRSA) positive.
_x000D_
Streptococcus viridans is an Upper Respiratory Tract and Throat Bacteria. This patient’s presentation does not involve that organ system so this option is incorrect.
_x000D_
_x000D_
-
Question 43 of 121
43. Question
A 26-year-old lady wishes to start a family but she is concerned as her 50-year-old mother had Adult Polycystic Kidney Disease (APCKD).
_x000D_
Examination reveals no specific abnormalities.
_x000D_
Which is the most appropriate initial screening test for Polycystic Kidney Disease (PKD) in this lady?
CorrectIncorrectHint
The answer lies between Renal Ultrasound Scan and Genetic Linkage Analysis.
_x000D_
Diagnosis is made by Multiple Bilateral Renal Cysts and a Positive Family History.
_x000D_
Genetic Linkage Studies can exclude or make the diagnosis in younger patients. The studies require blood from at least two affected family members.
_x000D_
Ultrasonography:
_x000D_
- _x000D_
- In PKD 1 Families, Age-Related Diagnostic Criteria Are Used: Two Cysts In Under 30 Years Age Group.
- At Least Two Cysts In Each Kidney In 30-59 Years Age Group. Four Cysts In Each Kidney For Over 60 Years Group.
_x000D_
_x000D_
_x000D_
- _x000D_
- Computed Tomography is more sensitive than Ultrasound Scan (USS) and may aid in diagnosis in younger patients.
_x000D_
_x000D_
Polycystic Kidney Disease (PKD) is an Autosomal Dominant Disorder. Mutations in at least three different genes can lead to Autosomal Dominant Polycystic Disease (APKD).
_x000D_
- _x000D_
- PKD-1 On Chromosome 16: 85% Of Cases. Polycystin 1 Is An Integral Membrane Glycoprotein Involved In Cell/Matrix Interactions.
- PKD-2 On Chromosome 4: 10% Of Cases. Polycystin 2 Which May Associate With Polycystin 1 Through A Common Signalling Pathway.
- A Third Gene Mutation Is Known But Its Exact Chromosomal Location Is Not.
_x000D_
_x000D_
_x000D_
-
Question 44 of 121
44. Question
A 25-year-old lady who is 22 weeks pregnant is diagnosed with Pyelonephritis.
_x000D_
She had suffered from Recurrent Urinary Infections since childhood and her family history reveals that her mother had a history of Hypertension and had been told she had a Kidney Problem.
_x000D_
Examination was normal and Serum Urea and Serum Creatinine were both normal.
_x000D_
What is the most likely diagnosis?
CorrectIncorrectHint
This lady has had Recurrent Urinary Tract Infections since childhood and now presents with Pyelonephritis.
_x000D_
Pyelonephritis is an uncommon infection in pregnancy and requires aggressive treatment with antibiotics.
_x000D_
It is associated with preterm labour in 4% and may lead to fetal distress.
_x000D_
Renal Stone Disease does predispose to developing Urinary Tract Infections but is less likely than Reflux Nephropathy.
_x000D_
The correct answer is Reflux Nephropathy. This lady has a long history of Urinary Tract Infections with probable underlying Reflux Scarring and this would predispose her to developing Pyelonephritis in pregnancy.
_x000D_
The answer is not Autosomal Dominant Polycystic Kidney Disease (ADPKD) as symptoms do not tend to occur before the age of 40 years.
_x000D_
Bladder Outlet Obstruction should not occur in pregnancy and would cause Hydronephrosis (HDN) and Worsening Renal Function.
_x000D_
Normal Physiological Urinary Stasis Of Pregnancy is incorrect as this should not cause Pyelonephritis.
-
Question 45 of 121
45. Question
A 25-year-old lady with Systemic Lupus Erythematosus (SLE) (Anti-Nuclear Antibody Positive [1:6400], Anti-dsDNA Antibody Positive) presents with a three weeks’ history of feeling generalised illness, feeling of being tired, worsening malar rash, and has mild pedal oedema.
_x000D_
On examination, her Blood Pressure is 180/100 mm Hg, and there are 3+ Proteins, 3+ Blood in her Urine.
_x000D_
What is the most likely diagnosis?
CorrectIncorrectHint
Hypertension, Pedal Oedema, Nephritic Urinary Sediments (Blood and Protein Positive) in a patient with Systemic Lupus Erythematosus (SLE) suggests the diagnosis of Class IV (Diffuse Proliferative Glomerulonephritis) or Class III (Focal Segmental Glomerulonephritis) Lupus Nephritis. ds-DNA Antibodies increase the risk of Lupus Nephritis.
_x000D_
Mesangial Glomerulonephritis presents with Mild Proteinuria.
_x000D_
Minimal Change Glomerulonephritis, or Membranous Glomerulonephritis present with Proteinuria, which may be in the Nephrotic Range.
_x000D_
The Renal Manifestations of Systemic Lupus Erythematosus (SLE) are highly variable, ranging from Mild Asymptomatic Proteinuria and / or Haematuria to Rapidly Progressive Uraemia. The various presentations are difficult to classify into clinical syndromes and histological classes. Although Lupus Nephritis affects a third of patients early in the disease it is frequently unrecognised until Nephritic and / or Nephrotic Syndrome with Renal Failure occur.
_x000D_
_x000D_
_x000D_
Histologically, a number of different types of renal diseases are recognised in Systemic Lupus Erythematosus (SLE), with Immune-Complex Mediated Glomerular Disease being the most common.
_x000D_
The Standard Classification divides these into Five Different Patterns:
_x000D_
- _x000D_
- I – No Disease.
- II – Mesangial.
- III – Focal Proliferative.
- IV – Diffuse Proliferative.
- V – Membranous.
_x000D_
_x000D_
_x000D_
_x000D_
_x000D_
_x000D_
Mesangial Nephritis represents the earliest and mildest form of Glomerular Involvement. It presents clinically as Microscopic Haematuria and / or Proteinuria. Hypertension is uncommon and Nephrotic Syndrome and Renal Impairment are very rarely seen. Biopsy demonstrates segmental areas of increased Mesangial Matrix and Cellularity. The prognosis is good and specific treatment is indicated only if the disease progresses.
_x000D_
Focal Proliferative Disease is more advanced, but still affects less than 50% of Glomeruli. Haematuria and Proteinuria is almost always seen, and Nephrotic Syndrome, Hypertension and Elevated Serum Creatinine may be present. Electron Microscopy shows Immune Deposits in the Subendothelial Space of the Glomerular Capillary Wall and the Mesangium. Prognosis is variable.
_x000D_
Diffuse Proliferative Glomerulonephritis is the most common and severe form of Lupus Nephritis. Haematuria and Proteinuria are almost always present, and Nephrotic Syndrome, Hypertension and Renal Impairment are common. Biopsies demonstrate profuse deposits of IgG within the Glomeruli. Immunosuppressive Therapy is required in these cases to prevent progression to End-Stage Renal Failure (ESRF).
_x000D_
Patients with Membranous Lupus Nephritis tend to present with Nephrotic Syndrome. Microscopic Haematuria and Hypertension may also be seen. Biopsies show diffuse thickening of the Glomerular Capillary Wall. Progression is variable, and immunosuppression is not always needed.
_x000D_
With regard to the management of Lupus Nephritis, a biopsy is indicated in those patients with Abnormal Urinalysis and / or Reduced Renal Function. This can provide a histological classification as well as information regarding activity, chronicity and prognosis.
_x000D_
Cyclophosphamide, Mycophenolate Mofetil and Azathioprine reduce mortality in proliferative forms of Lupus Glomerulonephritis.
_x000D_
-
Question 46 of 121
46. Question
A 26-year-old gentleman presents to the Nephrology Unit with proteinuria, haematuria, and sensorineural deafness.
_x000D_
Which of the following protein structures is likely to be abnormal?
CorrectIncorrectHint
The diagnosis is Alport’s Syndrome, which is a disorder of Type 4 Collagen assembly and is inherited as an X-Linked disorder in 85% of cases.
_x000D_
Fibrillin Gene abnormalities are associated with Marfan’s Syndrome.
_x000D_
Type 1 Collagen disorders are associated with Osteogenesis Imperfecta; it is the main type of collagen in tendon and bone.
_x000D_
Type 3 Collagen is the main component of Reticular Fibres.
_x000D_
-
Question 47 of 121
47. Question
A 25-year-old gentleman is referred to the Nephrology Clinic with Microscopic Haematuria. He also has Hypertension, which the General Practitioner diagnosed as Essential Hypertension and commenced him on Amlodipine 5 mg daily.
_x000D_
Further questioning reveals that his brother has Haematuria and Renal Impairment and his father died of a Stroke at the age of 43 years. On examination, his Blood Pressure is 150/90 mmHg, he has Bilateral Ballotable Kidneys.
_x000D_
Investigations Show:
_x000D_
_x000D_ _x000D_
_x000D_ _x000D_ Haemoglobin
_x000D_
_x000D_
_x000D_ 134 g/L
_x000D_
_x000D_
_x000D_ (135-180)
_x000D_
_x000D_
_x000D_
_x000D_ _x000D_ Total Leucocyte Count
_x000D_
_x000D_
_x000D_ 6.0 ×109/L
_x000D_
_x000D_
_x000D_ (4-10)
_x000D_
_x000D_
_x000D_
_x000D_ _x000D_ Platelet Count
_x000D_
_x000D_
_x000D_ 242 ×109/L
_x000D_
_x000D_
_x000D_ (150-400)
_x000D_
_x000D_
_x000D_
_x000D_ _x000D_ Serum Sodium
_x000D_
_x000D_
_x000D_ 140 mmol/L
_x000D_
_x000D_
_x000D_ (134-143)
_x000D_
_x000D_
_x000D_
_x000D_ _x000D_ Serum Potassium
_x000D_
_x000D_
_x000D_ 4.7 mmol/L
_x000D_
_x000D_
_x000D_ (3.5-5)
_x000D_
_x000D_
_x000D_
_x000D_ _x000D_ Serum Creatinine
_x000D_
_x000D_
_x000D_ 162 μmol/L
_x000D_
_x000D_
_x000D_ (60-120)
_x000D_
_x000D_
_x000D_
_x000D_ _x000D_ Urine Dipstick
_x000D_
_x000D_
_x000D_ Blood ++
_x000D_
_x000D_
_x000D_
_x000D_ _x000D_ Protein –
_x000D_
_x000D_
_x000D_
_x000D_
_x000D_
Which of the following is the investigation most appropriate to elucidate the underlying diagnosis?
CorrectIncorrectHint
The fact that his father and brother are both affected is suggestive of an Autosomal Dominant Inherited Disorder, probably Autosomal Dominant Polycystic Kidney Disorder (ADPKD).
_x000D_
Given that his father had a stroke at a young age, this raises the possibility of a Berry Aneurysm and subsequent Subarachnoid Haemorrhage (SAH).
_x000D_
Autosomal Dominant Polycystic Kidney Disorder (ADPKD) affects as many as 1 in 1000 individuals and is responsible for around 10% of the United Kingdom Dialysis Burden.
_x000D_
Polycystic Liver Disease is seen in 80% of patients but is usually asymptomatic. Abdominal Ultrasound has a sensitivity approaching 100% for patients above 20 years of age.
_x000D_
Hypertension should be aggressively managed, with Angiotensin-Converting Enzyme (ACE) Inhibitors the therapy of choice.
_x000D_
-
Question 48 of 121
48. Question
A 24-year-old lady presents with complaints of right loin pain, oliguria and coke-coloured urine for about one week.
_x000D_
Blood Pressure is increased at 160/80 mmHg and Urinary Dipstick shows Protein 4+ and Blood 4+. Microscopy is awaited.
_x000D_
Blood Tests Reveal:
_x000D_
_x000D_ _x000D_
_x000D_ _x000D_ Haemoglobin
_x000D_
_x000D_
_x000D_ 124 g/L
_x000D_
_x000D_
_x000D_ (115 – 165)
_x000D_
_x000D_
_x000D_
_x000D_ _x000D_ Total Leucocyte Count
_x000D_
_x000D_
_x000D_ 8.9 ×109/L
_x000D_
_x000D_
_x000D_ (4 – 11)
_x000D_
_x000D_
_x000D_
_x000D_ _x000D_ Platelet Count
_x000D_
_x000D_
_x000D_ 421 ×109/L
_x000D_
_x000D_
_x000D_ (150 – 400)
_x000D_
_x000D_
_x000D_
_x000D_ _x000D_ Serum Sodium
_x000D_
_x000D_
_x000D_ 138 mmol/L
_x000D_
_x000D_
_x000D_ (137 – 144)
_x000D_
_x000D_
_x000D_
_x000D_ _x000D_ Serum Potassium
_x000D_
_x000D_
_x000D_ 5.9 mmol/L
_x000D_
_x000D_
_x000D_ (3.5 – 4.9)
_x000D_
_x000D_
_x000D_
_x000D_ _x000D_ Serum Creatinine
_x000D_
_x000D_
_x000D_ 265 μmol/L
_x000D_
_x000D_
_x000D_ (60 – 110)
_x000D_
_x000D_
_x000D_
_x000D_ _x000D_ Serum Urea
_x000D_
_x000D_
_x000D_ 24.3 mmol/L
_x000D_
_x000D_
_x000D_ (2.5 – 7.5)
_x000D_
_x000D_
_x000D_
_x000D_
_x000D_
What is the most appropriate next investigation of this lady?
CorrectIncorrectHint
Although this lady has features of Nephritic Syndrome and may warrant Biopsy in the future, a Renal Ultrasound Scan is the best next investigation.
_x000D_
Renal Ultrasound allows one to assess Kidney Size, the presence of Hydronephrosis (HDN), Cysts or Tumours and any developmental abnormalities. Doppler Traces also allow Renal Blood Flow to be assessed.
_x000D_
If a clear identifiable precipitant is identified as the cause of the Acute Kidney Injury (AKI), an Ultrasound Scan may not be appropriate.
_x000D_
If a patient has Acute Urinary Retention and Serum Creatinine falls spontaneously with the insertion of a Catheter, they do not necessarily need an Ultrasound Scan.
_x000D_
If a patient has had an insult such as Profound Hypotension or has Acute Kidney Injury (AKI) in the context of Sepsis with a previously Normal Serum Creatinine, an Ultrasound Scan (USS) will not add much to the diagnosis unless they fail to recover with time.
_x000D_
However, the above patient warrants Ultrasound Imaging.
_x000D_
- _x000D_
- A Dimercaptosuccinic Acid (DMSA) Scan is not indicated for this patient.
- A Plain Abdominal KUB (Kidney-Ureter-Bladder) is less frequently used compared to a Computed Tomography (Kidney-Ureter-Bladder) (CT KUB) for Stone Disease, however, this patient has Nephritis.
- Angiography is not indicated for this patient.
- A Renal Biopsy may be indicated in the future, but an Ultrasound Scan (USS) should be performed first.
_x000D_
_x000D_
_x000D_
_x000D_
_x000D_
A Renal Ultrasound Scan should be performed in all patients with Acute Kidney Injury (AKI), unless a clear treatable cause is identified.
_x000D_
-
Question 49 of 121
49. Question
A 24-year-old lady presents with a 36-hour history of vomiting and epigastric pain. She has vomited over eight times in the last day.
_x000D_
There is no history of diarrhoea and the presumed diagnosis is Viral Gastroenteritis.
_x000D_
What is the most likely picture of her Acid Base Status?
CorrectIncorrectHint
Copious vomiting in this patient leads to loss of gastric contents which are predominantly acidic.
_x000D_
Loss of acid secretions from the stomach renders this patient deficient of acid in her blood; hence her Acid Base Status will be reflective of Metabolic Alkalosis. One may also expect a Lower Serum Potassium and Serum Chloride which are also lost in gastric secretions.
_x000D_
Primary Metabolic Acidosis, Mixed Metabolic Acidosis and Respiratory Alkalosis are incorrect because the patient is in acid deficit rather than acid excess.
_x000D_
There is no Primary Respiratory Pathology in the question and so Primary Respiratory Acidosis and Primary Respiratory Alkalosis are incorrect.
-
Question 50 of 121
50. Question
A 32-year-old gentleman presents to the Nephrology Clinic with Oedema and Proteinuria.
_x000D_
On examination, his Blood Pressure was 110/70 mmHg.
_x000D_
Investigations Reveal:
_x000D_
_x000D_ _x000D_
_x000D_ _x000D_ Serum Creatinine
_x000D_
_x000D_
_x000D_ 88 µmol/L
_x000D_
_x000D_
_x000D_ (60-110)
_x000D_
_x000D_
_x000D_
_x000D_ _x000D_ Serum Albumin
_x000D_
_x000D_
_x000D_ 25 g/L
_x000D_
_x000D_
_x000D_ (37-49)
_x000D_
_x000D_
_x000D_
_x000D_ _x000D_ Dipstick Urinalysis
_x000D_
_x000D_
_x000D_ No blood
_x000D_
_x000D_
_x000D_
_x000D_ _x000D_ Protein
_x000D_
_x000D_
_x000D_ ++++
_x000D_
_x000D_
_x000D_
_x000D_ _x000D_ Urinary Protein Excretion
_x000D_
_x000D_
_x000D_ 7g/24hr
_x000D_
_x000D_
_x000D_ (<0.2)
_x000D_
_x000D_
_x000D_
_x000D_
_x000D_
An Ultrasound Scan (USS) of Renal Tract shows Normal Right Kidney, Absent Left Kidney.
_x000D_
Which is the most appropriate course of action for this patient?
CorrectIncorrectHint
This patient has Nephrotic Syndrome, which is a combination of:
_x000D_
- _x000D_
- Proteinuria (Usually > 3 gm / 24 Hrs).
- Hypoalbuminaemia (< 35 gm / Litre).
- Oedema
- Hyperlipidaemia.
_x000D_
_x000D_
_x000D_
_x000D_
_x000D_
This patient has 7g/day proteinuria and Hypoalbuminemia, hence, the diagnosis.
_x000D_
_x000D_
In the young adult Histological Diagnoses are in general: Minimal Change > Mesangiocapillary > Focal Segmental Glomerulosclerosis (FSGS) > lupus > Membranous > Diabetes.
_x000D_
_x000D_
In the usual course, ideally a Renal Biopsy would be indicated as the next step to determine the cause of the Nephrotic Syndrome, however, as this patient has only one kidney, then this would be considered a relative contraindication for such a procedure.
_x000D_
The most appropriate course of action here would be to undergo a Trial of Steroid Therapy.
_x000D_
A High Protein Diet / Albumin Transfusion would be of little to no benefit – and the latter would need to be Salt-Poor.
_x000D_
Angiotensin-Converting Enzyme (ACE) Inhibitors reduce Proteinuria and slow deterioration in Glomerular Filtration Rate (GFR).
_x000D_
In this case, given the patient’s age, he may well have Minimal Change Glomerulonephritis (GN) (commonest in children), which is normally Steroid Responsive.
_x000D_
In general, Steroids are tried first and then second line agents such as Cyclosporine and Cyclophosphamide are introduced if needed.
-
Question 51 of 121
51. Question
A 31-year-old gentleman, who is long-term smoker, presents with a two-week history of cough, fever and haemoptysis.
_x000D_
Chest X-Ray demonstrates Diffuse Alveolar Infiltrates. Urine Dipstick demonstrates Red Cell Casts.
_x000D_
Full Blood Count (FBC) Shows:
_x000D_
_x000D_ _x000D_
_x000D_ _x000D_ Haemoglobin
_x000D_
_x000D_
_x000D_ 108 g/L
_x000D_
_x000D_
_x000D_ (130-180)
_x000D_
_x000D_
_x000D_
_x000D_ _x000D_ White Blood Cell Count
_x000D_
_x000D_
_x000D_ 5.1 ×109/L
_x000D_
_x000D_
_x000D_ (4-11)
_x000D_
_x000D_
_x000D_
_x000D_ _x000D_ Platelet Count
_x000D_
_x000D_
_x000D_ 376 ×109/L
_x000D_
_x000D_
_x000D_ (150-400)
_x000D_
_x000D_
_x000D_
_x000D_
_x000D_
Anti-Neutrophil Cytoplasmic Antibodies (ANCA) – Positive At Titre 1 In 3600.
_x000D_
Which of the following is the most likely diagnosis?
CorrectIncorrectHint
The combination of Haemoptysis (with radiological findings consistent with Pulmonary Haemorrhage) and Red Cell Casts in the Urine (indicating Glomerular Bleeding) should lead one to consider a diagnosis of Goodpasture’s Syndrome in this case.
_x000D_
Goodpasture’s Syndrome is an important, and potentially rapidly fatal, cause of Alveolar Haemorrhage and Rapidly Progressive Renal Failure. It is caused by Circulating Antiglomerular Basement Membrane Antibodies, and typically causes an Acute Glomerulonephritis. It usually presents in young men in their twenties and men and women in their sixties. Despite treatment the mortality of Goodpasture’s Syndrome is 11% and it has a high morbidity with 60% of patients becoming dependent on dialysis. Renal Impairment is caused by a Crescentic Glomerulonephritis.
_x000D_
It frequently has an eruptive presentation in the young, with:
_x000D_
- _x000D_
- Cough
- Fever
- Haemoptysis
- Haematuria
- Proteinuria, and
- Red Cell Casts.
_x000D_
_x000D_
_x000D_
_x000D_
_x000D_
_x000D_
_x000D_
Pulmonary Haemorrhage and Glomerular Bleeding can result in a drop in Haemoglobin.
_x000D_
The most common antibody associated with Goodpasture’s Syndrome is Antiglomerular Basement Membrane Antibodies (Anti-GBM). Antiglomerular Basement Membrane Antibodies (Anti-GBM) are directed against the Goodpasture’s Antigen, which is part of the Non-Collagenous Domain of the Alpha-3(4) Collagen Chain. However, recently it has been shown that in a significant number of patients with Goodpasture’s Syndrome Antineutrophil Cytoplasmic Antibodies (ANCA) can coexist with Antiglomerular Basement Membrane Antibodies (Anti-GBM) – in one study, 30% of patients had Positive Anti-Neutrophil Cytoplasmic Antibodies (ANCA) Serology. In this setting, Anti-Neutrophil Cytoplasmic Antibodies (ANCA) is usually Specific for p-ANCA and is directed against Myeloperoxidase.
_x000D_
In general, both antibodies can be detected at presentation. This seropositivity has been shown to have important clinical and prognostic implications, and these patients may develop Extra-Renal and Extra-Pulmonary Manifestations. In addition, they are more likely to have recurrent renal or pulmonary disease. Prognosis is debated with some studies saying it is more favourable in patients with Positive Anti-Neutrophil Cytoplasmic Antibodies (ANCA) and others showing a worse outcome in these patients.
_x000D_
Plasmapheresis and Immunosuppression (typically with Cyclophosphamide and Corticosteroid) are the treatments of choice in Goodpasture’s Syndrome. This has been shown to reduce Anti-GBM Antibodies most rapidly, which results in improved morbidity and mortality. Plasmapheresis is typically given daily or on alternate days for two to three weeks. Response is assessed by monitoring symptoms and Anti-GBM Antibody Titres. Cyclophosphamide and Prednisolone then continue, typically for six to nine months following remission.
_x000D_
Alport’s Syndrome is a familial Nephritis which presents with Haematuria, Progressive Renal Failure, Ocular Abnormalities and Sensorineural Deafness. It is caused by a mutation in the Type IV Collagen Genes. Inheritance is variable, but the majority are X-Linked Dominant (85%; 15% are Autosomal Recessive). There is a high spontaneous mutation rate, which means 20% of patients have no family history.
_x000D_
-
Question 52 of 121
52. Question
A 31-year-old lady is assessed at the antenatal clinic; she had no known medical disease prior to this pregnancy.
_x000D_
Which of the following scenarios is most compatible with Chronic Hypertension in pregnancy?
CorrectIncorrectHint
Chronic Hypertension in pregnancy is defined as Blood Pressure of at least 140 mmHg Systolic or 90 mmHg Diastolic before pregnancy, or for women who first present for care during pregnancy as in this case, before 20 weeks of gestation.
_x000D_
Proteinuria at the Third Trimester, if not present initially, favours the diagnosis of pre-eclampsia, instead of End-Organ Damage Of Chronic Hypertension.
_x000D_
Women with Chronic Hypertension have an increased chance of pre-eclampsia and therefore pre-eclampsia per se does not exclude Concurrent Chronic Hypertension. In fact, the condition of pre-eclampsia tends to develop at less than 34 weeks of gestation with Chronic Hypertension, earlier than is typical in women without Antecedent Hypertension.
_x000D_
Most women with Chronic Hypertension have a decrease in Blood Pressure during pregnancy. Their Blood Pressure falls toward the end of the First Trimester. Hence, Chronic Hypertension would be considered by first documentation of High Blood Pressure before 20 weeks of gestation.
-
Question 53 of 121
53. Question
A 31-year-old lady with IgA Nephropathy (Berger’s Disease) attended the Clinic shortly after having a Positive Pregnancy Test.
_x000D_
On physical examination, Pulse Rate was 60/Minute and Blood Pressure was 140/80 mmHg. Fundi and Cardiac Examinations were normal. There was no Pedal Oedema.
_x000D_
Urine Protein measured 0.7 gm daily. Her Serum Creatinine level was 60 μmol/L. Medications at that time were Lisinopril and Folic Acid.
_x000D_
Which one of the following recommendations is most appropriate?
CorrectIncorrectHint
Both ACE Inhibitor and Angiotensin-Receptor Blocker are contraindicated in pregnancy.
_x000D_
The use of ACE Inhibitor during the second half of the pregnancy (the fourth option) has been well known to be associated with Oligohydramnios (probably resulting from Impaired Fetal Renal Function) and Neonatal Anuria, and Fetal Death.
_x000D_
Although the old teaching might allow the use of Lisinopril during the First Trimester, an Observational Retrospective Cohort Study that included women with exposure to ACE Inhibitors in the First Trimester, as reported in 2006, raised the issue that the drug is associated with increased odds for Cardiovascular Defects and Central Nervous System Defects.
_x000D_
Renin-Angiotensin System Antagonists should be switched to other class of Antihypertensive Drugs (Before Conception, If Possible). Hence, options B, D & E are not appropriate.
_x000D_
Blood Pressure Goal during pregnancy, in general, is less aggressive (the third answer option); a very tight blood pressure control is linked with an increased risk of Fetal Growth Restriction (FGR). Pre-pregnancy doses of antihypertensive medications are not infrequently reduced, particularly in the Second Trimester.
-
Question 54 of 121
54. Question
A 30-year-old gentleman presents with Haematuria, Progressive Renal Impairment and Auditory Problems.
_x000D_
Of note, there is a strong family history of Renal Problems with family members requiring Dialysis. Further workup of this gentleman leads to a diagnosis of Alport’s Syndrome.
_x000D_
What is the characteristic Otological Problem associated with this condition?
CorrectIncorrectHint
Alport’s Syndrome is associated with a Gene Mutation resulting in a disruption in the formation of Type IV Collagen. This is due to impaired adhesion of the Organ Of Corti which contains the Auditory Sensory Cells. This is related to the Structural Integrity of the Basement Membrane, important for the Kidney, Middle Ear and Eye Function. Sensorineural Deafness results as a consequence.
_x000D_
It is not associated with Conductive Deafness.
_x000D_
There is no increased association with Ear Infections such as Otitis Media, Mastoiditis and Perforated Ear Drum. Hence these answers should not be selected.
_x000D_
-
Question 55 of 121
55. Question
A 30-year-old gentleman is referred to the Cardiologist by the Practice Nurse for management of Hypertension. She has seen him four times over the past five months and his Blood Pressure is persistently elevated at around 160/90 mmHg.
_x000D_
Surgeon, a colleague of the Cardiologist has seen him previously for some non-specific right upper quadrant abdominal pain.
_x000D_
On examination of the abdomen, Specialist can feel bilateral enlarged kidneys and a liver edge.
_x000D_
Investigations Show:
_x000D_
_x000D_ _x000D_
_x000D_ _x000D_ Haemoglobin
_x000D_
_x000D_
_x000D_ 127 g/L
_x000D_
_x000D_
_x000D_ (135-180)
_x000D_
_x000D_
_x000D_
_x000D_ _x000D_ White Blood Cell count
_x000D_
_x000D_
_x000D_ 6.6 ×109/L
_x000D_
_x000D_
_x000D_ (4-10)
_x000D_
_x000D_
_x000D_
_x000D_ _x000D_ Platelet Count
_x000D_
_x000D_
_x000D_ 185 ×109/L
_x000D_
_x000D_
_x000D_ (150-400)
_x000D_
_x000D_
_x000D_
_x000D_ _x000D_ Serum Sodium
_x000D_
_x000D_
_x000D_ 141 mmol/L
_x000D_
_x000D_
_x000D_ (134-143)
_x000D_
_x000D_
_x000D_
_x000D_ _x000D_ Serum Potassium
_x000D_
_x000D_
_x000D_ 4.7 mmol/L
_x000D_
_x000D_
_x000D_ (3.5-5)
_x000D_
_x000D_
_x000D_
_x000D_ _x000D_ Serum Creatinine
_x000D_
_x000D_
_x000D_ 184 μmol/L
_x000D_
_x000D_
_x000D_ (60-120)
_x000D_
_x000D_
_x000D_
_x000D_ _x000D_ Plasma Glucose
_x000D_
_x000D_
_x000D_ 4.6 mmol/L
_x000D_
_x000D_
_x000D_ (<6.0)
_x000D_
_x000D_
_x000D_
_x000D_ _x000D_ Urine Dipstick
_x000D_
_x000D_
_x000D_ Blood ++; Protein –
_x000D_
_x000D_
_x000D_
_x000D_ _x000D_
_x000D_
_x000D_
_x000D_
Which one of the following is most closely associated with his underlying condition?
CorrectIncorrectHint
A young man presenting with Renal Failure, Haematuria and Liver and Renal Masses raises the suspicion of Polycystic Kidney Disease (PKD).
_x000D_
Associated Liver Cysts are found in around 80% of individuals with Polycystic Kidney Disease (PKD). Pancreatic Cysts are rarer, and may in some cases be associated with Recurrent Pancreatitis.
_x000D_
Patients are at increased risk of Renal Stones, but the predominant increase is seen in Urate Stones (Uric Acid Stones), rather than other types.
_x000D_
Up to 25% of patients may have some degree of Mitral Valve Prolapse.
_x000D_
MODY 5 is associated with Hepatic and Renal Cysts and Diabetes Mellitus, but that is less likely to be the diagnosis here in the presence of a Normal Glucose.
_x000D_
Polycystic Kidney Disease (PKD) carries an Autosomal Dominant Pattern Of Inheritance but may occur as a De Novo Mutation in 5%.
_x000D_
_x000D_
-
Question 56 of 121
56. Question
A 29-year-old lady presents with generalised oedema of six months duration.
_x000D_
Investigations Reveal: Proteinuria (5.5 g/day), Hypoproteinemia and Hypercholesterolaemia, Serum Urea 10 mmol/L (2.5-7.5) and Serum Creatinine 200 mol/L (60-110). Renal Biopsy confirms Membranous Glomerulonephritis.
_x000D_
What will be the most appropriate management to get remission?
CorrectIncorrectHint
The Twin Aims of treating Membranous Nephropathy are:
_x000D_
- _x000D_
- First to induce a remission of the Nephrotic Syndrome, and
- Second to prevent the development of End-Stage Renal Failure (ESRF).
_x000D_
_x000D_
_x000D_
_x000D_
A Meta-analysis of four randomised controlled studies comparing treatments of Membranous Nephropathy showed that regimes comprising Chlorambucil or Cyclophosphamide, either alone or along with Steroids, were more effective than symptomatic treatment or treatment with Steroids alone in inducing remission of the Nephrotic Syndrome.
_x000D_
A small randomised controlled study of 20 patients with a Persistent Nephrotic Syndrome and Declining Renal Function suggested that Ciclosporin A slowed the rate of Decline Of Renal Function: this requires confirmation in a larger trial.
-
Question 57 of 121
57. Question
A 35-year-old gentleman is referred to Nephrology Clinic with Chronic Renal Dysfunction and is discovered to have Adult Polycystic Kidney Disease (APKD). His Blood Pressure is consistently elevated at 140-150/90 mmHg.
_x000D_
Which of the following Antihypertensives is the most appropriate for the management of this gentleman’s Blood Pressure?
CorrectIncorrectHint
Autosomal Dominant APKD-1 is a relatively common disorder accounting for approximately 8% of cases of End-Stage Renal Disease (ESRD).
_x000D_
The best drugs for this condition are Angiotensin-Converting Enzyme (ACE) Inhibitors (e.g. Captopril, Enalapril, or Lisinopril) or Angiotensin II Receptor Antagonist Blockers (e.g. Telmisartan, Losartan, Irbesartan, or Candesartan).
_x000D_
Calcium Channel Blockers are not encouraged to be used.
_x000D_
However, although widely used and recommended, there is no evidence that the Renin-Angiotensin System is affected in the disease.
-
Question 58 of 121
58. Question
A 34-year-old lady is diagnosed with Goodpasture’s Syndrome.
_x000D_
Which of the following therapies used in conjunction with Plasmapheresis and Corticosteroids would be expected to improve prognosis associated with the condition?
CorrectIncorrectHint
Studies reveal that without treatment mortality is as high as 90% in association with Goodpasture’s Syndrome.
_x000D_
However the prognosis is drastically improved with the removal of antibody through Plasmapheresis, Immunosuppression with Corticosteroids and Cyclophosphamide.
_x000D_
There are some studies revealing the potential of Mycophenolate Mofetil but the evidence is rather anecdotal.
_x000D_
-
Question 59 of 121
59. Question
A 33-year-old lady receives a Cadaveric Renal Transplantation after having had Chronic End-Stage Renal Failure (ESRF). Of note in her medical history is that she has a Neuropathic Bladder for which she performs Intermittent Self-Catheterisation.
_x000D_
Seven months after Renal Transplantation she presents with acute pain in the region of the Transplanted Kidney.
_x000D_
Which one of the following is the most likely reason for the pain?
CorrectIncorrectHint
Acute Urinary Retention would not commonly cause pain overlying a Transplanted Kidney, and as this lady self-catheterises, it would be an unlikely occurrence.
_x000D_
As time between the Transplant and this episode is 7 months, Allograft Rejection cannot be the underlying cause. This is because Chronic Rejection (More Than Three Months Post Transplant) is a painless process, with difficulty to control Hypertension, Proteinuria and Slowly Rising Serum Creatinine.
_x000D_
Accelerated Rejection (One To Five Days Post Transplant) can present with Fever, an Acutely Tender Swollen Graft, and Rapidly Rising Serum Creatinine. Acute Rejection (Five Days To Three Months) is Clinically Silent in the majority, but can present with a Swollen, Tender Kidney. Both of these can be discounted as it has been 7 months since the patient’s transplant.
_x000D_
Renal Infarction can also be discounted. This can be a Surgical Complication of Renal Transplantation, but it presents early with a Calyceal Fistula and Urinary Leak.
_x000D_
Renal Stones (Renal Calculi) could cause acute pain in the region of a Transplanted Kidney, but would be less likely in this patient than infection.
_x000D_
The answer in this case is Acute Pyelonephritis.
_x000D_
This patient is in the Intermediate Stage of the Post-Transplantation Immunosuppression, when the patient is most Immunocompromised (Three To Six Months Post-Transplant).
_x000D_
She is at High Risk of an Acute Episode of Pyelonephritis in the Transplanted Kidney, due to the Immunosuppression, the Neuropathic Bladder and Self-Catheterisation. This would present like an Acute Rejection Episode, with a Tender Swollen Graft, Low-Grade Pyrexia, and Deteriorating Graft Function.
_x000D_
This would be commonly associated with Septicaemia in this patient, and requires Parenteral Antibiotics.
_x000D_
_x000D_
If this lady were to be managed as a Transplant Rejection, with High Dose Intravenous Steroids, the result could be catastrophic
-
Question 60 of 121
60. Question
A 33-year-old gentleman with Type 1 Diabetes undergoes a 24-Hour Urine Collection.
_x000D_
Which of the following Urine Albumin Concentrations signifies Microalbuminuria?
CorrectIncorrectHint
Microalbuminuria is defined as a Urine Albumin Excretion of between 30 – 300 mg per 24 Hours.
_x000D_
A concentration above 300 mg / 24 Hours signifies Albuminuria and a concentration above 3.5 gm / 24 Hours signifies Overt Proteinuria.
_x000D_
Microalbuminuria is not just an indicator of early renal involvement but it also identifies Increased Cardiovascular Risk with an Approximate Twofold Cardiovascular Risk above the already increased risk in the Diabetic Population.
_x000D_
A useful surrogate of the Total Albumin Excretion is the Albumin : Creatinine Ratio. The Urinary Albumin : Creatinine Ratio is measured using the First Morning Urine Sample where practicable.
_x000D_
Microalbuminuria is indicated where there is an Albumin : Creatinine Ratio ≥ 2.5 mg/mmol (Men) or 3.5 mg / mmol (Women).
_x000D_
Proteinuria is indicated by a Ratio of ≥ 30 mg / mmol.
-
Question 61 of 121
61. Question
A 33-year-old gentleman presents at Respiratory Clinic with worsening shortness of breath, wheeze, lethargy and nausea. He has a history of asthma which has been managed by a Pulmonologist with Salbutamol and a twice daily combination of a long-acting Beta-2 Agonist and an Inhaled Steroid Inhaler.
_x000D_
On examination, his Blood Pressure is 150/90 mmHg, his Pulse Rate is 80/Minute and Regular. He has extensive wheeze throughout both lung fields.
_x000D_
Investigations Show:
_x000D_
_x000D_ _x000D_
_x000D_ _x000D_ Haemoglobin
_x000D_
_x000D_
_x000D_ 120 g/L
_x000D_
_x000D_
_x000D_ (135-180)
_x000D_
_x000D_
_x000D_
_x000D_ _x000D_ Total Leucocyte Count
_x000D_
_x000D_
_x000D_ 10.3 ×109/L
_x000D_
_x000D_
_x000D_ (4-10)
_x000D_
_x000D_
_x000D_
_x000D_ _x000D_ Eosinophils 30%
_x000D_
_x000D_
_x000D_ –
_x000D_
_x000D_
_x000D_
_x000D_ _x000D_ Platelet Count
_x000D_
_x000D_
_x000D_ 195 ×109/L
_x000D_
_x000D_
_x000D_ (150-400)
_x000D_
_x000D_
_x000D_
_x000D_ _x000D_ Serum Sodium
_x000D_
_x000D_
_x000D_ 142 mmol/L
_x000D_
_x000D_
_x000D_ (134-143)
_x000D_
_x000D_
_x000D_
_x000D_ _x000D_ Serum Potassium
_x000D_
_x000D_
_x000D_ 4.7 mmol/L
_x000D_
_x000D_
_x000D_ (3.5-5)
_x000D_
_x000D_
_x000D_
_x000D_ _x000D_ Serum Creatinine
_x000D_
_x000D_
_x000D_ 190 μmol/L
_x000D_
_x000D_
_x000D_ (60-120)
_x000D_
_x000D_
_x000D_
_x000D_ _x000D_ Erythrocyte Sedimentation Rate (ESR)
_x000D_
_x000D_
_x000D_ 61
_x000D_
_x000D_
_x000D_ (<10)
_x000D_
_x000D_
_x000D_
_x000D_ _x000D_ Urine Dipstick
_x000D_
_x000D_
_x000D_ Blood++; Protein ++
_x000D_
_x000D_
_x000D_
_x000D_ _x000D_
_x000D_
_x000D_
_x000D_
Which of the following is the most likely diagnosis?
CorrectIncorrectHint
The history of rapidly worsening symptoms of Asthma, accompanied by Hypertension, raised Serum Creatinine and Blood and Proteinuria are features of Churg-Strauss Syndrome.
_x000D_
Churg-Strauss is a Small and Medium Sized Artery Vasculitis. p-Anti-Neutrophil Cytoplasmic Antibodies (pANCA) is positive in 70% of patients.
_x000D_
High Dose Steroids with the addition of Cyclophosphamide or Azathioprine are standard therapy for the condition.
_x000D_
With treatment, the one-year survival rate for Churg-Strauss is 90%; 62% at five years.
-
Question 62 of 121
62. Question
A 33-year-old gentleman had a Blood Pressure of 160/100 mmHg.
_x000D_
Clinical examination was normal.
_x000D_
Which one of the following would suggest Secondary Hypertension?
CorrectIncorrectHint
It is rather young for a 33-year-old to be Hypertensive but the presence of such a degree of Urinary Protein would suggest that the lesion is of renal; origin Polyarteritis Nodosa etc.
_x000D_
The Potassium Concentration is normal and although it does not exclude Conn’s Disease, it is certainly not suggestive.
_x000D_
Left Ventricular Hypertrophy (LVH) would be found with Sustained Hypertension of any aetiology, as would Arteriovenous (AV).
_x000D_
The Creatinine Clearance is normal.
-
Question 63 of 121
63. Question
A 32-year-old lady presented with Hypertension (160/110 mmHg), Elevated Titres of Antibodies to Double-Stranded DNA, and Proteinuria (1 gm per 24 hours).
_x000D_
Renal Biopsy revealed WHO Class II Lupus Nephritis (Mesangial Disease).
_x000D_
What is the most appropriate single treatment for this patient?
CorrectIncorrectHint
The Renal Manifestations of Systemic Lupus Erythematosus (SLE) are highly variable, ranging from Mild Asymptomatic Proteinuria and / or Haematuria to Rapidly Progressive Uraemia. The various presentations are difficult to classify into Clinical Syndromes and Histological Classes. Although Lupus Nephritis affects a third of patients early in the disease it is frequently unrecognised until Nephritic and / or Nephrotic Syndrome with Renal Failure occur.
_x000D_
Histologically, a number of different types of renal disease are recognised in Systemic Lupus Erythematosus (SLE), with Immune-Complex Mediated Glomerular Disease being the most common. The up to date International Society of Nephrology / Renal Pathology Society 2003 Classification divides these into Six Different Patterns:
_x000D_
- _x000D_
- I – Minimal Mesangial.
- II – Mesangial Proliferative.
- III – Focal.
- IV – Diffuse.
- V – Membranous.
- VI – Advanced Sclerosis.
_x000D_
_x000D_
_x000D_
_x000D_
_x000D_
_x000D_
_x000D_
Glomeruli appear normal by Light Microscopy in Minimal Mesangial Lupus Nephritis, but Immunofluorescence demonstrates Mesangial Immune Deposits.
_x000D_
Mesangial Proliferative Nephritis presents clinically as Microscopic Haematuria and / or Proteinuria. Hypertension is uncommon and Nephrotic Syndrome and Renal Impairment are very rarely seen. Biopsy demonstrates Segmental Areas of Increased Mesangial Matrix and Cellularity, with Mesangial Immune Deposits. A few Isolated Subepithelial or Subendothelial Deposits may be visible by Immunofluorescence. The prognosis is good and specific treatment is only indicated if the disease progresses.
_x000D_
_x000D_
Focal Disease is more advanced, but still affects less than 50% of Glomeruli. Haematuria and Proteinuria are almost always seen, and Nephrotic Syndrome, Hypertension and Elevated Creatinine may be present. Biopsy demonstrates Active or Inactive Focal, Segmental or Global Endocapillary Glomerulonephritis or Extracapillary Glomerulonephritis involving less than 50% of Glomeruli, typically with Focal Subendothelial Immune Deposits, with or without Mesangial Alterations. It is further subdivided:
_x000D_
- _x000D_
- A: Active Lesions: Focal Proliferative Lupus Nephritis.
- A/C: Active and Chronic Lesions: Focal Proliferative and Sclerosing Lupus Nephritis.
- C: Chronic Inactive Lesions with Glomerular Scars: Focal Sclerosing Lupus Nephritis.
_x000D_
_x000D_
_x000D_
_x000D_
Prognosis is variable.
_x000D_
Diffuse Glomerulonephritis is the most common and severe form of Lupus Nephritis. Haematuria and Proteinuria are almost always present, and Nephrotic Syndrome, Hypertension and Renal Impairment Common. Biopsies demonstrate Active or Inactive Diffuse, Segmental or Global Endocapillary Glomerulonephritis or Extracapillary Glomerulonephritis involving more than 50% of all Glomeruli, typically with Diffuse Subendothelial Immune Deposits, with or without Mesangial Alterations. This class is divided into Diffuse Segmental (IV-S) when more than 50% of the involved Glomeruli have Segmental Lesions, and Diffuse Global (IV-G) when more than 50% of involved Glomeruli have Global Lesions. Segmental is defined as a Glomerular Lesion that involves less than half of the Glomerular Tuft.
_x000D_
- _x000D_
- IV-S (A): Active Lesions, Diffuse Segmental Proliferative Lupus Nephritis.
- IV-G (A): Active Lesions, Diffuse Global Proliferative.
- IV-S (A/C): Active and Chronic Lesions, Diffuse Segmental Proliferative and Sclerosing Lupus Nephritis.
- IV-S (C): Chronic Inactive Lesions with Scars, Diffuse Segmental Sclerosing Lupus Nephritis.
- IV-G (C): Chronic Inactive Lesions with Scars: Diffuse Global Sclerosing Lupus Nephritis
_x000D_
_x000D_
_x000D_
_x000D_
_x000D_
_x000D_
Immunosuppressive Therapy is required in these cases to prevent Progressive to End-Stage Renal Failure (ESRF).
_x000D_
Patients with Membranous Lupus Nephritis tend to present with Nephrotic Syndrome. Microscopic Haematuria and Hypertension may also be seen. Biopsies show Global or Segmental Subepithelial Immune Deposits or their Morphologic Sequelae, with or without Mesangial Alterations. It may occur in combination with Class III or IV, in which case both are diagnosed. Progression is variable, and Immunosuppression is not always needed.
_x000D_
In Advanced Sclerosis, more than 90% of Glomeruli are globally sclerosed without residual activity.
_x000D_
With regard to the management of Lupus Nephritis, a Biopsy is indicated in those patients with abnormal urinalysis and / or reduced renal function. This can provide a histological classification as well as information regarding activity, chronicity and prognosis. Cyclophosphamide, Mycophenolate Mofetil and Azathioprine reduce mortality in proliferative forms of Lupus Glomerulonephritis.
_x000D_
IgA Nephropathy is a form of Glomerulonephritis characterised by the deposition of IgA in the Glomeruli. It is a very rare lesion in Systemic Lupus Erythematosus (SLE).
_x000D_
AA Amyloidosis is a systemic disorder characterised by extracellular tissue deposition of fibrils that are composed of Amyloid A Protein (an Acute-Phase Protein produced by Hepatocytes). It occurs in the course of Chronic Inflammatory Disease, but an association with Systemic Lupus Erythematosus (SLE) is very unusual.
_x000D_
Focal Segmental Glomerulonephritis is one of the most common glomerular diseases to result in End-Stage Renal Failure (ESRF). It may occur as a primary condition, or in association with a number of vasculitic disorders (rarely SLE).
_x000D_
Minimal Change Nephropathy is classically a diagnosis of childhood. There is Diffuse Loss of Podocyte Foot Processes, which results in Nephrotic Syndrome but not usually the other features described above.
-
Question 64 of 121
64. Question
A 32-year-old gentleman with a history of heavy alcohol intake presents to the Nephrology Clinic with Macroscopic Haematuria.
_x000D_
He reports having an Upper Respiratory Tract Infection (URTI) in the last two days. His Renal Function continues to decline and he is evaluated in the Renal Unit. His Serum C3 is normal.
_x000D_
What is a Biopsy of his Kidneys most likely to show?
CorrectIncorrectHint
The history of Alcohol Excess and Macroscopic Haematuria soon after an Upper Respiratory Tract Infection (URTI) should point the candidate towards IgA Nephropathy. This is further confirmed by the Normal C3 Level.
_x000D_
_x000D_
C4d Staining Positive is incorrect as this refers to the detection of BK Virus by the C4d Stain.
_x000D_
Podocyte Effacement is incorrect as this is diagnostic of Minimal Change Disease.
_x000D_
The Hump-Like Appearance In Subepithelial Space is characteristic of Post-Streptococcal Glomerulonephritis.
_x000D_
The Tram Track Appearance On Light Microscopy represents Membranoproliferative Glomerulonephritis and therefore should not be selected.
_x000D_
_x000D_
-
Question 65 of 121
65. Question
A 32-year-old gentleman who is receiving regular Haemodialysis (HD) is noted to have a Plasma Potassium of 7.0 mmol/L (3.5-4.9) before a Dialysis Session. Usually his Plasma Potassium is less than 5.5 mmol/L.
_x000D_
Which food combination from the dietary history would be most likely to cause the High Plasma Potassium concentration?
CorrectIncorrectHint
In particular, Tomato and Banana have High Potassium Content and patients should be advised to avoid such foods.
-
Question 66 of 121
66. Question
A 40-year-old male complains of feeling unwell with pyrexia of 39°C. He has rigors with aches all over and has flu-like symptoms in general. He has rectal irritation, pain on defaecation and perineal discomfort. He has pain on micturition and passes threads in his urine. Rectal examination reveals a tender, swollen prostate. Which of the following is the likely diagnosis:
CorrectIncorrectHint
This young male patient has the typical features of acute prostatitis. He needs analgesia; urine is cultured and treatment is started with trimethoprim or ciprofloxacin.
_x000D_
If not promptly treated, these patients may develop a prostatic abscess as shown by high temperature with rigors, severe perineal and rectal pain with tenesmus and, on rectal examination, an enlarged, hot, tender fluctuant prostate.
_x000D_
The treatment of a prostatic abscess is periurethral resection and deroofing of the abscess cavity.
-
Question 67 of 121
67. Question
A 40-year-old gentleman presents to the Urology Clinic with Renal Colic and has confirmed Renal Stones on Radiological Imaging.
_x000D_
He is treated with analgesia and sent home with follow-up by the Urology Team. He manages to pass a stone in his urine and this is sent for analysis.
_x000D_
What is the most common composition of Renal Stones in the general population?
CorrectIncorrectHint
Calcium Oxalate Stones are the most frequent, followed by Calcium Phosphate. Together these make up a significant majority of stones.
_x000D_
Cystine Stones are rare and associated with Tubular Defects. This answer should not be selected.
_x000D_
Magnesium Ammonium Phosphate Stones are associated with Urea Splitting Organisms and are not common.
_x000D_
Uric Acid Stones make up about 5 – 10% of Stones and are less common than Calcium Based Stones.
_x000D_
-
Question 68 of 121
68. Question
A 40-year-old gentleman developed Helicobacter pylori related Duodenal Ulcer after Kidney Transplantation.
_x000D_
He did not use Aspirin; other concurrent medication included Cyclosporine, Prednisolone, Azathioprine and Amlodipine. The patient reported no known drug allergy.
_x000D_
What is the most reasonable eradication treatment regimen?
CorrectIncorrectHint
Clarithromycin is relatively contraindicated in Kidney Transplant Recipients because of its interaction with Cyclosporine.
_x000D_
Quadruple Therapy containing a Proton Pump Inhibitor (PPI), Bismuth, Metronidazole, and Tetracycline, has been shown in Meta-Analysis Of Comparative Randomised Controlled Trials to achieve a similar Eradication Rate to Clarithromycin-based Triple Therapy.
_x000D_
Monotherapies and Dual Therapies (the fifth and fourth answer choices) – usually a proton pump inhibitor and one antibiotic – have always had disappointing results in eradication of Helicobacter pylori.
_x000D_
The second and third answer choices are theoretically effective in Helicobacter pylori infection, but Clarithromycin interacts with Cyclosporine. Being an Enzyme Inhibitor, this Macrolide will result in an (undesirable) increase the blood level of Cyclosporine.
_x000D_
Bismuth-Based Quadruple Therapy, as recommended in the Maastricht Consensus Report, is the main option for Second-Line Therapy.
-
Question 69 of 121
69. Question
A 40 year old diabetic and hypertensive female presents with high grade fever with shivering and chills since two days. She has had similar symptoms two months ago when she was diagnosed with UTI. Urinalysis shows 40-50 WBC/hpf. Which of the following statements are false?
CorrectIncorrectHint
Tuberculous cystitis is secondary to renal tuberculosis and not due to haematogenous or lymphogenous spread.
_x000D_
Therefore, changes commence around the ureteric orifices and trigone in the form of pallor of the mucosa and submucosal oedema. Tubercles appear later.
-
Question 70 of 121
70. Question
A 39-year-old lady underwent a renal transplant which never functioned.
_x000D_
A biopsy revealed pathological features consistent with Acute Rejection associated with Anti-HLA Antibodies.
Which type of Immunoglobulin is expected to account for this process?CorrectIncorrectHint
This Acute Rejection is recognised and due to Anti-IgG Antibodies to the Human Leukocyte Antigen (HLA) Incompatible Tissues with Primary Activation Of T Cells.
_x000D_
The acute response is treated with Immunosuppressants.
_x000D_
-
Question 71 of 121
71. Question
A 39-year-old gentleman attends the Nephrology Clinic with weight gain and shortness of breath.
_x000D_
Laboratory results show a Low Serum Albumin, Raised Serum Cholesterol and Urine Dipstick shows 3+ Protein.
_x000D_
What is the minimum value of Protein: Creatinine Ratio that would be classed as ‘Nephrotic Range’ from the answers below?
CorrectIncorrectHint
The Triad of Proteinuria, Hypoalbuminaemia and Oedema typifies the Nephrotic Syndrome.
_x000D_
The minimum threshold for Proteinuria which is defined as ‘Nephrotic’ is 300 mg/mmol. Hence each of the options below this level are incorrect and should not be selected.
-
Question 72 of 121
72. Question
A 38-year-old gentleman is referred with Chronic Renal Dysfunction and is discovered to have Adult Polycystic Kidney Disease (APKD).
_x000D_
Which of the following Proteins is associated with the development of Adult Polycystic Kidney Disease (APKD)?
CorrectIncorrectHint
Autosomal Dominant APKD-1 is a relatively common disorder accounting for approximately 8% of cases of End-Stage Renal Disease (ESRD).
_x000D_
85% of cases are due to the defect in Polycystic Kidney Disease (PKD)-1 Locus on Chromosome 16p13.3.
_x000D_
Polycystic Kidney Disease (PKD)-1 encodes a large protein, Polycystin, which seems to be involved in Cell to Cell-Matrix Interaction.
_x000D_
-
Question 73 of 121
73. Question
A 37-year-old gentleman presents with right loin pain and haematuria.
_x000D_
He comments that he has had four episodes of similar symptoms in the past. On examination, he is afebrile and has mild pallor.
_x000D_
Investigations Show:
_x000D_
_x000D_ _x000D_
_x000D_ _x000D_ Serum Sodium
_x000D_
_x000D_
_x000D_ 142 mmol/L
_x000D_
_x000D_
_x000D_ (137-144)
_x000D_
_x000D_
_x000D_
_x000D_ _x000D_ Serum Potassium
_x000D_
_x000D_
_x000D_ 3.3 mmol/L
_x000D_
_x000D_
_x000D_ (3.5-4.9)
_x000D_
_x000D_
_x000D_
_x000D_ _x000D_ Serum Chloride
_x000D_
_x000D_
_x000D_ 118 mmol/L
_x000D_
_x000D_
_x000D_ (95-107)
_x000D_
_x000D_
_x000D_
_x000D_ _x000D_ Serum Bicarbonate
_x000D_
_x000D_
_x000D_ 15 mmol/L
_x000D_
_x000D_
_x000D_ (20-28)
_x000D_
_x000D_
_x000D_
_x000D_ _x000D_ Serum Calcium
_x000D_
_x000D_
_x000D_ 2.7 mmol/L
_x000D_
_x000D_
_x000D_ (2.2-2.6)
_x000D_
_x000D_
_x000D_
_x000D_ _x000D_ Serum Urea
_x000D_
_x000D_
_x000D_ 20 mmol/L
_x000D_
_x000D_
_x000D_ (2.5-7.5)
_x000D_
_x000D_
_x000D_
_x000D_ _x000D_ Urinalysis
_x000D_
_x000D_
_x000D_ pH 7.7; Protein 1+; Red Blood Cell 1+; White Cell Count 1+
_x000D_
_x000D_
_x000D_
_x000D_
_x000D_
Which one of the following is the most likely diagnosis?
CorrectIncorrectHint
This gentleman has will have Metabolic Acidosis as shown by low bicarbonate levels (HCO3 = 15) with Failure to excrete H+ in the urine (alkaline urine, pH 7.7), pointing to a diagnosis of Renal Tubular Acidosis (RTA) of Distal Tubular type (Type1).
_x000D_
Type 1 Renal Tubular Acidosis (RTA) or dRTA is characterised by a failure of the Alpha-Intercalated Cells of the Distal Tubule to excrete Hydrogen Ions. This results in an inability to acidify the urine, and subsequently Metabolic Acidosis (due to build-up of H+) and Hypokalaemia (as K+ Reabsorption is linked to H+ Excretion).
_x000D_
- _x000D_
- Alkaline Urine increases the risk of Calcium Deposition, and therefore Nephrocalcinosis is a feature of Type 1 Renal Tubular Acidosis (dRTA)._x000D_
- _x000D_
- Subsequent Bone Demineralisation results in Rickets in children and Osteomalacia in adults.
_x000D_
_x000D_
_x000D_
_x000D_
_x000D_
Type 2 Renal Tubular Acidosis (pRTA) , on the other hand, is caused by a failure of Proximal Tubular Cells to resorb Bicarbonate from the Urine, which also results in Metabolic Acidosis.
_x000D_
- _x000D_
- As the Distal Tubule functions normally, the acidosis is less severe than Type 1 Renal Tubular Acidosis, and Urine has a pH of less than 5.3.
- Rather than being a solitary defect, type 2 Renal Tubular Acidosis (pRTA) is usually associated with a more generalised dysfunction of Tubular Cells which manifests as Fanconi Syndrome (Phosphaturia, Glycosuria, Aminoaciduria, Uricosuria, Tubular Proteinuria).
- Phosphate wasting results in Marked Bone Demineralisation.
_x000D_
_x000D_
_x000D_
_x000D_
The presence of a Urine pH of more than 5.3 here, with symptoms suggestive of a Renal Tract Stone, means that Type 1 Renal Tubular Acidosis (RTA Type1) is the most likely diagnosis.
_x000D_
Bartter Syndrome and Conn’s Syndrome are causes of Hypokalaemia and Metabolic Alkalosis, which does not fit with the Electrolyte Values given above.
-
Question 74 of 121
74. Question
A 37-year-old gentleman presents to the Clinic with new onset Renal Failure and a Non-Blanching Rash across his legs.
_x000D_
In addition, he describes a history of Recurrent Sinus Infections and Nose Bleeds (Epistaxis).
_x000D_
Which of the following tests are most likely to be diagnostic in this case?
CorrectIncorrectHint
Rash, Renal failure, Recurrent Sinus infections & Epistaxis indicate Wegener’s Granulomatosis aka Granulomatosis with Polyangiitis and hence c-ANCA is the most likely diagnostic test of those listed.
_x000D_
Granulomatosis with Polyangiitis is a disease of Granulomatous Inflammation involving the Kidney, Upper Respiratory and Sinus Tracts. It is most often associated with the Anti-Cytoplasmic ANCA (c-ANCA) Positivity.
_x000D_
Anti-Nuclear Antibody (ANA) can be positive in a wide range of Connective Tissue Diseases and is not the best diagnostic test listed and so is incorrect.
_x000D_
p-Anti-Neutrophil Cytoplasmic Antibodies [p-ANCA] is associated with Microscopic Polyangiitis and not Granulomatosis with Polyangiitis and so should not be selected.
_x000D_
Rheumatoid Factor (RA Factor) and Serum Electrophoresis for Myeloma are not diagnostic tests in this case and so are incorrect.
-
Question 75 of 121
75. Question
A 36-year-old lady is brought into the Emergency Department with Protracted Seizures. Laboratory studies reveal that her Serum Sodium is 113 mmol/Litre.
_x000D_
The Team of Doctors decide to treat her with ‘Hypertonic Saline’ – stronger than the Normal Physiologic Concentration.
_x000D_
What is deemed to be the Physiologic Concentration of Saline?
CorrectIncorrectHint
There are few indications for Hypertonic Saline. Ongoing Seizures Secondary to Hyponatraemia is one such indication.
_x000D_
There are a number of Hypertonic Saline Solution Concentrations above the Normal Physiological Value of 0.9%. The question asks for the Physiological Saline Concentration and so 0.9% is correct.
_x000D_
Other answers listed are not Physiologic and should not be selected.
_x000D_
Saline at 0.45% concentration is often termed Half Normal Saline.
_x000D_
The remaining options represent Hypertonic Saline Concentrations that can be administered.
-
Question 76 of 121
76. Question
A 36-year-old lady presents to the Infectious Disease Team following a new diagnosis of Human Immunodeficiency Virus (HIV).
_x000D_
Her CD4 Count is 150 cells/mm3 and a Viral Load is 10,000. She is commenced on Anti-Retroviral Therapy.
_x000D_
At a follow-up appointment three weeks later she has routine blood tests, of which her Serum Creatinine is shown below. She is euvolaemic and has not taken any additional medications over the counter.
_x000D_
On presentation her results showed:
_x000D_
_x000D_ _x000D_
_x000D_ _x000D_ Serum Creatinine
_x000D_
_x000D_
_x000D_ 82 µmol/l
_x000D_
_x000D_
_x000D_ (60 – 90)
_x000D_
_x000D_
_x000D_
_x000D_
_x000D_
and three weeks later:
_x000D_
_x000D_ _x000D_
_x000D_ _x000D_ Serum Creatinine
_x000D_
_x000D_
_x000D_ 218 µmol/l
_x000D_
_x000D_
_x000D_ (60 – 90)
_x000D_
_x000D_
_x000D_
_x000D_
_x000D_
Which of the following answers is most likely to be responsible for her Acute Renal Impairment?
CorrectIncorrectHint
This question requires knowledge of the side effects of Anti-Retroviral Medications.
_x000D_
Tenofovir is associated with Acute and Chronic Renal Impairment and of the answers given is the most likely cause of the Acute Renal Impairment.
_x000D_
Non-Compliance With Medications should not be selected as there is no reason why this would specifically affect renal function. This would be assessed with repeat Viral Loads and CD4 Counts to which we do not have access.
_x000D_
Human Immunodeficiency Virus [HIV] associated Nephropathy is incorrect given the speed of deterioration in renal function.
_x000D_
Of the Anti-Retroviral Medications, Tenofovir is associated with Acute Renal Dysfunction more than Efavirenz and Lamivudine.
_x000D_
_x000D_
-
Question 77 of 121
77. Question
A 45 year old chronic smoker presents with reddish coloured urine since one week. He does not complain of any pain while passing urine. No history of fever or travel. Urinalysis shows 40-50 RBC/hpf. Which of the following statements are false about hematuria?
CorrectIncorrectHint
Haematuria, whether macroscopic or microscopic, is always abnormal Haematuria from a malignant cause is usually painless although renal cell carcinoma presents with loin pain.
-
Question 78 of 121
78. Question
A 44-year-old lady presents to Nephrology Clinic. She has End Stage Renal Disease (ESRD) due to Type 1 Diabetes Mellitus, and is awaiting Renal Transplant. She has been using Peritoneal Dialysis for five months.
_x000D_
She is complaining of fatigue and the Nephrologist notes her Haemoglobin to be 95 g/L. The Nephrologist is considering commencing Erythropoietin (EPO) treatment.
_x000D_
What is the most common side effect of Erythropoietin (EPO) when used in patients with Chronic Kidney Disease (CKD)?
CorrectIncorrectHint
Erythropoietin (EPO) Therapy may be considered in patients with Chronic Kidney Disease (CKD) who have Anaemia to increase Haemoglobin Concentration to 110 – 120 g/L.
_x000D_
Serious side-effects of Erythropoietin (EPO), when used in Chronic Kidney Disease (CKD), are:
_x000D_
- _x000D_
- Hypertension – 20% Of Patients Require Increased Antihypertensive Therapy
- Seizures
- Thromboembolic Disease
- Anaphylaxis
- Failure Of Treatment – This may be due to Untreated Iron Deficiency, Marrow Fibrosis, Drug Therapy, Development Of Antibodies Against The Treatment, Testosterone Deficiency In Males, or Poor Compliance.
_x000D_
_x000D_
_x000D_
_x000D_
_x000D_
_x000D_
Blurred Vision and Hepatotoxicity are not recognised side effects.
_x000D_
New or worsening Hypertension is a common side effect of Erythropoietin (EPO) Therapy. 20% of patients will require Increased Antihypertensive Therapy. Erythropoietin (EPO) may even precipitate a Hypertensive Crisis.
_x000D_
Hypokalaemia is not a recognised side effect. Instead, Hyperkalaemia may occur.
_x000D_
Thrombocytopenia is not a recognised side effect. A dose-dependent rise in Platelet Count is common, due to Erythropoietin (EPO) Affinity for Thrombopoietin Receptors.
-
Question 79 of 121
79. Question
A 43-year-old lady who was recently admitted at Hospital with general malaise, has been found to have Deranged Renal Function. The Registrar arranges ‘An Urgent Renal Scan’ to exclude Obstruction of the Kidneys.
_x000D_
Which of the following is most appropriate?
CorrectIncorrectHint
Acute Imaging of the kidneys is intended primarily to exclude Obstructive Uropathy, which would be demonstrated on Ultrasound Imaging. Ultrasound Imaging is a safe, non-invasive means rapidly to exclude a correctable cause of Renal Impairment. It is readily available in most Hospitals and can be performed by a Sonographer or Radiologist.
_x000D_
A Computed Tomography (CT) Scan KUB (Kidneys-Ureters-Bladder) is indicated when the Ultrasound Scan (USS) (and Clinical History) is suggestive of the presence of Renal Calculi, whether or not they are causing obstruction.
_x000D_
Magnetic Resonance Imaging of the Kidneys can help determine the nature of a lesion seen on Ultrasound Imaging. It is expensive, time-consuming, and not available in all locations. Furthermore many patients with Magnetic Materials inserted, for example, Pacemakers, Defibrillators are precluded from entering the Strong Magnetic Field. These factors preclude its use as a First-Line Approach to exclude Acute Obstruction of the Kidney.
_x000D_
Magnetic Resonance Angiography (MRA) is helpful to exclude Renal Artery Stenosis. This will not confirm or refute the presence of post-renal obstruction.
_x000D_
A Plain Abdominal X-Ray will occasionally demonstrate the presence of Radio-Opaque Renal Calculi, but cannot confirm or refute the presence of Post-Renal Obstruction. It should not be done routinely in all patients with Acute Renal Impairment unless there is a strong clinical suspicion of Renal Calculi [in which case most patients will proceed to having a Computed Tomography (CT) Scan KUB (Kidneys-Ureters-Bladder) Examination].
-
Question 80 of 121
80. Question
A 42-year-old lady, undergoing Haemodialysis, developed high fever one hour after initiation of Dialysis.
_x000D_
Her dialysis was performed using a Tunnelled Right Internal Jugular Vein Catheter, which has been in place and functioning for one year. No other Vascular Access has been created.
_x000D_
The current Catheter Exit Site looked clean. Examination showed no Cardiac Murmur. Blood Cultures were drawn from the Catheter and her Peripheral Arm; both grew Methicillin Resistant Staphylococcus aureus (MRSA).
_x000D_
Which one of the following best describes the correct treatment approach?
CorrectIncorrectHint
Catheter Removal is strongly recommended in Staphylococcus aureus bloodstream infection (CLBSI- Catheter Linked Blood Stream Infection) given the high risk of recurrence in these patients.
_x000D_
Flucloxacillin should not be used in Methicillin Resistant Staphylococcus aureus (MRSA) Infection; Vancomycin is the drug of choice.
_x000D_
Daptomycin might be needed if the Minimum Inhibitory Concentration is ≥ 2 μg/ml indicative of Heterogeneous Vancomycin Intermediate S. aureus (hVISA).
_x000D_
Catheter Removal is also recommended in Non-Staphylococcus Aureus Catheter-Related Bloodstream Infection in the following circumstances:
_x000D_
- _x000D_
- Severe Sepsis.
- Haemodynamic Instability.
- Evidence Of Metastatic Infection, Or
- Persistence Of Bacteraemia After 48-72 Hours Of Effective Antibiotics.
_x000D_
_x000D_
_x000D_
_x000D_
_x000D_
_x000D_
Antibiotic Lock Therapy involves instillation of high-dose antibiotics (prepared using Heparin) at the end of each dialysis session into the catheter to maintain high concentrations within the Dialysis Catheter.
_x000D_
The reported success rate to salvage a Tunnelled Catheter using a combination of Systemic Antimicrobials and Antibiotic Lock Therapy is only 40% to 55% with S. aureus (compared with 75% to 84% with Coagulase Negative Staphylococci).
-
Question 81 of 121
81. Question
A 42-year-old lady was referred for Proteinuria (2.6 g daily). Her Serum Creatinine Level was 120 µmol/L. The Referral Letter mentioned a Low Serum Complement C3 Level.
_x000D_
With reference to the latter information, which of the following comments are relevant to her disease?
_x000D_
- _x000D_
- A history of Infective Endocarditis is of relevance.
- A detailed Medical History and Physical Examination to search for Infection Focus.
- Diagnosis of Myeloma should be suspected.
- Hepatitis B and Hepatitis C Serology should be sought.
- Laboratory Testing should include Anti-Nuclear Antibody (ANA) and Anti-Double-Stranded DNA Antibody.
_x000D_
_x000D_
_x000D_
_x000D_
_x000D_
CorrectIncorrectHint
This is a case of Hypocomplementaemia Glomerular Disease. Differential Diagnosis should include:
_x000D_
- _x000D_
- Post Infectious Glomerulonephritis (Classically Infective Endocarditis)
- Lupus Nephritis
- Membranoproliferative Glomerulonephritis
- Mixed Cryoglobulinaemia
_x000D_
_x000D_
_x000D_
_x000D_
_x000D_
Her Proteinuria points to Glomerular Pathology. Glomerular Diseases with Low Complement Levels narrow down the Differential Diagnosis.
_x000D_
Membranoproliferative Glomerulonephritis is classically associated with decreased Serum C3 (and a Normal C4, indicating activation of the alternative pathway of complement). This indicates one form of Chronic Immune Complex Disease (see above for the common examples).
_x000D_
On the other hand, Lupus Nephritis is associated with activation of the classical pathway and often associated with suppression of both C3 and C4. This is considered a correct answer although the question did not mention the C4 Level.
_x000D_
The median age at diagnosis of Multiple Myeloma is 70 years, much older than this case.
_x000D_
Renal Impairment is unrelated to Immune Complex; it occurs mostly as a result of Direct Tubular Damage from Excess Protein Load, Dehydration, Hypercalcaemia, and the use of Nephrotoxic Medications.
_x000D_
-
Question 82 of 121
82. Question
A 42-year-old lady comes to the Clinic complaining of lethargy and joint pains over the past three months and most recently, increasing nausea and anorexia over the past two to three weeks. She also had a photosensitive rash on her cheeks, for which her General Practitioner has prescribed Topical Sun Block.
_x000D_
On examination, she has a Malar Rash. Her Blood Pressure is 160/100 mmHg, and Pulse Rate is Regular at 82/Minute. Otherwise, physical examination is unremarkable.
_x000D_
Investigations Showed:
_x000D_
_x000D_ _x000D_
_x000D_ _x000D_ Haemoglobin
_x000D_
_x000D_
_x000D_ 102 g/L
_x000D_
_x000D_
_x000D_ (115-165)
_x000D_
_x000D_
_x000D_
_x000D_ _x000D_ Total Leucocyte Count
_x000D_
_x000D_
_x000D_ 9.8 ×109/L
_x000D_
_x000D_
_x000D_ (4-11)
_x000D_
_x000D_
_x000D_
_x000D_ _x000D_ Platelet Count
_x000D_
_x000D_
_x000D_ 137 ×109/L
_x000D_
_x000D_
_x000D_ (150-400)
_x000D_
_x000D_
_x000D_
_x000D_ _x000D_ Serum Sodium
_x000D_
_x000D_
_x000D_ 142 mmol/L
_x000D_
_x000D_
_x000D_ (135-146)
_x000D_
_x000D_
_x000D_
_x000D_ _x000D_ Serum Potassium
_x000D_
_x000D_
_x000D_ 5.5 mmol/L
_x000D_
_x000D_
_x000D_ (3.5-5)
_x000D_
_x000D_
_x000D_
_x000D_ _x000D_ Serum Creatinine
_x000D_
_x000D_
_x000D_ 245 μmol/L
_x000D_
_x000D_
_x000D_ (79-118)
_x000D_
_x000D_
_x000D_
_x000D_ _x000D_ Anti-Nuclear Antibody (ANA)
_x000D_
_x000D_
_x000D_ Positive
_x000D_
_x000D_
_x000D_
_x000D_ _x000D_ Urine Dipstick
_x000D_
_x000D_
_x000D_ Blood +; Protein ++
_x000D_
_x000D_
_x000D_
_x000D_
_x000D_
Which of the following is likely to be the most sensitive marker of disease activity?
CorrectIncorrectHint
This patient’s clinical picture is consistent with an Exacerbation Of Systemic Lupus Erythematosus (SLE), which has led to Nephritis and Renal Impairment.
_x000D_
Whilst the first, third and fourth antibodies may be positive in patients with Lupus, the C3 Complement component is consumed during an Acute Attack and is, therefore, the best marker of disease activity.
_x000D_
In this situation, where Significant Renal Impairment is already noted, Steroids and Cyclophosphamide in combination are the treatment of choice.
_x000D_
-
Question 83 of 121
83. Question
A 42-year-old gentleman required high doses of intravenous diuretics after his renal transplant for the purposes of fluid management.
_x000D_
Soon after administration, he developed hearing loss, tinnitus and vertigo.
_x000D_
Which diuretic is most likely to have caused this?
CorrectIncorrectHint
Use of diuretics has variable side effects, predominantly Electrolyte Derangement.
_x000D_
Loop Diuretics such as Furosemide are associated with Ototoxicity.
_x000D_
Thiazide Diuretics such as Bendroflumethiazide do not have this association; neither do Potassium Sparing Diuretics such as Spironolactone and Triamterene.
_x000D_
Acetazolamide, a Carbonic Anhydrase Inhibitor, is not usually associated with Ototoxicity.
-
Question 84 of 121
84. Question
A 41-year-old lady presents with Polyuria and is passing 4.1 litres of urine per day. She was recently prescribed and started on a new medication.
_x000D_
Investigations Show:
_x000D_
_x000D_ _x000D_
_x000D_ _x000D_ Serum Sodium
_x000D_
_x000D_
_x000D_ 140 mmol/L
_x000D_
_x000D_
_x000D_ (137-144)
_x000D_
_x000D_
_x000D_
_x000D_ _x000D_ Plasma Osmolality
_x000D_
_x000D_
_x000D_ 300 mosmol/L
_x000D_
_x000D_
_x000D_ (275-290)
_x000D_
_x000D_
_x000D_
_x000D_ _x000D_ Urine Osmolality
_x000D_
_x000D_
_x000D_ 212 mosmol/L
_x000D_
_x000D_
_x000D_ (350-1000)
_x000D_
_x000D_
_x000D_
_x000D_
_x000D_
Which of the following drugs had been prescribed?
CorrectIncorrectHint
This lady has Eunatraemia, Hypertonicity (High Serum Osmolality), and Inappropriately Diluted Urine which are consistent with Diabetes Insipidus.
_x000D_
Of the drugs listed Lithium would be the most likely to cause a Nephrogenic Diabetes Insipidus.
_x000D_
-
Question 85 of 121
85. Question
A 41-year-old gentleman, who is engaged in Building Construction Works was referred to the Hospital because of severe leg cramps, after working in hot weather for over six hours. He was dehydrated.
_x000D_
Investigations Revealed:
_x000D_
_x000D_ _x000D_
_x000D_ _x000D_ Serum Albumin
_x000D_
_x000D_
_x000D_ 60 g/L
_x000D_
_x000D_
_x000D_ (37 – 49)
_x000D_
_x000D_
_x000D_
_x000D_ _x000D_ Serum Calcium
_x000D_
_x000D_
_x000D_ 3.11 mmol/L
_x000D_
_x000D_
_x000D_ (2.2 – 2.6)
_x000D_
_x000D_
_x000D_
_x000D_ _x000D_ Serum Creatinine
_x000D_
_x000D_
_x000D_ 305 μmol/L
_x000D_
_x000D_
_x000D_ (60 – 110)
_x000D_
_x000D_
_x000D_
_x000D_
_x000D_
What is the most appropriate immediate management?
CorrectIncorrectHint
The patient is suffering from Heat Cramps (Painful Muscular Cramping During Or After Exertion In Hot Environments).
_x000D_
He has developed Haemoconcentration and Elevated Protein (and Albumin), leading to a phenomenon of Pseudohypercalcaemia.
_x000D_
- _x000D_
- In this condition, the Total Serum Calcium is High but the Ionised Calcium should be Normal.
_x000D_
_x000D_
As ground rule, the first step in the diagnostic evaluation of Hypercalcaemia (and more often Hypocalcaemia) should be to make sure that the “Abnormality” in the Total Serum Calcium Levels is not due to Abnormal Albumin Concentrations. In cases of Abnormal Albumin Level, Adjusted Calcium might better reflect the Correct Calcium Level.
_x000D_
Only Ionised (or Free) Calcium is physiologically active and should be checked if in doubt, such as in patients with very Abnormal Albumin Level. Extensive investigation, such as Protein Electrophoresis, should be omitted if Pseudohypercalcaemia is confirmed.
_x000D_
Bisphosphonate should not be started in this gentleman with Acute Kidney Injury (AKI) (and is not indicated because of Pseudohypercalcaemia rather than Genuine Hypercalcaemia).
_x000D_
In 2011 an FDA Announcement contained a new contraindication and updated warning on Kidney Impairment for Zoledronic Acid. The drug is contraindicated in patients with Creatinine Clearance less than 35 mL/min or in patients with evidence of Acute Renal Impairment. This warning follows a previous New England Journal of Medicine correspondence article describing 72 cases (FDA Adverse Event Reporting System) of Renal Failure associated with Zoledronic Acid from 2001 to 2003.
_x000D_
Pseudohypercalcaemia is asymptomatic and should not affect the Electrocardiogram (ECG).
_x000D_
Forced Diuresis is not a consideration in this case given the presence of Dehydration.
-
Question 86 of 121
86. Question
A 48-year-old gentleman, on regular Haemodialysis, complained of weakness and exertional fatigue. On examination, his Blood Pressure was 170/100 mmHg (Pre-Dialysis) and 160/90 mmHg (Post-Dialysis).
_x000D_
Pre-Dialysis Investigations Revealed:
_x000D_
_x000D_ _x000D_
_x000D_ _x000D_ Haemoglobin
_x000D_
_x000D_
_x000D_ 95 g/L
_x000D_
_x000D_
_x000D_ (130-180)
_x000D_
_x000D_
_x000D_
_x000D_ _x000D_ Serum Potassium
_x000D_
_x000D_
_x000D_ 7.1 mmol/L
_x000D_
_x000D_
_x000D_ (3.5-4.9)
_x000D_
_x000D_
_x000D_
_x000D_ _x000D_ Serum Creatinine
_x000D_
_x000D_
_x000D_ 1260 µmol/L
_x000D_
_x000D_
_x000D_ (60-110)
_x000D_
_x000D_
_x000D_
_x000D_ _x000D_ Serum Corrected Calcium
_x000D_
_x000D_
_x000D_ 2.2 mmol/L
_x000D_
_x000D_
_x000D_ (2.2-2.6)
_x000D_
_x000D_
_x000D_
_x000D_
_x000D_
Which intervention is most likely to improve his symptoms?
CorrectIncorrectHint
Patient presented with chief complaints of weakness and exertional fatigue. This question asks how to improve his symptoms. These symptoms can be attributed to Anemia of CKD. Changing the question around, such as how to improve blood pressure or how to improve prognosis, alters the answer.
_x000D_
There are several deficiencies in the management of this patient with End Stage Renal Failure (ESRF). The symptoms described are more in keeping with his anaemia.
_x000D_
A mild Hypocalcaemia and Hyperkalaemia would not give rise to his symptoms. This also applies to his Uncontrolled Hypertension. The combination of high pre- and post-dialysis Blood Pressure, and high pre-dialysis Serum Potassium indicate that this patient is receiving Inadequate Dialysis. Both Procedural Issues (Insufficient Blood Flow Rate, Dialysis Time and Frequency and Needle Size) and Access Issues should be addressed. However, this is unlikely to drastically improve his symptoms and therefore correcting his Anaemia remains the correct option here.
_x000D_
Bearing in mind the above the question specifically asks what intervention would improve his symptoms. As his symptoms are likely caused by anaemia, one would have to elect Erythropoietin (EPO) irrespective of blood pressure, calcium, etc.
_x000D_
The causes of anaemia in haemodialysis patients are numerous. Firstly, there is insufficient production of Erythropoietin (EPO) and, secondly, Iron Deficiency is present in most. The tubing of dialysis equipment causes continued low-level blood loss, there may be chronic inflammation any other vitamin deficiencies (or folic acid) and gastrointestinal absorption of iron is often reduced. In addition, uraemic toxins can inhibit erythropoiesis, and it appears there is shortened red cell survival in chronic renal failure (possibly due to haemolysis).
_x000D_
If left untreated, the Anaemia of Chronic Kidney Disease (CKD) is associated with deterioration in cardiac function, increased risk of stroke, decreased cognition and mental acuity and fatigues. Treating anaemia in these patients markedly improves quality of life, and decreases morbidity and mortality. The Kidney Disease Outcomes Quality Initiative (KDOQI) and European Revised Evidence-Based Practice 2004 Guidelines recommend maintaining a Haemoglobin >110 g/L with a Haematocrit <33%.
_x000D_
Ideally, before starting Erythropoietin (EPO) in renal patients, one should get their Haematinics (Iron, B12, Folate) to ensure they are replete of all. If any are found to be low they should be replaced. However, there is not an option to check and replace these and therefore Erythropoietin (EPO) is the most correct answer here.
-
Question 87 of 121
87. Question
A 48-year-old gentleman is diagnosed with Diabetic Nephropathy.
_x000D_
If this gentleman had Type 1 Diabetes, his chances of progressing to End-Stage Renal Disease (ESRD) would be approximately 50%.
_x000D_
What percentage of Type 2 Diabetics with Diabetic Nephropathy would be expected to progress to End-Stage Renal Disease (ESRD)?
CorrectIncorrectHint
The majority of patients with Diabetic Nephropathy have Type 2 Diabetes, however this is due to higher prevalence of Type 2, rather than higher incidence of Nephropathy (as incidence is in fact higher in Type 1 Diabetes Mellitus).
_x000D_
There are a number of stages in the development of Nephropathy with Glomerular Hyperfiltration being an early feature. Nephropathy itself is signalled by the excretion of trace amounts of Protein in the Urine (Microalbuminuria).
_x000D_
The progression of the disease may be attenuated by Stringent Blood Pressure Control (with an Angiotensin-Converting Enzyme Inhibitor [ACEi]) and Strict Glycaemic Control.
_x000D_
-
Question 88 of 121
88. Question
A 47-year-old lady with Type 1 Diabetes Mellitus (T1DM) has not attended the Diabetic Clinic for five years.
_x000D_
Examination shows no abnormalities.
_x000D_
Investigations Show:
_x000D_
_x000D_ _x000D_
_x000D_ _x000D_ Haemoglobin
_x000D_
_x000D_
_x000D_ 91 g/L
_x000D_
_x000D_
_x000D_ (115-165)
_x000D_
_x000D_
_x000D_
_x000D_ _x000D_ Mean Corpuscular Volume (MCV)
_x000D_
_x000D_
_x000D_ 93 fL
_x000D_
_x000D_
_x000D_ (80-96)
_x000D_
_x000D_
_x000D_
_x000D_ _x000D_ Haematocrit
_x000D_
_x000D_
_x000D_ 27%
_x000D_
_x000D_
_x000D_ –
_x000D_
_x000D_
_x000D_
_x000D_ _x000D_ HbA1c
_x000D_
_x000D_
_x000D_ 88 mmol/mol
_x000D_
_x000D_
_x000D_ (20-42)
_x000D_
_x000D_
_x000D_
_x000D_ _x000D_ 10.5%
_x000D_
_x000D_
_x000D_ (3.8-6.4)
_x000D_
_x000D_
_x000D_
_x000D_
_x000D_
Blood Smear shows Normochromic, Normocytic Anaemia.
_x000D_
Which of the following is the most likely cause?
CorrectIncorrectHint
The most likely cause is progressive renal failure which leads to reduced release of Erythropoietin (EPO) from the kidneys.
_x000D_
-
Question 89 of 121
89. Question
A 47-year-old lady presents with fatigue and lethargy, and has established End Stage Renal Failure (ESRF). She has been on Haemodialysis (HD) for the past two years and receives Dialysis for three hours, three times a week, at a Regional Haemodialysis Centre.
_x000D_
At one of her regular visits for Haemodialysis, her Blood Pressure is 140/90 mmHg. Further Investigations Reveal:
_x000D_
_x000D_ _x000D_
_x000D_ _x000D_ Serum Potassium
_x000D_
_x000D_
_x000D_ 5.4 mmol/L
_x000D_
_x000D_
_x000D_ (3.5-4.9)
_x000D_
_x000D_
_x000D_
_x000D_ _x000D_ Serum Corrected Calcium
_x000D_
_x000D_
_x000D_ 2.14 mmol/L
_x000D_
_x000D_
_x000D_ (2.2-2.6)
_x000D_
_x000D_
_x000D_
_x000D_ _x000D_ Haemoglobin
_x000D_
_x000D_
_x000D_ 88 g/L
_x000D_
_x000D_
_x000D_ (115-165)
_x000D_
_x000D_
_x000D_
_x000D_ _x000D_ Serum Creatinine
_x000D_
_x000D_
_x000D_ 1260 μmol/L
_x000D_
_x000D_
_x000D_ (60-110)
_x000D_
_x000D_
_x000D_
_x000D_
_x000D_
Which of the following is the best option for management of this lady’s symptoms?
CorrectIncorrectHint
This lady’s main complaint is of fatigue. Treating her with Erythropoietin (EPO) would therefore be the most appropriate therapy particularly with her Anaemia. There are Multiple Potential Causes of Fatigue in patients on Renal Dialysis. Treating her Anaemia is a good first step to improving her symptoms.
_x000D_
Her Serum Potassium is a little high, but would be considered acceptable pre-dialysis. Increasing Dialysis Time is unlikely to be acceptable to most patients and Renal Units though this is an option some patients would choose, especially those on ‘home Haemodialysis’. This can certainly improve fatigue due to Uraemia.
_x000D_
As with many patients on Dialysis, this lady has a Low Serum Calcium. This is due to inadequate 1-Hydroxylation of 25-Hydroxy Vitamin D. Parathyroid Hormone (PTH) is usually elevated in an attempt to maintain reasonable Serum Calcium Concentrations. Calcium can also be lost in the dialysate and the use of High-Calcium Dialysis Bags can help reduce this. This Calcium Level is acceptable.
-
Question 90 of 121
90. Question
A 47-year-old gentleman with Chronic Renal Failure presents to Clinic complaining of increasing fatigue and weakness.
_x000D_
He receives three hours of Haemodialysis (HD), thrice weekly. His Blood Pressure is measured at 175/110 mmHg pre-dialysis and 164/95 mmHg post-dialysis.
_x000D_
Investigations Pre-Dialysis Show:
_x000D_
_x000D_ _x000D_
_x000D_ _x000D_ Haemoglobin
_x000D_
_x000D_
_x000D_ 93 g/L
_x000D_
_x000D_
_x000D_ (130-180)
_x000D_
_x000D_
_x000D_
_x000D_ _x000D_ Serum Potassium
_x000D_
_x000D_
_x000D_ 7.1 mmol/L
_x000D_
_x000D_
_x000D_ (3.5-4.9)
_x000D_
_x000D_
_x000D_
_x000D_ _x000D_ Serum Creatinine
_x000D_
_x000D_
_x000D_ 1563 µmol/L
_x000D_
_x000D_
_x000D_ (60-110)
_x000D_
_x000D_
_x000D_
_x000D_ _x000D_ Corrected Serum Calcium
_x000D_
_x000D_
_x000D_ 2.0 mmol/L
_x000D_
_x000D_
_x000D_ (2.2-2.6)
_x000D_
_x000D_
_x000D_
_x000D_
_x000D_
Which of the following options is most appropriate initial management for this patient?
CorrectIncorrectHint
Haemodialysis (HD) is a complex procedure.
_x000D_
This question deals with the Complex Concept Of Dialysis Adequacy.
_x000D_
- _x000D_
- The prime aim of Long-Term Haemodialysis is to remove Nitrogenous Metabolic End-Products, remove Fluid and maintain Electrolyte, Fluid and Acid-Base Equilibrium.
- Although toxin removal is critical, the removal of any particular toxin is a poor measure of haemodialysis adequacy because the removal rate depends on the pre-dialysis serum concentration.
- ‘Clearance’ is therefore used to indicate dialysis adequacy, and most commonly the clearance of urea is used._x000D_
- _x000D_
- Clearance is the ratio of removal rate to blood concentration.
_x000D_
_x000D_
- Removal rate can be measured instantaneously by sampling blood on either side of the dialyser and multiplying the difference by the inflow rate.
- Clearance is the removal rate divided by the inflow concentration._x000D_
- _x000D_
- However, this only provides a measure of dialysis at one point in time.
_x000D_
_x000D_
- The adequacy of an entire haemodialysis session is best measured by the fall in solute concentration from before dialysis to after._x000D_
- _x000D_
- This is calculated using complex equations and is expressed as Kt/V.
- The current recommendation for adequate dialysis for three treatments per week are a Kt/V of 1.2.
- The details are not needed for MRCP but are described in the references below for those who are interested.
_x000D_
_x000D_
_x000D_
_x000D_
_x000D_
_x000D_
_x000D_
_x000D_
_x000D_
_x000D_
_x000D_
A more crude assessment of the adequacy of dialysis can be obtained by noting the magnitude of the decrease in Blood Urea Concentration (the ‘Urea Reduction Ratio’).
_x000D_
In addition to assessing the reduction in Serum Urea, it is standard practice in the United Kingdom to take biochemical and haematological measurements before and after Haemodialysis (HD) Sessions at regular intervals [monthly for In-Hospital Haemodialysis (HD) patients and at least 3 monthly for In-Home Haemodialysis (HD) patients]. Adequate Haemodialysis (HD) is indicated by Pre-Dialysis Serum Bicarbonate Levels of 18-24 mmol/L, Serum Potassium 4.0-6.0 mmol/L, Serum Phosphate 1.1-1.7 mmol/L, and Serum Calcium and Serum Albumin within normal range. It is also recommended that Pre-Dialysis Haemoglobin Concentration should be maintained between 100-120 g/L.
_x000D_
Cardiovascular morbidity and mortality remain very high in Haemodialysis (HD). Control of Blood Pressure is central to this, and improving Adequacy Of Dialysis has proven value in overcoming this. Blood Pressure varies significantly in Haemodialysis (HD) patients depending on the time taken: Pre-Dialysis, Post-Dialysis or Inter-Dialysis. There is a lack of evidence regarding which value correlates best with long-term outcome. Hypertension Pre-Dialysis can be used a surrogate marker for Inadequate Ultrafiltration during Haemodialysis (HD). Available evidence indicates that control of a patient’s Fluid Volume influences Cardiovascular Outcome. Volume and Blood Pressure are linked and it is, therefore, important to Optimise Ultrafiltration and Dry Weight to control Blood Pressure.
_x000D_
All efforts should be taken to ensure Haemodialysis (HD) patients are Euvolaemic and Normotensive, which include counselling on Serum Sodium and Fluid Restriction, Adequate Ultrafiltration and the use of medication. A High Pre-Dialysis or Inter-Dialysis Blood Pressure may be related to excessive Sodium and Water Ingestion during the Inter-Dialysis Period or a High Dialysate Sodium Level, whereas a High Post-Dialysis Blood Pressure may reflect inadequate achievement of Dry Weight. Weight Gain between dialyses of more than 4.8% is associated with increased mortality.
_x000D_
The combination of High Pre-Dialysis and Post-Dialysis Blood Pressure, and High Pre-Dialysis Potassium indicate that this patient is receiving Inadequate Dialysis. Both procedural issues (insufficient Blood Flow Rate, Dialysis Time and Frequency and Needle Size) and Access Issues should be addressed. If these fail to improve the situation a different dialysis modality should be considered, such as more frequent or sustained Haemodialysis (HD). From the available options, increasing the duration of the dialysis session should be done initially.
_x000D_
His Haemoglobin is also below the recommended level for a dialysis patient but one need to measure Haematinics prior rather than jumping in with Erythropoietin (EPO) Treatment. Many Haemodialysis (HD) patients are Iron Deplete, and in these cases, Intravenous Iron is indicated rather than Erythropoietin (EPO) in the first instance. Anti-Hypertensive Medication can be considered if Adequate Ultrafiltration fails to control this patient’s Blood Pressure.
-
Question 91 of 121
91. Question
A 47-year-old gentleman presents at Urology Clinic with loin pain and haematuria and is found to have Polycystic Kidney Disease (PKD) on Ultrasound Scan.
_x000D_
Of note, his father died of a Brain Haemorrhage in his 50s. Genetic Testing reveals that the gentleman has the PKD-1 Gene Mutation.
_x000D_
On which Chromosome is this Gene Mutation found?
CorrectIncorrectHint
The PKD-1 Gene Mutation is found on Chromosome 16p.
_x000D_
The PKD-2 Gene Mutation is found on Chromosome 4 and gives rise to a Milder Phenotype.
_x000D_
The other Chromosomes do not contain the PKD-1 Gene.
-
Question 92 of 121
92. Question
A 46-year-old lecturer came to the Nephrology Clinic; he was found to have Stage 3 Chronic Kidney Disease during a Health Check. He was willing to discuss future Renal Replacement Therapy.
_x000D_
Which of the following is / are considered to be contraindication (s) to Peritoneal Dialysis?
_x000D_
- _x000D_
- Presence Of Colostomy.
- Heparin Allergy.
- Hepatitis B Infection
- History Of Complex Abdominal Surgery With Adhesion.
_x000D_
_x000D_
_x000D_
_x000D_
CorrectIncorrectHint
Peritoneal Dialysis makes use of a Closed System (Peritoneal Cavity) to allow Effective Dialysis.
_x000D_
Colostomies are thought to increase the risk of Peritonitis with Peritoneal Dialysis. The decision to use Peritoneal Dialysis (PD) in patients with a Colostomy or Ileostomy is based on individuals, but they are thought to be relative contraindications and Haemodialysis (HD) is often felt to be more appropriate.
_x000D_
Complex Abdominal Surgery and Resultant Extensive Adhesion damage the Peritoneal Membrane (Peritoneal Fibrosis) and lead to Compartments within the Peritoneum. This is now considered a relative contraindication to Peritoneal Dialysis.
_x000D_
Heparin Anticoagulation, in contrast to Extracorporeal Haemodialysis Therapy, is not needed for Peritoneal Dialysis.
_x000D_
Simple Abdominal Surgery, however, does not preclude Peritoneal Dialysis; examples include Cholecystectomy, Appendectomy (Appendicectomy / Appendisectomy) or Caesarean Section.
-
Question 93 of 121
93. Question
A 45-year-old lady with a recent diagnosis of Systemic Sclerosis is referred to Hospital with a complaint of headaches and blurred vision. She has a medical history of Asthma.
_x000D_
On examination, her Blood Pressure is 220/120 mmHg and there is Bilateral Papilloedema.
_x000D_
Which of the following medications should be prescribed immediately?
CorrectIncorrectHint
Systemic Sclerosis is a Systemic Disorder characterised by skin thickening due to the deposition of collagen in the dermis. Adverse prognostic features are Renal, Cardiac or Pulmonary Involvement.
_x000D_
A major complication is the development of Scleroderma Renal Crisis. This is characterised by the abrupt onset of Severe Hypertension, usually with Retinopathy, together with rapid deterioration of Renal Function and Heart Failure.
_x000D_
In addition, patients may present with headaches, fever and malaise. It develops in 5 – 10% of patients with Diffuse Systemic Sclerosis especially associated with diffuse cutaneous or rapidly progressive forms of Systemic Sclerosis, and patients in whom a High Dose of Corticosteroid has been started.
_x000D_
Renal Crisis is linked with a Positive Antinuclear Antibody (ANA) Speckled Pattern, Anti-RNA Polymerase I and II Antibodies and absence of Anti-Centromere Antibodies.
_x000D_
It usually presents early, within four years of diagnosis. The pathogenic mechanisms leading to Renal Damage are not completely understood but they involve Endothelial Cell Damage and Intimal Thickening of the Renal Arteries, resulting in Hyperplasia of the Juxtaglomerular Apparatus and Increased Renin Release. Renal Biopsy is not necessary for patients presenting with classical features of Renal Crisis.
_x000D_
The clinical presentation is typically with the symptoms of Malignant Hypertension:
_x000D_
- _x000D_
- Hypertensive Retinopathy Associated With Visual Disturbances.
- Heart Failure and Pulmonary Oedema.
_x000D_
_x000D_
_x000D_
_x000D_
_x000D_
Renal Function is impaired and usually rapidly deteriorates. Hypertension is almost always severe with a Diastolic Blood Pressure over 100 mmHg in 90% of patients. There is Hypertensive Retinopathy in about 85% of patients with Exudates and Haemorrhages and if severe, Papilloedema. There may also be Microangiopathic Haemolytic Anaemia, Thrombocytopenia and Raised Renin Levels.
_x000D_
Scleroderma Renal Crisis is a Medical Emergency. Aggressive treatment is required to prevent the occurrence of Irreversible Vascular Injury. First Line Treatment is Gradual Reduction in Blood Pressure (10-15 mmHg Per Day) with an Angiotensin-Converting Enzyme (ACE) Inhibitor until the Diastolic Pressure reaches 85-90 mmHg. This approach leads to a response in 90% of patients by reversing the Angiotensin-II Mediated Vasoconstriction.
_x000D_
An Abrupt Fall in Blood Pressure should be avoided as it can further Diminish Renal Perfusion and increase the risk of Acute Tubular Necrosis. Therefore, Parenteral Antihypertensive Agents (for example, Intravenous Nitroprusside or Intravenous Labetalol) should be avoided.
_x000D_
Calcium Channel Blockers, usually Nifedipine, may be added where there is Inadequate Reduction of Blood Pressure with Angiotensin-Converting Enzyme (ACE) Inhibitors alone. Additional Oral Hypotensive Agents (for example, Labetalol) can be used if required, and if Pulmonary Oedema is present a Nitrate Infusion may be indicated. There is anecdotal evidence that Intravenous Prostacyclin helps the Microvascular Lesion without precipitating Hypotension, and this is used in some United Kingdom Centres.
_x000D_
Deterioration in Renal Function can be rapid, with Gross Pulmonary Oedema; therefore patients with Scleroderma Renal Crisis should be managed in Hospitals with facilities for Dialysis.
_x000D_
Early aggressive treatment with Angiotensin-Converting Enzyme (ACE) Inhibitors has improved prognosis in Renal Crisis, although 40% of patients will require Dialysis and mortality at five years is 30-40%. Median Time to recovery is one year, and typically occurs within three years. Prognosis is worse for males. Patients who need Dialysis for more than two years can be considered for Renal Transplantation. The Recurrence Rate has been estimated to be approximately 20%.
_x000D_
Care should be taken not to confuse Scleroderma Renal Crisis with Malignant Hypertension. Malignant Hypertension is a Clinical Syndrome characterised by Marked Elevation of Blood Pressure, with Widespread Acute Arteriolar Injury. It has a number of different causes and treatment differs depending on the underlying condition. The pathogenesis overlaps, but Idiopathic Malignant Hypertension tends to involve the Smaller Vessels than in Scleroderma Renal Crisis.
-
Question 94 of 121
94. Question
A 45-year-old lady presented to ER with complaints of worsening shortness of breath. She gives history of pain abdomen and fever since three days and decreased urine production since two days. Which of the following statements are false?
CorrectIncorrectHint
All patients with renal failure will not require renal replacement therapy.
_x000D_
The cause of renal failure should be established – prerenal, renal or postrenal. The patient is treated depending on the cause. When all conservative measures helped by a nephrologist have failed and the patient has severe life-threatening hyperkalaemia, some form of dialysis is indicated.
_x000D_
-
Question 95 of 121
95. Question
A 51-year-old gentleman, a chronic smoker, who had been diagnosed with Type 2 Diabetes Mellitus (T2DM) three years ago was hospitalised because of a foot ulcer. Below Knee Amputation was performed because of Necrotising Fasciitis.
_x000D_
Pre-operatively, his Serum Creatinine was 79 µmol/L. After recovery from the Surgery, a repeat Serum Creatinine showed a level of 55 µmol/L.
_x000D_
What is the most likely explanation for the decrease in Serum Creatinine Level?
CorrectIncorrectHint
The use of Serum Creatinine to estimate the Glomerular Filtration Rate (GFR) can be misleading when the patient has Abnormal Muscle Mass (such as Leg Amputation).
_x000D_
We assume an Improvement of Kidney Function when there is a Falling Serum Creatinine Level; this statement holds true only in the absence of a large change in Muscle Mass (or Meat Intake).
_x000D_
The generation of Serum Creatinine is primarily determined by Dietary Intake and Muscle Mass, which probably accounts for the apparent ‘Improvement’ in the Serum Creatinine Level after an Amputation.
_x000D_
Dialysis cannot be chosen as an explanation for the decrease in Serum Creatinine Level in this instance because the question mentioned no clear indication for Dialysis.
-
Question 96 of 121
96. Question
A 50-year-old lady has been an Inpatient for the past 12 days for treatment of Bronchopneumonia. She has developed the onset of chills, fever, and skin rash over the past two days.
_x000D_
Peripheral Blood Film reveals Eosinophilia. On Urinalysis, she has Proteinuria (Urine Protein: 2+). There is no past history of Renal Disease. Her HbA1c is normal.
_x000D_
Which of the following diagnoses would be most strongly suggested by these findings?
CorrectIncorrectHint
The findings are typical of a Drug-Induced Acute Interstitial Nephritis.
_x000D_
Acute Interstitial Nephritis is characterised by Interstitial Inflammation and Oedema. Left untreated this results in Interstitial Fibrosis. A definitive diagnosis is established by Renal Biopsy, although Eosinophiluria and Gallium 67 Scanning are also suggestive.
_x000D_
_x000D_
60-70% of cases of Acute Interstitial Nephritis are induced by exposure to drugs. The mechanism is via a delayed T-Cell Hypersensitivity or Cytotoxic T-Cell Reaction. This is not typically dose-dependent. Multiple medications have been implicated, and the presentation and laboratory findings vary according to the class of drug involved.
_x000D_
Agents which are commonly implicated are:
_x000D_
- _x000D_
- Beta-Lactam Antibiotics (Especially Methicillin)
- Sulphonamides
- Non-Steroidal Anti-Inflammatory Drugs (NSAIDs)
- Diuretics (Thiazides, Furosemide)
- Antivirals (Aciclovir, Foscarnet)
- Allopurinol, and
_x000D_
_x000D_
_x000D_
_x000D_
_x000D_
_x000D_
_x000D_
_x000D_
Classic presenting features include fever, maculopapular rash, and arthralgia. Mild eosinophilia is common, and eosinophiluria is pathognomonic.
_x000D_
Cessation of the causative agent is critical in the treatment of Acute Interstitial Nephritis.
_x000D_
Corticosteroids can have a beneficial effect, especially if initiated early.
_x000D_
In general, the prognosis of drug-induced Acute Interstitial Nephritis is good, and partial or complete recovery of renal function is normally seen.
_x000D_
Post-Streptococcal Glomerulonephritis (GN) appears weeks after the acute infection.
_x000D_
Berger’s Disease (IgA Nephropathy) is characterised by Haematuria and often follows a ‘flu-Like’ Illness.
_x000D_
Serum Sickness is a possibility, but it is much less common than Interstitial Nephritis.
-
Question 97 of 121
97. Question
A 50-year-old lady is referred to the Emergency Department with worsening renal function and two episodes of haemoptysis (a teacup full each time) over the past 24 hours. She has been seen on a number of occasions by the General Practitioner with fevers, arthralgia, and a rash over the past four months. She takes no regular medication.
_x000D_
On examination her Blood Pressure is 150/90, Pulse Rate is 85/Minute and Regular. Livedo Reticularis and a purpuric rash affecting both legs are noted during examination. There is wheeze on auscultation of the chest, O2 saturation on Room Air is 92%.
_x000D_
Investigations Show:
_x000D_
_x000D_ _x000D_
_x000D_ _x000D_ Haemoglobin
_x000D_
_x000D_
_x000D_ 106g/l
_x000D_
_x000D_
_x000D_ 115-160
_x000D_
_x000D_
_x000D_
_x000D_ _x000D_ White Blood Cell Count
_x000D_
_x000D_
_x000D_ 11.1×109/l
_x000D_
_x000D_
_x000D_ 6-10
_x000D_
_x000D_
_x000D_
_x000D_ _x000D_ Platelet Count
_x000D_
_x000D_
_x000D_ 143 x 109/l
_x000D_
_x000D_
_x000D_ 150-400
_x000D_
_x000D_
_x000D_
_x000D_ _x000D_ Serum Sodium
_x000D_
_x000D_
_x000D_ 142 mmol/l
_x000D_
_x000D_
_x000D_ 135-145
_x000D_
_x000D_
_x000D_
_x000D_ _x000D_ Serum Potassium
_x000D_
_x000D_
_x000D_ 5.2 mmol/l
_x000D_
_x000D_
_x000D_ 3.5-5.5
_x000D_
_x000D_
_x000D_
_x000D_ _x000D_ Serum Creatinine
_x000D_
_x000D_
_x000D_ 210 µmol/l(112 µmol/l4m prior)
_x000D_
_x000D_
_x000D_ 50-90
_x000D_
_x000D_
_x000D_
_x000D_ _x000D_ Erythrocyte Sedimentation Rate (ESR)
_x000D_
_x000D_
_x000D_ 71 mm/hr
_x000D_
_x000D_
_x000D_ <10
_x000D_
_x000D_
_x000D_
_x000D_ _x000D_ C-Reactive Protein (CRP)
_x000D_
_x000D_
_x000D_ 110 mg/dl
_x000D_
_x000D_
_x000D_ <10
_x000D_
_x000D_
_x000D_
_x000D_ _x000D_ Antinuclear Antibody (ANA)
_x000D_
_x000D_
_x000D_ Positive
_x000D_
_x000D_
_x000D_
_x000D_
_x000D_ _x000D_ MPO Anti-Neutrophil Cytoplasmic Antibodies (ANCA)
_x000D_
_x000D_
_x000D_ Positive
_x000D_
_x000D_
_x000D_
_x000D_
_x000D_
_x000D_
Which of the following is the most likely diagnosis?
CorrectIncorrectHint
The answer is Microscopic Polyangiitis. The picture of Worsening Renal Function, Coupled With Haemoptysis, Chest Signs and a Purpuric Rash is consistent with Microscopic Polyangiitis.
_x000D_
_x000D_
_x000D_
MPO ANCA Positivity further supports the diagnosis. PR3 ANCA is more consistent with a Granulomatous Polyangiitis such as Granulomatosis with Polyangiitis (which classically presents with Sinusitis / Nasal Congestion). Biopsy is important in establishing the diagnosis, with the skin lesions an obvious initial source of tissue.
_x000D_
Anti-GBM Antibodies are associated with Goodpasture’s Syndrome, in which more Rapid Renal Dysfunction and more Marked Pulmonary Haemorrhage would be more Classical. Mixed Connective Tissue Disease combines features of Systemic Sclerosis, Myositis, Systemic Lupus Erythematosus (SLE) and Rheumatoid Arthritis (RA), and therefore the symptoms described here would not be classical.
-
Question 98 of 121
98. Question
A 50-year-old gentleman had Recurrent Nephrolithiasis.
_x000D_
Renal Function Tests and Serum Calcium measurements were normal.
_x000D_
A 24-Hour Urine Collection revealed:
_x000D_
_x000D_ _x000D_
_x000D_ _x000D_ Volume
_x000D_
_x000D_
_x000D_ 3.2 Litres
_x000D_
_x000D_
_x000D_
_x000D_ _x000D_ Calcium
_x000D_
_x000D_
_x000D_ 14 mmol/24 hours
_x000D_
_x000D_
_x000D_ (2.5-7.5)
_x000D_
_x000D_
_x000D_
_x000D_ _x000D_ Oxalate
_x000D_
_x000D_
_x000D_ 205 mmol/24 hours
_x000D_
_x000D_
_x000D_ (90-450)
_x000D_
_x000D_
_x000D_
_x000D_ _x000D_ Uric Acid
_x000D_
_x000D_
_x000D_ 3.5 mmol/24 hours
_x000D_
_x000D_
_x000D_ (1.48-4.45)
_x000D_
_x000D_
_x000D_
_x000D_ _x000D_ Citrate
_x000D_
_x000D_
_x000D_ 2.2 mmol/24 hours
_x000D_
_x000D_
_x000D_ (0.3-3.4)
_x000D_
_x000D_
_x000D_
_x000D_
_x000D_
What therapy should be tried first to reduce Stone Formation?
CorrectIncorrectHint
It is likely this gentleman has Absorptive Hypercalciuria, which predisposes to Stone Formation.
_x000D_
A combination of treatments is usually required to reduce the chance of Stone Formation, including Dietary Calcium Restriction and Pharmacological Management.
_x000D_
Both Thiazide Diuretics and Potassium Citrate can be used to reduce Urinary Excretion of Calcium. Potassium Citrate is generally preferred as it has fewer side effects, and is, therefore, better tolerated.
_x000D_
Dietary Calcium Restriction alone has minimal effect on Calciuria, given the large amount of Calcium that can be mobilised from Bone.
_x000D_
Allopurinol is indicated if there is Hyperuricosuria.
_x000D_
Penicillamine is used in the management of Hypercalciuria associated with Wilson’s Disease.
-
Question 99 of 121
99. Question
A 50-year-old gentleman presents to the Cardiology Clinic with a longstanding history of Hypertension.
_x000D_
Investigations show a Serum Urea of 10.2 mmol/L (2.5-7.5) and a Serum Creatinine of 150 µmol/L (60-110).
_x000D_
Which one of the following would suggest a diagnosis of Acute Glomerulonephritis?
CorrectIncorrectHint
Casts containing Erythrocytes (Red Cell Casts) are an indication of Renal Bleeding and are typically found when there is Acute Glomerular Inflammation caused by Glomerulonephritis or Vasculitis.
_x000D_
The other answer options are non-specific and do not suggest an Acute Glomerulonephritis.
-
Question 100 of 121
100. Question
A 50-year-old gentleman was referred to a Higher Centre by his Primary Care Physician for IVU for renal stones. Which of the following statements are true?
CorrectIncorrectHint
Uric acid calculi are radiolucent and hence not visible on a plain X-ray. IVU has the advantage over US in that an IVU shows renal function, which US does not.
_x000D_
- _x000D_
- However, IVU can be dangerous as, in a minority of patients, the intravenous contrast can cause severe hypersensitivity reactions.
_x000D_
-
Question 101 of 121
101. Question
A 50-year-old gentleman was seen for an Insurance Medical Examination.
_x000D_
He was entirely asymptomatic, but his Serum Urate Concentration was noted to be 0.5 mmol/L (0.23-0.46).
_x000D_
What is the most appropriate management for this gentleman?
CorrectIncorrectHint
Uric Acid is the major product of the Catabolism Of Purines, Adenosine Monophosphate (AMP) and Guanosine Monophosphate (GMP) from Nucleic Acids.
_x000D_
Guanosine Monophosphate (GMP) and Adenosine Monophosphate (AMP) are catabolised by a series of enzymes which remove a Single Phosphate Group and the Ribose Sugar, eventually forming Xanthine. Xanthine is then converted to Uric Acid by the Enzyme Xanthine Oxidase. Uric Acid can then be excreted by the Kidneys.
_x000D_
When Serum Uric Acid Levels are high, there is a risk of developing Gout, which causes Monoarticular Inflammation, Gouty Arthropathy and Tophaceous Deformity. Xanthine Oxidase Inhibitors are used in the treatment of Gout, to reduce the amount of Uric Acid formed.
_x000D_
The patient above requires lifestyle advice about reducing intake of substances containing High Purines (including some Alcohol and Red Meat). Should he become symptomatic, treatment would be advisable. In Acute Gout, treatment is with Non-Steroidal Anti-Inflammatory Drugs (NSAIDs) or other Anti-Inflammatory Drugs such as Colchicine.
_x000D_
A Xanthine Oxidase Inhibitor such as Allopurinol should be started after the acute attack to reduce the risk of recurrence.
_x000D_
Causes Of Hyperuricaemia Include:
_x000D_
Increased Formation Of Uric Acid:
_x000D_
Primary:
_x000D_
- _x000D_
- Idiopathic and Inherited Metabolic Disease.
_x000D_
_x000D_
Secondary:
_x000D_
- _x000D_
- Alcohol.
- Psoriasis.
- Increased Nucleic Acid Turnover: Leukaemia and Myeloma.
- Tissue Hypoxia.
- Excess Dietary Purine Intake.
_x000D_
_x000D_
_x000D_
_x000D_
_x000D_
_x000D_
Decreased Excretion Of Uric Acid:
_x000D_
Primary:
_x000D_
- _x000D_
- Idiopathic.
_x000D_
_x000D_
Secondary:
_x000D_
- _x000D_
- Acute Or Chronic Renal Disease.
- Organic Acidosis: Lactate, Ketones.
- Lead Poisoning.
- Drugs: Aspirin And Thiazides.
- Down’s Syndrome.
_x000D_
_x000D_
_x000D_
_x000D_
_x000D_
-
Question 102 of 121
102. Question
A 49-year-old gentleman who underwent a renal transplant two years previously is well established on Ciclosporin. He currently takes the Capimune Formulation.
_x000D_
He comes to see Nephrologist as there has been a manufacturing problem of Capimune and he cannot get Capimune until it is back in stock in three days time.
_x000D_
What is best course of action that can be taken by Nephrologist?
CorrectIncorrectHint
Different formulations of Ciclosporin have different Pharmacokinetic Properties and it is essential that Ciclosporin is prescribed by brand and not generically.
_x000D_
When switching from one formulation to another, the patient must be very closely monitored.
_x000D_
It would be sensible to obtain advice from the Local Renal Unit before switching therapy.
-
Question 103 of 121
103. Question
A 55-year-old gentleman presents to the Cardiology Clinic having problems controlling his Blood Pressure, despite taking three oral anti-hypertensive agents, Ramipril 10 mg, Indapamide 2.5 mg, and Amlodipine 5 mg.
_x000D_
Over the past few weeks, he has been monitoring his Blood Pressure at home and it is rarely below 160/90 mmHg.
_x000D_
On examination in the Clinic, his Blood Pressure is 160/95 mmHg, his Pulse Rate is 88/Minute and Regular. He has a Left Carotid Bruit. Respiratory and Abdominal examinations are unremarkable.
_x000D_
Investigations Show:
_x000D_
_x000D_ _x000D_
_x000D_ _x000D_ Haemoglobin
_x000D_
_x000D_
_x000D_ 122 g/L
_x000D_
_x000D_
_x000D_ (135-177)
_x000D_
_x000D_
_x000D_
_x000D_ _x000D_ Total Leucocyte Count
_x000D_
_x000D_
_x000D_ 6.5 ×109/L
_x000D_
_x000D_
_x000D_ (4-11)
_x000D_
_x000D_
_x000D_
_x000D_ _x000D_ Platelet Count
_x000D_
_x000D_
_x000D_ 204 ×109/L
_x000D_
_x000D_
_x000D_ (150-400)
_x000D_
_x000D_
_x000D_
_x000D_ _x000D_ Serum Sodium
_x000D_
_x000D_
_x000D_ 141 mmol/L
_x000D_
_x000D_
_x000D_ (135-146)
_x000D_
_x000D_
_x000D_
_x000D_ _x000D_ Serum Potassium
_x000D_
_x000D_
_x000D_ 4.9 mmol/L
_x000D_
_x000D_
_x000D_ (3.5-5)
_x000D_
_x000D_
_x000D_
_x000D_ _x000D_ Serum Creatinine
_x000D_
_x000D_
_x000D_ 189 μmol/L
_x000D_
_x000D_
_x000D_ (79-118)
_x000D_
_x000D_
_x000D_
_x000D_
_x000D_
Renal Ultrasound Scan showed significant size discrepancy, with the Left Kidney 2.0 cm smaller than the Right Kidney.
_x000D_
Which of the following is the most appropriate investigation to confirm the diagnosis?
CorrectIncorrectHint
The presence of difficulty to treat Hypertension, Renal Impairment, Evidence Of Other Atherosclerotic Disease (Carotid Bruit) and Discrepant Renal Size makes Renovascular Disease a distinct possibility here.
_x000D_
Renovascular Disease is the term given to disease of the Arterial Supply of the Kidney(s) which results in Renal Hypoperfusion. This leads to Hyperactivation of the Renin-Angiotensin-Aldosterone Axis, which results in Hypertension and Chronic Kidney Disease (CKD).
_x000D_
The choice of best test for diagnosis of Renovascular Disease remains controversial. In practice, individual centres may have their own protocol for Screening Tests. However, for the purpose of the MRCP Examination the following points should be followed:
_x000D_
MR Angiography (MRA) should be considered the Optimum Non-Invasive Screening Test for Renovascular Disease, and can be performed safely in patients with Chronic Kidney Disease (CKD) Stage 3 and Stage 4. However, there is increasing concern regarding Gadolinium-Related Nephrogenic Systemic Fibrosis and Guidelines may, therefore, change in the future.
_x000D_
Computed Tomography (CT) Angiography is commonly used but can be complicated by Radio-Contrast Nephropathy in patients with Chronic Kidney Disease (CKD).
_x000D_
Duplex Ultrasound Combines measurement of Proximal Renal Artery Blood Flow Velocity with Intrarenal Restrictive Index. It is an accurate test for detection of Renovascular Disease, and assessment of severity, but is time-consuming and high operator dependent.
_x000D_
Intra-Arterial Angiography is now reserved for patients with complex anatomy, and when confirming severity prior to revascularisation.
_x000D_
An Intravenous Urogram (IVU) is used to assist with the diagnosis of Urinary Tract Stones, rather than Renovascular Disease.
_x000D_
_x000D_
-
Question 104 of 121
104. Question
A 55-year-old gentleman has been referred to the Outpatient Clinic due to deteriorating renal function.
_x000D_
A diagnosis of Autosomal Dominant Polycystic Kidney Disorder (ADPKD) is made. His family history reveals that his mother died of a stroke at 50 years of age, and that his father is still alive. He is concerned regarding the inheritance of the disorder.
_x000D_
What is the probability that his son will inherit it?
CorrectIncorrectHint
Autosomal Dominant Polycystic Kidney Disorder (ADPKD) is an Autosomal Dominant Condition and typically presents between the ages of 30-50.
_x000D_
Patients develop Deteriorating Renal Function and associated Hypertension.
_x000D_
As well as Renal Cysts, they may also have Hepatic and Berry Aneurysms (maternal history may be highly relevant here).
_x000D_
His chance of passing this condition to his son is 50%.
-
Question 105 of 121
105. Question
A 55-year-old gentleman attends the Cardiology Clinic. He has a history of Myocardial Infarction (MI) and is taking Simvastatin for secondary prevention.
_x000D_
However, his brother recently experienced Rhabdomyolysis and he is now concerned about the risk of Rhabdomyolysis due to his Simvastatin.
_x000D_
Which Statin is associated with the lowest risk of Rhabdomyolysis?
CorrectIncorrectHint
Statins are generally very safe medications but confer a small risk of Rhabdomyolysis. Data are conflicting about the likely incidence of Rhabdomyolysis in Statin users, but certain Statins appear to have a lower incidence than others.
_x000D_
Statins and their associated risk of Rhabdomyolysis:
_x000D_
_x000D_ _x000D_
_x000D_ _x000D_ Atorvastatin
_x000D_
_x000D_
_x000D_ 0.04%
_x000D_
_x000D_
_x000D_
_x000D_ _x000D_ Cerivastatin
_x000D_
_x000D_
_x000D_ 3.16%
_x000D_
_x000D_
_x000D_
_x000D_ _x000D_ Fluvastatin
_x000D_
_x000D_
_x000D_ <0.04%
_x000D_
_x000D_
_x000D_
_x000D_ _x000D_ Pravastatin
_x000D_
_x000D_
_x000D_ 0.04%
_x000D_
_x000D_
_x000D_
_x000D_ _x000D_ Lovastatin
_x000D_
_x000D_
_x000D_ 0.19%
_x000D_
_x000D_
_x000D_
_x000D_ _x000D_ Simvastatin
_x000D_
_x000D_
_x000D_ 0.12%
_x000D_
_x000D_
_x000D_
_x000D_
_x000D_
The Lipophilic Statins, Simvastatin, Lovastatin and Cerivastatin are all associated with a higher incidence of Rhabdomyolysis compared to Hydrophilic Statins.
_x000D_
It is possible that the Lipophilic Statins have a greater ability to cross the Myocyte Cell Membrane and cause direct effects on intracellular organelles.
_x000D_
Cerivastatin has been withdrawn from the market as the risk of Rhabdomyolysis was considered unacceptably high.
_x000D_
_x000D_
_x000D_
Statins: Benefits & Adverse effects
_x000D_
_x000D_
_x000D_
-
Question 106 of 121
106. Question
A 54-year-old lady with a history of Bipolar Disorder presents following an overdose of her mood stabilising medications; she cannot remember the name of the medications.
_x000D_
On examination, she is Drowsy and Hyperreflexic and Blood Investigations show a Worsening Hypokalaemic Metabolic Acidosis.
_x000D_
The Nephrology Team has been informed and they are arranging for Urgent Dialysis.
_x000D_
Which is the most appropriate immediate management while waiting for Dialysis?
CorrectIncorrectHint
IV Sodium Bicarbonate is the most suitable option here to correct the Worsening Acidosis, although admittedly, Dialysis is the Optimum Treatment in what sounds like Severe Lithium Toxicity.
_x000D_
IV Furosemide will not improve symptoms of Lithium Toxicity and there is no mention of Hypoglycaemia in this patient ruling out the need for IV Dextrose.
_x000D_
IV Albumin use is predominantly for Rapid Volume Expansion and the patient above does not have any need for this as such.
_x000D_
Oral Sodium Bicarbonate would not be of use in a patient with Rapidly Worsening Acidosis as the time taken for absorption and availability in the blood would be too long.
-
Question 107 of 121
107. Question
A 54-year-old gentleman has a history of Hypertension, managed with Amlodipine and Indapamide. His Primary Care Physician recently tried to commence Ramipril, but had to curtail this as his Serum Creatinine rose from 129 to 194 after one week of therapy with Ramipril.
_x000D_
He smokes 20 cigarettes per day and is a Vasculopath, having suffered a Transient Ischaemic Attack (TIA) one year earlier.
_x000D_
On examination in the Clinic, his Blood Pressure is 160/90 mmHg, Pulse is regular at 80/Minute and a left carotid bruit is audible.
_x000D_
Investigations Show:
_x000D_
_x000D_ _x000D_
_x000D_ _x000D_ Haemoglobin
_x000D_
_x000D_
_x000D_ 125 g/L
_x000D_
_x000D_
_x000D_ (135-177)
_x000D_
_x000D_
_x000D_
_x000D_ _x000D_ Total Leucocyte Count
_x000D_
_x000D_
_x000D_ 6.5 ×109/L
_x000D_
_x000D_
_x000D_ (4-11)
_x000D_
_x000D_
_x000D_
_x000D_ _x000D_ Platelet Count
_x000D_
_x000D_
_x000D_ 290 ×109/L
_x000D_
_x000D_
_x000D_ (150-400)
_x000D_
_x000D_
_x000D_
_x000D_ _x000D_ Serum Sodium
_x000D_
_x000D_
_x000D_ 141 mmol/L
_x000D_
_x000D_
_x000D_ (135-146)
_x000D_
_x000D_
_x000D_
_x000D_ _x000D_ Serum Potassium
_x000D_
_x000D_
_x000D_ 4.2 mmol/L
_x000D_
_x000D_
_x000D_ (3.5-5)
_x000D_
_x000D_
_x000D_
_x000D_ _x000D_ Serum Creatinine
_x000D_
_x000D_
_x000D_ 124 μmol/L
_x000D_
_x000D_
_x000D_ (79-118)
_x000D_
_x000D_
_x000D_
_x000D_ _x000D_ Ultrasound Scan
_x000D_
_x000D_
_x000D_ Right Kidney Smaller Than The Left.
_x000D_
_x000D_
_x000D_
_x000D_ _x000D_ Renal Magnetic Resonance Angiography (MRA)
_x000D_
_x000D_
_x000D_ Suggestive of 80% Right Renal Artery Stenosis.
_x000D_
_x000D_
_x000D_
_x000D_
_x000D_
Which of the following is the most appropriate next step in his management?
CorrectIncorrectHint
There is considerable debate about the success of Angioplasty / Stenting or Vascular Surgery in patients with Atherosclerotic Renal Artery Stenosis (RAS) and difficult to manage Hypertension. The most recent study by Bax et. al. showed no difference in outcomes between those patients managed with Medical Therapy versus Stenting.
_x000D_
Other studies suggest that complications of Stenting or Angioplasty outweigh any benefits Versus Medical Therapy.
_x000D_
There is no evidence to support re-introducing Angiotensin-Converting Enzyme (ACE) Inhibition or substituting an Angiotensin Receptor Blocker (ARB) with respect to potential outcomes. As such, in the second instance, response to Increased Medical Therapy for Blood Pressure should be assessed.
-
Question 108 of 121
108. Question
A 53-year-old gentleman comes to the Outpatient Department complaining of nausea and fatigue. He was previously seen with symptoms of sinusitis, and a saddle nose deformity.
_x000D_
Most recently he has begun to complain of shortness of breath and a chronic cough.
_x000D_
On examination, he is Hypertensive at 160/90 mmHg. There are bilateral inspiratory crackles on auscultation of the chest.
_x000D_
Investigations Show:
_x000D_
_x000D_ _x000D_
_x000D_ _x000D_ Haemoglobin
_x000D_
_x000D_
_x000D_ 120 g/L
_x000D_
_x000D_
_x000D_ (135-180)
_x000D_
_x000D_
_x000D_
_x000D_ _x000D_ White Blood Cell Count
_x000D_
_x000D_
_x000D_ 10.2 ×109/L
_x000D_
_x000D_
_x000D_ (4-10)
_x000D_
_x000D_
_x000D_
_x000D_ _x000D_ Platelet Count
_x000D_
_x000D_
_x000D_ 185 ×109/L
_x000D_
_x000D_
_x000D_ (150-400)
_x000D_
_x000D_
_x000D_
_x000D_ _x000D_ Serum Sodium
_x000D_
_x000D_
_x000D_ 140 mmol/L
_x000D_
_x000D_
_x000D_ (134-143)
_x000D_
_x000D_
_x000D_
_x000D_ _x000D_ Serum Potassium
_x000D_
_x000D_
_x000D_ 5.3 mmol/L
_x000D_
_x000D_
_x000D_ (3.5-5)
_x000D_
_x000D_
_x000D_
_x000D_ _x000D_ Serum Creatinine
_x000D_
_x000D_
_x000D_ 238 μmol/L
_x000D_
_x000D_
_x000D_ (60-120)
_x000D_
_x000D_
_x000D_
_x000D_ _x000D_ Erythrocyte Sedimentation Rate (ESR)
_x000D_
_x000D_
_x000D_ 74
_x000D_
_x000D_
_x000D_ (<10)
_x000D_
_x000D_
_x000D_
_x000D_ _x000D_ Urine Dipstick
_x000D_
_x000D_
_x000D_ Blood++; Protein ++
_x000D_
_x000D_
_x000D_
_x000D_
_x000D_
Which of the following is the most likely diagnosis?
CorrectIncorrectHint
Granulomatosis with Polyangiitis is a multi-system disorder characterised by necrotising granulomas of medium and small blood vessels. Patients have constitutional symptoms such as fever, lethargy and weight loss.
_x000D_
Organ-specific Involvement Includes:
_x000D_
- _x000D_
- Eyes
- Upper Airway
- Lung
- Renal
- Nervous System
- Skin, and
_x000D_
_x000D_
_x000D_
_x000D_
_x000D_
_x000D_
_x000D_
_x000D_
Men are affected slightly more often than women and typically present in the fourth or fifth decade. Patients often present with Ocular or ENT Symptoms for some time before a diagnosis is made.
_x000D_
The Autoimmune Profile is likely to reveal a positive Circulating Anti-Neutrophil Cytoplasmic Antibodies (cANCA), and Renal Biopsy is the usual way to obtain a Tissue Diagnosis.
_x000D_
_x000D_
Treatment is with High Dose Steroids and Cyclophosphamide, which has revolutionised this disease which is otherwise rapidly fatal.
-
Question 109 of 121
109. Question
A 52-year-old gentleman is admitted with Cardiogenic Shock due to an Acute Myocardial Infarction (MI).
_x000D_
His Urine Output drops over the next few days. His Serum Urea increases to 19 mmol/L (2.5-7.5), with Serum Creatinine of 308 µmol/L (60-110). Urinalysis reveals no Protein or Glucose, a Trace Blood, and Numerous Hyaline Casts.
_x000D_
Several days later he develops Polyuria and his Serum Urea and Serum Creatinine fall.
_x000D_
Which of the following pathologic findings is most likely to be seen in his kidneys?
CorrectIncorrectHint
He would have findings of Ischaemic Acute Tubular Necrosis from Cardiogenic Shock.
_x000D_
Fusion Of Podocyte Foot Processes is seen in Minimal Change Glomerulonephritis.
_x000D_
Glomerular Crescents can complicate any Glomerulopathy but, along with Mesangial Immune Complex Deposition, is usually an Immune-Mediated Process.
_x000D_
Hyperplastic Arteriolosclerosis is the ‘onion Skin’ appearance of Arterioles in Malignant Hypertension.
-
Question 110 of 121
110. Question
A 52-year-old gentleman is admitted to the Hospital with a third attack of renal stones in the last five months.
_x000D_
He suffers from Crohn’s Disease and has previously had a limited Small Bowel Resection, but his disease is now quiescent. Apparently, there is a history of high calcium levels in other blood relatives.
_x000D_
On examination, his Blood Pressure is 110/70 mmHg, his BMI is 19.5, he has a midline scar consistent with a previous laparotomy.
_x000D_
Investigations Show:
_x000D_
_x000D_ _x000D_
_x000D_ _x000D_ Haemoglobin
_x000D_
_x000D_
_x000D_ 121 g/L
_x000D_
_x000D_
_x000D_ (115-165)
_x000D_
_x000D_
_x000D_
_x000D_ _x000D_ Total Leucocyte Count
_x000D_
_x000D_
_x000D_ 6.6 ×109/L
_x000D_
_x000D_
_x000D_ (4-11)
_x000D_
_x000D_
_x000D_
_x000D_ _x000D_ Platelet Count
_x000D_
_x000D_
_x000D_ 274 ×109/L
_x000D_
_x000D_
_x000D_ (150-400)
_x000D_
_x000D_
_x000D_
_x000D_ _x000D_ Serum Sodium
_x000D_
_x000D_
_x000D_ 140 mmol/L
_x000D_
_x000D_
_x000D_ (135-146)
_x000D_
_x000D_
_x000D_
_x000D_ _x000D_ Serum Potassium
_x000D_
_x000D_
_x000D_ 4.2 mmol/L
_x000D_
_x000D_
_x000D_ (3.5-5)
_x000D_
_x000D_
_x000D_
_x000D_ _x000D_ Serum Creatinine
_x000D_
_x000D_
_x000D_ 84 µmol/L
_x000D_
_x000D_
_x000D_ (79-118)
_x000D_
_x000D_
_x000D_
_x000D_ _x000D_ Serum Calcium
_x000D_
_x000D_
_x000D_ 2.90 mmol/L
_x000D_
_x000D_
_x000D_ (2.20-2.67)
_x000D_
_x000D_
_x000D_
_x000D_ _x000D_ Parathyroid Hormone (PTH)
_x000D_
_x000D_
_x000D_ Upper limit of normal range.
_x000D_
_x000D_
_x000D_
_x000D_
_x000D_
Which of the following is the most likely diagnosis?
CorrectIncorrectHint
This gentleman has a Raised Serum Calcium with an Inappropriately High Parathyroid Hormone (PTH). The remainder of his bloods are normal, with no evidence of renal failure or malabsorption.
_x000D_
Familial Hypocalciuric Hypercalcaemia is an Autosomal Dominant Condition and is the most likely diagnosis in this case.
_x000D_
The disease most often leads to asymptomatic elevated levels of Serum Calcium, although some patients with the condition may suffer recurrent episodes of renal stones.
_x000D_
The inherited condition is usually caused by a mutation in the Calcium-Sensing Receptor Gene. The perceived lack of Calcium Levels by the Parathyroid leads to Resetting Of Calcium and Parathyroid Hormone (PTH) to Higher Levels. It does not require any treatment.
_x000D_
Primary Hyperparathyroidism is caused by Parathyroid Adenomas or Hyperplasia, which results in Raised Parathyroid Hormone (PTH) and subsequently Raised Plasma Calcium and Urinary Calcium. Alkaline Phosphatase is raised, and Serum Phosphate is reduced.
_x000D_
Secondary Hyperparathyroidism is Compensatory Hypertrophy of All Four Glands due to Hypocalcaemia [due to Chronic Kidney Disease (CKD), or Malabsorption]. Parathyroid Hormone (PTH) is raised and Calcium is low or normal.
_x000D_
Tertiary Hyperparathyroidism develops after a prolonged period of Secondary Hyperparathyroidism. The Parathyroid Glands become Autonomous and Both Parathyroid Hormone (PTH) and Calcium are raised.
_x000D_
PTH-Related Protein is responsible for 80% of Hypercalcaemia in Malignancy and acts on the Same Receptors as Parathyroid Hormone (PTH). It is secreted by Squamous Cell Tumours, Breast and Renal Tumours. Serum Calcium is raised, but Parathyroid Hormone (PTH) will be low.
_x000D_
Plasma Calcium is tightly regulated by Parathyroid Hormone (PTH) and Vitamin D, which act on the Gastrointestinal Tract, Kidney, and Bone. Parathyroid Hormone (PTH) releases Calcium from Bone and inhibits its excretion from the Kidney. Vitamin D promotes calcium absorption from the Gastrointestinal Tract. Plasma Calcium Levels are detected by a Calcium-Sensing Receptor on the Parathyroid Glands.
-
Question 111 of 121
111. Question
A 52-year-old gentleman is admitted to Hospital with a third attack of Renal Stones in the last five months. He suffers from Crohn’s Disease and has previously had a Limited Small Bowel Resection, but his disease is now quiescent.
_x000D_
On examination, his Blood Pressure is 110/70 mmHg, his BMI is 19.5 kg/m2, and he has a Midline Scar consistent with a previous Laparotomy.
_x000D_
Investigations Show:
_x000D_
_x000D_ _x000D_
_x000D_ _x000D_ Haemoglobin
_x000D_
_x000D_
_x000D_ 120 g/L
_x000D_
_x000D_
_x000D_ (135-177)
_x000D_
_x000D_
_x000D_
_x000D_ _x000D_ Total Leucocyte Count
_x000D_
_x000D_
_x000D_ 5.9 ×109/L
_x000D_
_x000D_
_x000D_ (4-11)
_x000D_
_x000D_
_x000D_
_x000D_ _x000D_ Platelet Count
_x000D_
_x000D_
_x000D_ 172 ×109/L
_x000D_
_x000D_
_x000D_ (150-400)
_x000D_
_x000D_
_x000D_
_x000D_ _x000D_ Serum Sodium
_x000D_
_x000D_
_x000D_ 139 mmol/L
_x000D_
_x000D_
_x000D_ (135-146)
_x000D_
_x000D_
_x000D_
_x000D_ _x000D_ Serum Potassium
_x000D_
_x000D_
_x000D_ 3.9 mmol/L
_x000D_
_x000D_
_x000D_ (3.5-5)
_x000D_
_x000D_
_x000D_
_x000D_ _x000D_ Serum Creatinine
_x000D_
_x000D_
_x000D_ 133 μmol/L
_x000D_
_x000D_
_x000D_ (79-118)
_x000D_
_x000D_
_x000D_
_x000D_ _x000D_ 24-Hour Urinary Oxalate Excretion
_x000D_
_x000D_
_x000D_ Increased
_x000D_
_x000D_
_x000D_ –
_x000D_
_x000D_
_x000D_
_x000D_
_x000D_
Which of the following is likely to be the most effective and appropriate intervention?
CorrectIncorrectHint
Several mechanisms have been postulated to explain the development of Hyperoxaluria in patients with Intestinal Disease.
_x000D_
These include Increased Colonic Permeability, Reduced Free Intestinal Calcium available to Bind Oxalate, and decreased Levels of Oxalobactor Formigenes to Degrade Intestinal Oxalate.
_x000D_
Reducing Intake Of Offal is most helpful at Reducing Urate Excretion; foods such as Chocolate, Rhubarb, and Nuts are high in Oxalate. One contributor to this patient’s Increased Oxalate Excretion is undoubtedly his Partial Small Bowel Resection, and Increasing Dietary Calcium Intake decreases Urinary Oxalate Excretion by reducing absorption (as free oxalate is bound).
_x000D_
Therefore, the most effective and appropriate intervention from those given is to increase his oral fluid intake significantly. Increased fluid intake restores fluid lost through the digestive tract, and also acts as a dilutional inhibitor of crystal and stone formation.
_x000D_
Other treatments which can help Enteric Hyperoxaluria include:
_x000D_
- _x000D_
- Calcium, Cholestyramine and Magnesium – Bind strongly to Free Intestinal Oxalate, Preventing Absorption.
- Iron and Aluminium – Act As Intestinal Oxalate-Binding Agents.
- Potassium Citrate – Alkalinises The Urine, Which Reduces Urinary Oxalate Excretion.
_x000D_
_x000D_
_x000D_
-
Question 112 of 121
112. Question
A 51-year-old lady presents to the Clinic with bilateral leg swelling.
_x000D_
Urine Dipstick shows 4+ Protein and Serum Albumin is 15 gm/l (Normal Range: 35-50 gm/l). Renal Function is within normal range.
_x000D_
Further Urinalysis indicates Nephrotic-Range Proteinuria. Further to this, a Renal Biopsy is performed which shows Thickened Glomerular Capillary Loops.
_x000D_
Which of the following may be a cause for this presentation?
CorrectIncorrectHint
Vignette reflects a scenario of Membranous Nephropathy. MN is the second most common cause of Nephrotic Syndrome in adults in the Western Countries, accounting for 22-33% of cases and coming in just behind Focal Segmental Glomerulosclerosis. It is primary (idiopathic) in two-thirds of cases.
_x000D_
Secondary Causes Include:
_x000D_
- _x000D_
- Autoimmune Disease
- Infections
- Drugs (Captopril, Gold, Non-Steroidal Anti-Inflammatory Drugs [NSAIDs]), or
- Malignancy.
_x000D_
_x000D_
_x000D_
_x000D_
_x000D_
Infectious causes of Membranous Nephropathy include:
_x000D_
- _x000D_
- Hepatitis B
- Hepatitis C
- Syphilis, and
- Malaria.
_x000D_
_x000D_
_x000D_
_x000D_
_x000D_
It can also be seen in Human Immunodeficiency Virus (HIV) although this disease is more often associated with a Focal Glomerulosclerosis.
_x000D_
Candida spp. are not causes of Membranous Nephropathy.
_x000D_
Although related to Haemolytic Uraemic Syndrome, Escherichia coli 0157 is not a cause of Membranous Nephropathy.
_x000D_
- _x000D_
- pyloriis related to Peptic Ulcer Disease and is not a cause of Membranous Nephropathy.
_x000D_
_x000D_
Hepatitis B is a secondary cause of Membranous Nephropathy and patients with this syndrome should be tested.
_x000D_
Although a cause of Atypical Pneumonia, Mycoplasma spp. is not a cause of Membranous Nephropathy.
-
Question 113 of 121
113. Question
A 59-year-old gentleman with End-Stage Renal Disease (ESRD) is receiving Haemodialysis and Erythropoietin (EPO).
_x000D_
Which of the following does Erythropoietin (EPO) Therapy cause?
CorrectIncorrectHint
Hypertension is a frequent problem associated with Erythropoietin (EPO) induced Seizures.
_x000D_
A particular symptom is the onset of sudden stabbing migraine-like headache and should raise awareness to the possibility of Hypertensive Crisis.
_x000D_
Other adverse effects of treatment with Erythropoietin (EPO) include:
_x000D_
- _x000D_
- Hyperkalaemia In Uraemic Patients.
- Increased Packed Cell Volume (PCV) (Especially With Misuse By Normal Individuals).
- Shunt Thrombosis.
- Induction Of Iron Deficiency.
- Skin Rashes.
- Urticaria, and
- Flu-like Illness.
_x000D_
_x000D_
_x000D_
_x000D_
_x000D_
_x000D_
_x000D_
_x000D_
-
Question 114 of 121
114. Question
A 59-year-old lady is admitted with sudden onset of chest pain and is diagnosed with an acute Myocardial Infarction (MI).
_x000D_
Her acute illness is complicated by Low Blood Pressure and Poor Tissue Perfusion for several days. Her Serum Lactate becomes elevated. Her Serum Urea and Serum Creatinine are noted to be increasing.
_x000D_
_x000D_ _x000D_
_x000D_ _x000D_ _x000D_
_x000D_
_x000D_ Day 1
_x000D_
_x000D_
_x000D_ Day 2
_x000D_
_x000D_
_x000D_ Day 3
_x000D_
_x000D_
_x000D_ Normal Range
_x000D_
_x000D_
_x000D_
_x000D_ _x000D_ Serum Urea (mmol/L)
_x000D_
_x000D_
_x000D_ 8.2
_x000D_
_x000D_
_x000D_ 23
_x000D_
_x000D_
_x000D_ 33
_x000D_
_x000D_
_x000D_ 2.5-7.5 mmol/L
_x000D_
_x000D_
_x000D_
_x000D_ _x000D_ Serum Creatinine (µmol/L)
_x000D_
_x000D_
_x000D_ 117
_x000D_
_x000D_
_x000D_ 138
_x000D_
_x000D_
_x000D_ 198
_x000D_
_x000D_
_x000D_ 60-110 µmol/L
_x000D_
_x000D_
_x000D_
_x000D_
_x000D_
Granular and Hyaline Casts are present on Microscopic Urinalysis.
_x000D_
Which Renal Lesion is most likely to be present in this situation?
CorrectIncorrectHint
Ischaemia, typically in Hypotensive hospitalised patients, is the most frequent antecedent to Acute Tubular Necrosis.
_x000D_
Blood Pressure should be maintained in Cardiogenic Shock with Fluids and / or Inotropic Agents
-
Question 115 of 121
115. Question
A 58-year-old gentleman who has received haemodialysis for many years presents with deteriorating discomfort in both shoulders. Past medical history included Bilateral Carpal Tunnel Decompression.
_x000D_
His Investigations Reveal:
_x000D_
_x000D_ _x000D_
_x000D_ _x000D_ Haemoglobin
_x000D_
_x000D_
_x000D_ 105 g/L
_x000D_
_x000D_
_x000D_ (130-180)
_x000D_
_x000D_
_x000D_
_x000D_ _x000D_ Erythrocyte Sedimentation Rate (ESR)
_x000D_
_x000D_
_x000D_ 33 mm/1st hr
_x000D_
_x000D_
_x000D_ (1-10)
_x000D_
_x000D_
_x000D_
_x000D_ _x000D_ Serum C-Reactive Protein (CRP)
_x000D_
_x000D_
_x000D_ 13 mg/L
_x000D_
_x000D_
_x000D_ (<10)
_x000D_
_x000D_
_x000D_
_x000D_ _x000D_ Serum Urate
_x000D_
_x000D_
_x000D_ 0.56 mmol/L
_x000D_
_x000D_
_x000D_ (<0.45)
_x000D_
_x000D_
_x000D_
_x000D_
_x000D_
What is the most likely diagnosis?
CorrectIncorrectHint
The features of shoulder pain associated with a past history of Carpal Tunnel Syndrome in a patient receiving haemodialysis suggest a diagnosis of Beta 2 Microglobulin Amyloidosis.
_x000D_
Amyloid deposits composed of Beta 2 Microglobulin as the major constituent protein are mainly localised in joints and periarticular bone and lead to destructive arthropathy which tends to develop five to ten years after the initiation of dialysis.
_x000D_
Death from Amyloidosis of Gut and Heart may occur after 20 years of dialysis.
_x000D_
-
Question 116 of 121
116. Question
A 56-year-old lady presents with a urinary tract infection and left loin pain, for the fourth time in six months. On each occasion Proteus has been identified.
_x000D_
She is successfully treated for this episode with a one-week course of Trimethoprim, and a Computed Tomography (CT) Scan KUB (Kidneys-Ureters-Bladder) is reported as showing a Left Staghorn Calculus.
_x000D_
Which of the following is the most likely chemical composition of the Staghorn Calculus?
CorrectIncorrectHint
The correct answer is Ammonium Magnesium Phosphate (Struvite Stones).
_x000D_
Struvite Stones are associated with Proteus infection because Proteus creates a situation where Ammonia and Urine pH are both increased, leading to ideal conditions for stone formation.
_x000D_
Urease Inhibitors have been used as medical therapy to reduce stone formation in conjunction with Antibiotic Therapy for underlying Proteus, although surgical stone removal may be required.
_x000D_
Cystine Stones are associated with Cystinuria, Urate Stones with Gout, and Oxalate Stones are associated with Short Bowel Syndrome.
_x000D_
Calcium Phosphate Stones are seen in Renal Tubular Acidosis.
-
Question 117 of 121
117. Question
A 57-year-old gentleman is admitted to Hospital with a three-week history of breathlessness and dry cough.
_x000D_
He has a medical history of longstanding asthma and intermittent tension headaches for which he takes simple analgesia.
_x000D_
On clinical examination, he appears pale and unwell. His Blood Pressure is 170/90 mmHg. Heart Sounds are normal and the Chest is clear. A few Non-Blanching Skin Lesions less than 5 mm in size are found on Lower Limbs.
_x000D_
Investigations Show:
_x000D_
_x000D_ _x000D_
_x000D_ _x000D_ Haemoglobin
_x000D_
_x000D_
_x000D_ 85 g/L
_x000D_
_x000D_
_x000D_ (130-180)
_x000D_
_x000D_
_x000D_
_x000D_ _x000D_ White Blood Cell Count
_x000D_
_x000D_
_x000D_ 11 ×109/L
_x000D_
_x000D_
_x000D_ (4-11)
_x000D_
_x000D_
_x000D_
_x000D_ _x000D_ Neutrophils
_x000D_
_x000D_
_x000D_ 6.8 ×109/L
_x000D_
_x000D_
_x000D_ (1.5-7)
_x000D_
_x000D_
_x000D_
_x000D_ _x000D_ Lymphocytes
_x000D_
_x000D_
_x000D_ 1.9 ×109/L
_x000D_
_x000D_
_x000D_ (1.5-4)
_x000D_
_x000D_
_x000D_
_x000D_ _x000D_ Eosinophils
_x000D_
_x000D_
_x000D_ 1.1 ×109/L
_x000D_
_x000D_
_x000D_ (0.04-0.4)
_x000D_
_x000D_
_x000D_
_x000D_ _x000D_ Erythrocyte Sedimentation Rate (ESR)
_x000D_
_x000D_
_x000D_ 57 mm/hr
_x000D_
_x000D_
_x000D_ (0-15)
_x000D_
_x000D_
_x000D_
_x000D_ _x000D_ Serum C-Reactive Protein (CRP)
_x000D_
_x000D_
_x000D_ 48 mg/L
_x000D_
_x000D_
_x000D_ (<10)
_x000D_
_x000D_
_x000D_
_x000D_ _x000D_ Serum Sodium
_x000D_
_x000D_
_x000D_ 133 mmol/L
_x000D_
_x000D_
_x000D_ (137-144)
_x000D_
_x000D_
_x000D_
_x000D_ _x000D_ Serum Potassium
_x000D_
_x000D_
_x000D_ 4.5 mmol/L
_x000D_
_x000D_
_x000D_ (3.5-4.9)
_x000D_
_x000D_
_x000D_
_x000D_ _x000D_ Serum Creatinine
_x000D_
_x000D_
_x000D_ 655 mmol/lL
_x000D_
_x000D_
_x000D_ (60-110 μmol/L)
_x000D_
_x000D_
_x000D_
_x000D_
_x000D_
Urine Dipstick Shows: Blood 2+ and Protein 3+.
_x000D_
Renal Ultrasound Scan: Both kidneys are normal in size; No evidence of Urinary Obstruction.
_x000D_
What is the most likely diagnosis?
CorrectIncorrectHint
Analgesic Nephropathy is incorrect. This may cause insidious onset of Renal Failure but is not associated with Elevated Inflammatory Markers, Eosinophilia, or Asthma.
_x000D_
Churg-Strauss Syndrome (CSS) is the correct answer. Acute presentation with Glomerulonephritis, Eosinophilia, Skin Vasculitis, and Elevated Inflammatory Markers on a background of long standing history of Asthma makes Churg-Strauss the most likely diagnosis.
_x000D_
Churg-Strauss Syndrome (CSS) is a rare form of Small-Vessel Vasculitis, characterised by Asthma, Allergic Rhinitis and Prominent Peripheral Blood Eosinophilia.
_x000D_
-
Question 118 of 121
118. Question
A 57-year-old gentleman was found on Pedestrian Walkway as stuporous and smelling of alcohol and was taken to Emergency Department by the Police.
_x000D_
Observations in the Emergency Department reveal a Core Temperature of 34°C, a Pulse Rate of 50 Beats Per Minute and Blood Pressure of 120/80 mmHg.
_x000D_
Investigations Reveal:
_x000D_
_x000D_ _x000D_
_x000D_ _x000D_ Serum Creatinine
_x000D_
_x000D_
_x000D_ 325 µmol/L
_x000D_
_x000D_
_x000D_ (60-110)
_x000D_
_x000D_
_x000D_
_x000D_ _x000D_ Gamma Glutamyl Transferase (GGT)
_x000D_
_x000D_
_x000D_ 41 U/L
_x000D_
_x000D_
_x000D_ (10-40)
_x000D_
_x000D_
_x000D_
_x000D_ _x000D_ Serum Aspartate Aminotransferase (AST)
_x000D_
_x000D_
_x000D_ 558 U/L
_x000D_
_x000D_
_x000D_ (1-40)
_x000D_
_x000D_
_x000D_
_x000D_ _x000D_ Serum Lactate Dehydrogenase (LDH)
_x000D_
_x000D_
_x000D_ 1530 U/L
_x000D_
_x000D_
_x000D_ (10-250)
_x000D_
_x000D_
_x000D_
_x000D_ _x000D_ Dipstick Urine
_x000D_
_x000D_
_x000D_ Blood +++
_x000D_
_x000D_
_x000D_
_x000D_
_x000D_
Urine Microscopy showed no cells or organisms.
_x000D_
What is the most likely cause of the Raised Serum Creatinine Concentration?
CorrectIncorrectHint
The elevated Serum Creatinine is most likely due to Rhabdomyolysis as the patient was found unconscious, is hypothermic, and is likely to have sustained muscle injury.
_x000D_
The latter is confirmed by an Elevated Serum Aspartate Aminotransferase (AST) and Lactate Dehydrogenase (LDH) but the normal Serum Gamma-Glutamyl Transpeptidase (GGT) argues against these being released from the liver.
_x000D_
Rhabdomyolysis is strongly suggested by the fact that Urinalysis is strongly positive for blood, whereas Urine Microscopy is negative for Red Blood Cells.
_x000D_
The Positive Urinalysis is caused by Myoglobin, a Muscle Protein released during Muscle Damage; this appears in the Urine and can cause Acute Renal Failure.
-
Question 119 of 121
119. Question
A 56-year-old gentleman with intermittent claudication was found to have renal impairment.
_x000D_
Investigations Revealed:
_x000D_
_x000D_ _x000D_
_x000D_ _x000D_ Serum Creatinine
_x000D_
_x000D_
_x000D_ 185 mmol/L
_x000D_
_x000D_
_x000D_ (60-100)
_x000D_
_x000D_
_x000D_
_x000D_ _x000D_ Urinalysis
_x000D_
_x000D_
_x000D_ Protein++
_x000D_
_x000D_
_x000D_
_x000D_
_x000D_
_x000D_
Renal Ultrasound Scan (USS) revealed a Right Kidney of 7.0 cms and a Left Kidney of 10.0 cms (Normal Dimensions: 10 – 14 cm).
_x000D_
Which investigation should be requested to establish the diagnosis?
CorrectIncorrectHint
This patient has Renovascular Disease with a Right Renal Artery Stenosis.
_x000D_
The Gold Standard for establishing the diagnosis of Renal Artery Stenosis is renal Arteriography and this is commonly performed with Magnetic Resonance Angiography (MRA).
_x000D_
In one-third of cases, the disease is bilateral; 40% may have Peripheral Vascular Disease and there may be Proteinuria.
_x000D_
-
Question 120 of 121
120. Question
A 55-year-old lady is referred to the Nephrology Team with Progressive Renal Impairment and for consideration of Renal Replacement Therapy in the future.
_x000D_
She has a history of Type 1 Diabetes Mellitus (T1DM) since her teens. Her Renal Ultrasound Scan shows normally sized kidneys and her Urine Dipstick is Positive for Protein. A Renal Biopsy is undertaken.
_x000D_
Which of the following findings would be diagnostic of Diabetic Related Kidney Injury?
CorrectIncorrectHint
: Kimmelstiel-Wilson Lesions depict Nodular Glomerulosclerosis which is characteristically seen in Diabetic Kidney Disease.
_x000D_
This option should, therefore, be selected as it is the only Biopsy finding that is specific to Diabetic Related Kidney Disease.
_x000D_
C4d Staining is used for detection of BK Virus after Renal Transplantation and is therefore incorrect.
_x000D_
Congo Red Stain should not be selected as this is used to detect Amyloidosis.
_x000D_
Owl’s Eyes Inclusion depicts the Biopsy Appearance of the Reed-Sternberg Cell in Hodgkin’s Lymphoma or Cytomegalovirus (CMV) Infection, so this is incorrect.
_x000D_
Ring Sideroblasts are Peripheral Blood Film findings in Lead Toxicity, so this is incorrect.
-
Question 121 of 121
121. Question
A 58-year-old gentleman established on Peritoneal Dialysis presents with abdominal pain, fever and cloudy drainage.
_x000D_
A diagnosis of Peritoneal Dialysis (PD) Peritonitis is suspected.
_x000D_
Which of the following laboratory findings is most useful in establishing the diagnosis of Peritoneal Dialysis (PD) Peritonitis?
CorrectIncorrectHint
Peritoneal Dialysis (PD) Peritonitis is an important complication of Peritoneal Dialysis. The delineation describes a typical presentation.
_x000D_
The diagnosis of PD-associated peritonitis requires any two of the following features:
_x000D_
(1) Clinical features consistent with peritonitis, i.e., abdominal pain or cloudy dialysis effluent;
_x000D_
(2) Dialysis effluent white cell count >100/μl (after a dwell time of at least 2 hours), with >50% neutrophils; and
_x000D_
(3) Positive dialysis effluent culture
_x000D_
A high suspicion for the diagnosis is required and empirical treatment is often started.
_x000D_
Peritoneal Dialysis (PD) Fluid White Blood Cell Count (WBCC) of greater than 100/mm3 is diagnostic of Peritoneal Dialysis (PD) Peritonitis and should be selected.
_x000D_
Raised Serum C-Reactive Protein (CRP) may be associated but is not necessarily diagnostic.
_x000D_
A Peritoneal Dialysis (PD) Fluid Neutrophil percentage of greater than 50% and not 10% is in keeping with Peritoneal Dialysis (PD) Peritonitis.
_x000D_
_x000D_
Algorithm for the management of peritoneal dialysis-related peritonitis